Adult Health Test 3 final

Lakukan tugas rumah & ujian kamu dengan baik sekarang menggunakan Quizwiz!

A client with hypertensive emergency is being treated with sodium nitroprusside (Nipride). In a dilution of 50 mg/ 250 mL, how many micrograms of Nipride are in each milliliter? ________________________ mcg.

200 mcg First, calculate the number of milligrams per milliliter: Next, calculate the number of micrograms in each milligram: CN: Pharmacological and parenteral

Essential hypertension would be diagnosed in a 40-year-old male whose blood pressure readings were consistently at or above which of the following? 1. 120/ 90 mm Hg. 2. 130/ 85 mm Hg. 3. 140/ 90 mm Hg. 4. 160/ 80 mm Hg.

3. American Heart Association standards define hypertension as a consistent systolic blood pressure level greater than 140 mm Hg and a consistent diastolic blood pressure level greater than 90 mm Hg.

what causes sinus brady?

dysfunction of SA node

What is fibrinolytic therapy? How does it work?

open up coronary artery to start repurfusion

The auscultatory area in the left midclavicular line at the level of the fifth ICS is the a. aortic area b. mitral area c. tricuspid area d. pulmonic area

B

Diagnostic Studies for acute pyelonephritis

1. History 2. Physical examination a. Palpation for CVA pain 3. Laboratory tests a. Urinalysis b. Urine for culture and sensitivity c. CBC with differential d. Blood culture (if bacteremia is suspected) 4. Ultrasound 5. IVP 6. CT scan 7. If bacteremia is a possibility, close observation and monitoring of vitals are essential. 8. Prompt recognition and treatment of septic shock may prevent irreversible damage or death

Collaborative care of Acute Pyelonephritis

1. Hospitalization for patients with severe infections and complications a. Such as nausea and vomiting with dehydration 2. Signs/symptoms typically improve within 48 to 72 hours after therapy is started. 3. Drug therapy a. Antibiotics i. Parenteral in hospital to rapidly establish high drug levels b. NSAIDs or antipyretic drugs i. Fever ii. Discomfort c. Urinary analgesics 4. Relapses may be treated with 6-week course of antibiotics. 5. Follow-up urine culture and imaging studies 6. Reinfection treated as individual episodes or managed with long-term therapy a. Prophylaxis may be used for recurrent infection.

When computing a heart rate from the ECG tracing, the nurse counts 15 of the small blocks between the R waves of a patient whose rhythm is regular. From these data, the nurse calculates the patient's heart rate to be which of the following? A) a. 60 Beats/min B) b. 75 Beats/min C) c. 100 Beats/min D) d. 150 Beats/min

C Since each small block on the ECG paper represents 0.04 seconds, 1500 of these blocks represents 1 minute. By dividing the number of small blocks (15 in this case) into 1500, the nurse can calculate the heart rate in a patient whose rhythm is regular (in this case, 100).

A patient has a junctional escape rhythm on the monitor. The nurse will expect the patient to have a heart rate of how many beats/minute? A) 15 to 20 B) 20 to 40 C) 40 to 60 D) 60 to 100

C) 40 to 60

In teaching a patient with hypertension about controlling the condition, the nurse recognizes that a. all patients with elevated BP require medication b. it is not necessary to limit salt in the diet if taking a diuretic c. obese persons must achieve a normal weight in order to lower BP d. lifestyle modifications are indicated for all persons with elevated BP

D

The ECG monitor of a patient in the cardiac care unti following an MI indicates ventricular bigeminy with a rate of 50 beats/minute. The nurse would anticipate. a. performing defibrillation b. treatment with IV lidocaine c. insertion of a temporary transvenous pacemaker d. assessing the patient's response to the dysrhythmia

D

The nurse is providing care for a patient who has been admitted to the hospital for the treatment of nephrotic syndrome. Which of the following is a priority nursing assessment in the care of this patient? A Assessment of pain and level of consciousness B Assessment of serum calcium and phosphorus levels C Blood pressure and assessment for orthostatic hypotension D Daily weights and measurement of the patient's abdominal girth

D Peripheral edema is characteristic of nephrotic syndrome and a key nursing responsibility in the care of patients with the disease is close monitoring of abdominal girth, weights, and extremity size. Pain, level of consciousness, and blood pressure are less important in the care of patients with nephrotic syndrome. Abnormal calcium and phosphorus levels are not commonly associated with the etiology of nephrotic syndrome.

4. The nurse is caring for a patient receiving IV furosemide (Lasix) 40 mg and enalapril (Vasotec) 5 mg PO bid for ADHF with severe orthopnea. When evaluating the patient response to the medications, the best indicator that the treatment has been effective is a. weight loss of 2 pounds overnight. b. improvement in hourly urinary output. c. reduction in systolic BP. d. decreased dyspnea with the head of the bed at 30 degrees.

D Rationale: Because the patient's major clinical manifestation of ADHF is orthopnea (caused by the presence of fluid in the alveoli), the best indicator that the medications are effective is a decrease in crackles. The other assessment data also may indicate that diuresis or improvement in cardiac output have occurred but are not as useful in evaluating this patient's response. Cognitive Level: Application Text Reference: p. 825 Nursing Process: Evaluation NCLEX: Physiological Integrity

Which information will the nurse include when teaching a patient who is scheduled to have a permanent pacemaker inserted for treatment of chronic atrial fibrillation with slow ventricular response? A) The pacemaker prevents or minimizes ventricular irritability. B) The pacemaker paces the atria at rates up to 500 impulses/minute. C) The pacemaker discharges if ventricular fibrillation and cardiac arrest occur. D) The pacemaker stimulates a heart beat if the patient's heart rate drops too low.

D) The pacemaker stimulates a heart beat if the patient's heart rate drops too low. The permanent pacemaker will discharge when the ventricular rate drops below the setrate. The pacemaker will not decrease ventricular irritability or discharge if the patient develops ventricular fibrillation. Since the patient has a slow ventricular rate, overdrive pacing will not be used.

After the nurse teaches the patient with stage 1 hypertension about diet modifications that should be implemented, which diet choice indicates that the teaching has been effective? A) The patient avoids eating nuts or nut butters. B) The patient restricts intake of dietary protein. C) The patient has only one cup of coffee in the morning. D) The patient has a glass of low-fat milk with each meal.

D) The patient has a glass of low-fat milk with each meal.

A, B, C, D

The nursing management of a patient of a patient is sickle cell crisis includes (select all that apply): A. monitoring CBC B. blood transfusion if required and iron chelation C. optimal pain management and oxygen therapy D. rest as needed and DVT prophylaxis E. administration of IV iron and diet high in iron content

Which complication of chronic kidney disease is treated with erythropoietin (EPO)? a. Anemia b. Hypertension c. Hyperkalemia d. Mineral and bone disorder

a. Erythropoietin is used to treat anemia, as it stimulates the bone marrow to produce red blood cells.

Metabolic acidosis occurs in the oliguric phase of AKI as a result of impairment of a. ammonia synthesis. b. excretion of sodium. c. excretion of bicarbonate. d. conservation of potassium.

a. Metabolic acidosis occurs in AKI because the kidneys cannot synthesize ammonia or excrete acid products of metabolism, resulting in an increased acid load. Sodium is lost in urine because the kidneys cannot conserve sodium. Impaired excretion of potassium results in hyperkalemia. Bicarbonate is normally generated and reabsorbed by

Planning for Acute Pyelonephritis

a. Patient will have i. Relief of pain ii. Normal body temperature iii. No complications iv. Normal renal function v. No recurrence of symptoms

A patient with AKI has a serum potassium level of 6.7 mEq/L (6.7 mmol/L) and the following arterial blood gas results: pH 7.28, PaCO2 30 mm Hg, PaO2 86 mm Hg, HCO3 − 18 mEq/L (18 mmol/L). The nurse recognizes that treatment of the acid-base problem with sodium bicarbonate would cause a decrease in which value? a. pH b. Potassium level c. Bicarbonate level d. Carbon dioxide level

b. During acidosis, potassium moves out of the cell in exchange for H+ ions, increasing the serum potassium level. Correction of the acidosis with sodium bicarbonate will help to shift the potassium back into the cells. A decrease in pH and the bicarbonate and PaCO2 levels would indicate worsening acidosis.

When caring for the patient with interstitial cystitis, what can the nurse teach the patient to do? a. Avoid foods that make the urine more alkaline. b. Use high-potency vitamin therapy to decrease the autoimmune effects of the disorder. c. Always keep a voiding diary to document pain, voiding frequency, and patterns of nocturia. d. Use the dietary supplement calcium glycerophosphate (Prelief) to decrease bladder irritation.

d. Calcium glycerophosphate (Prelief) alkalinizes the urine and can help to relieve the irritation from acidic foods. A diet low in acidic foods is recommended and if a multivitamin is used, high-potency vitamins should be avoided because these products may irritate the bladder. A voiding diary is useful in diagnosis but does not need to be kept indefinitely.

*1. In teaching a patient with pyelonephritis about the disorder, the nurse informs the paitent that the organisms that cause pyelonephritis most commonly reach the kidneys through* a. the bloodstream b.the lymphatic system c. a descending infection d. an ascending infection

d. an ascending infection

A nurse is caring for a group of hospitalized clients. Which client is at greatest risk for infection and sepsis? 1. 18-year-old who had an emergency splenectomy 2. 22-year-old with recently diagnosed sickle cell anemia 3. 38-year-old with hemolytic anemia 4. 40-year-old alcoholic with liver disease

1. Removal of the spleen causes the client to have reduced immune function. Without a spleen, people are less able to remove disease-causing organisms. Sickle cell anemia causes pain and discomfort owing to the changed cell morphology. Acute pain, especially at joints, is the greatest threat to this client. A low red blood cell (RBC) count can contribute to a client's risk for infection, but this client is more at risk for low oxygen levels and ensuing fatigue.The liver plays a role in blood coagulation. This client is more at risk for coagulation problems than for infection.

Etiology and Pathophysiology of Acute Pyelonephritis

- Urosepsis ***BAD*** a. Systemic infection from urologic source (gets into bloodstream) b. Prompt diagnosis/treatment critical i. Can lead to septic shock and death 1. Septic shock: Outcome of unresolved bacteremia involving gram-negative organism ii. Want to intervene before patient reaches this point iii. BP drops iv. Spread n body 2. Usually begins with colonization and infection of lower tract via ascending urethral route 3. Frequent causes a. Escherichia coli (most common) b. Proteus c. Klebsiella d. Enterobacter 4. Preexisting factor usually present a. Vesicoureteral reflux i. Backward movement of urine from lower to upper urinary tract (any disease that causes this can cause urosepsis and pyelonephritis and the longer the catheter is in increases these chances as well) b. Dysfunction of lower urinary tract i. Obstruction from BPH ii. Stricture iii. Urinary stone 5. Commonly starts in renal medulla and spreads to adjacent cortex a. Early intervention is important, do not want this to reoccur 6. Recurring episodes lead to scarred, poorly functioning kidney and chronic pyelonephritis.

Diagnosing a UTI

-Dipstick -Urinalysis (urine for C and S) -Voided midstream collection, Front to back wipe, urinate some into toilet and after initial, begin collecting urine -OR catheterized specimen collected -Urine is checked for sensitivity to certain antibiotics; See which antibiotic will work best, Combination drug Bactrim (Foley antagonist and sulfonamide)—uncomplicated, Other drugs Macrodim, ampicillin, amoxicillin—all in text book, Simple infection—Must take antibiotics 3 days for women and 7-14 days for men, Complicated infection—take antibiotics 7-14 days for both sexes - Imaging studies-IVP, CT scans (not done in uncomplicated infections

The nurse is developing a care plan with an older adult and is instructing the client that hypertension can be a silent killer. The nurse should instruct the client to be aware of signs and symptoms of other system failures and encourage the client to report signs of which of the following diseases that are often a result of undeteced high blood pressure? 1. Cerebrovascular accidents (CVAs). 2. Liver disease. 3. Myocardial infarction. 4. Pulmonary disease.

1. Hypertension is referred to as the silent killer for adults, because until the adult has significant damage to other systems, the hypertension may go undetected. CVAs can be related to long-term hypertension. Liver or pulmonary disease is not generally associated with hypertension. Myocardial infarction is generally related to coronary artery disease.

When teaching a client about propranolol hydrochloride, the nurse should base the information on the knowledge that propranolol: 1. Blocks beta-adrenergic stimulation and thus causes decreased heart rate, myocardial contractility, and conduction. 2. Increases norepinephrine secretion and thus decreases blood pressure and heart rate. 3. Is a potent arterial and venous vasodilator that reduces peripheral vascular resistance and lowers blood pressure. 4. Is an angiotensin-converting enzyme inhibitor that reduces blood pressure by blocking the conversion of angiotensin I to angiotensin II.

1. Propranolol is a beta-adrenergic blocking agent. Actions of propranolol include reducing heart rate, decreasing myocardial contractility, and slowing conduction. Propranolol does not increase norepinephrine secretion, cause vasodilation, or block conversion of angiotensin I to angiotensin II.

The client has had hypertension for 20 years. The nurse should assess the client for? 1. Renal insufficiency and failure. 2. Valvular heart disease. 3. Endocarditis. 4. Peptic ulcer disease.

1. Renal disease, including renal insufficiency and failure, is a complication of hypertension. Effective treatment of hypertension assists in preventing this complication. Valvular heart disease, endocarditis, and peptic ulcer disease are not complications of hypertension.

Clinical manifestations of Acute Pyelonephritis

1. Mild fatigue 2. Chills 3. Fever 4. Vomiting 5. Malaise 6. Flank pain (usually on affected side) 7. Lower urinary tract symptoms characteristic of cystitis 8. Costovertebral tenderness usually present on affected side 9. Manifestations usually subside in a few days, even without therapy. a. Bacteriuria and pyuria still persist.

A patient with a recent diagnosis of heart failure has been prescribed furosemide (Lasix) in an effort to A. reduce preload. B. decrease afterload. C. increase contractility. D. promote vasodilation.

ANS: A Diuretics such as furosemide are used in the treatment of heart failure to mobilize edematous fluid, reduce pulmonary venous pressure, and reduce preload. They do not directly influence contractility, afterload, or vessel tone. Reference: 804

A P wave on an ECG represents an impulse A. arising at the SA node and repolarizing the atria. B. arising at the SA node and depolarizing the atria. C. arising at the AV node and depolarizing the atria. D. arising at the AV node and spreading to the bundle of His.

ANS: B The first wave, P, begins with firing of the SA node and represents depolarization of the fibers of the atria. Reference: 717

A patient with a tricuspid valve disorder has impaired blood flow between the A. vena cava and right atrium. B. left atrium and left ventricle. C. right atrium and right ventricle. D. right ventricle and pulmonary artery.

ANS: C The tricuspid valve is located between the right atrium and the right ventricle. Reference: 716

You are monitoring the electrocardiogram of a patient admitted with ACS. Which ECG characteristics most suggest ischemia? A. Sinus rhythm with a pathologic Q wave B. Sinus rhythm with an elevated ST segment C. Sinus rhythm with a depressed ST segment D. Sinus rhythm with premature atrial contractions

ANS: C The typical ECG changes seen in myocardial ischemia include ST-segment depression or T-wave inversion, or both. Reference: 837

The nurse expects the patient with restrictive cardiomyopathy to present with which dysrhythmia? A. First-degree heart block B. Sinus dysrhythmia C. Supraventricular tachycardia D. Third-degree heart block

ANS:C The most common dysrhythmias are supraventricular tachycardia and atrioventricular block. The other dysrhythmias do not usually occur. Reference: 863

After reviewing the laboratory test results, the nurse calls the health care provider about which client? 1.44-year-old receiving warfarin (Coumadin) with an international normalized ratio (INR) of 3.0 2. 46-year-old with a fever and a white blood cell (WBC) count of 500/µl 3. 49-year-old with hemophilia and a platelet count of 150,000/mm3 4. 52-year-old who has had a hemorrhage with a reticulocyte count of 0.8%

2. This client is neutropenic and is at risk for sepsis unless interventions such as medications to improve WBC level and antibiotics are prescribed. The INR of 3.0 indicates a therapeutic Coumadin level. This platelet count is normal. An elevated reticulocyte count is expected after hemorrhage.

A nurse is teaching a client about what to expect during a bone marrow biopsy. Which statement by the nurse accurately describes the procedure? 1. "The doctor will place a small needle in your back and will withdraw some fluid." 2. "You may experience a crunching sound or a scraping sensation as the needle punctures your bone." 3. "You will be alone because the procedure is a sterile one; we cannot allow additional people to contaminate the area." 4. "You will be sedated, so you will not be aware of anything."

2. This description is accurate. Proper expectations minimize the client's fear during the procedure. A very large-bore needle is used for a bone marrow biopsy, not a small needle. The puncture is made in the hip or in the sternum, not the back. The nurse, or sometimes a family member, is available to the client for support during a bone marrow biopsy. The procedure is sterile at the site of the biopsy, but others can be present without contamination at the site. A local anesthetic agent is injected into the skin around the site. The client may also receive a mild tranquilizer or a rapid-acting sedative (such as lorazepam [Ativan]) but will not be completely sedated. Clients are aware of what is happening during the procedure.

The client realizes the importance of quitting smoking, and the nurse develops a plan to help the client achieve this goal. Which of the following nursing interventions should be the initial step in this plan? 1. Review the negative effects of smoking on the body. 2. Discuss the effects of passive smoking on environmental pollution. 3. Establish the client's daily smoking pattern. 4. Explain how smoking worsens high blood pressure.

3. A plan to reduce or stop smoking begins with establishing the client's personal daily smoking pattern and activities associated with smoking. It is important that the client understands the associated health and environmental risks, but this knowledge has not been shown to help clients change their smoking behavior.

How should the aspirin been taken

160mg or 325mg Use chewable aspirin for fast effect

A client treated for hypertension with furosemide (Lasix), atenolol (Tenormin), and ramipril (Altace) develops a second degree heart block Mobitz type 1. Which of the following actions should the nurse take? 1. Administer a 250 mL fluid bolus. 2. Withhold the atenolol. 3. Prepare for cardioversion. 4. Set up for an arterial line.

2. The client may be asymptomatic and the underlying cause should be assessed. Drugs that block the AV node should be avoided, such as beta blockers (Atenolol), calcium channel blockers, digoxin, and amiodarone. Symptomatic clients are treated with atropine and transcutaneous pacing. There is no indication for a fluid bolus, cardioversion, or arterial line.

Acute Pyelonephritis

2. Acute Pyelonephritis: inflammation of the kidney 3. Similar treatment as UTI but more significant symptoms 4. Usually a bacteria or fungal issue 5. 250,000 cases of this and more than 1000 hospitalizations 6. Women 5x more likely to contract than men 7. Women have lower mortality rate than men 8. Occurs in about 1-2% of women—can lead to premature labor and low birth weight infants 9. Can happen with urinary reflux from lower UT to upper 10. Clinical symptoms: extremely painful, patient is sick, vomiting, chills, fever, bacteria and hematuria and WBC in urine 11. Blood testing: leukocytosis with left shift (bacteria) 12. Studies: renal ultrasound, IVP, VCUG, CAT scan, urine cultures checks 13. Mild symptoms may not require hospital stay but treatable outpatient 14. Severe symptoms: may require hospital stay, IV antibiotics and fluids with analgesic

Which client does the medical unit charge nurse assign to an LPN/LVN? 1.23-year-old scheduled for a bone marrow biopsy with conscious sedation 2. 35-year-old with a history of a splenectomy and a temperature of 100.9° F (38.3° C) 3. 48-year-old with chronic microcytic anemia associated with alcohol use 4. 62-year-old man with atrial fibrillation and an international normalized ratio (INR) of 6.6

3. This client has a chronic condition that is not considered life threatening. A bone marrow biopsy with conscious sedation requires more complex assessment or nursing care and should be assigned to RN staff members. A history of a splenectomy and a temperature require more complex assessment or nursing care and should be assigned to RN staff members. Atrial fibrillation and an international normalized ratio (INR) of 6.6 require more complex assessment or nursing care and should be assigned to RN staff members.

In which type of dialysis does the patient dialyze during sleep and leave the fluid in the abdomen during the day? a. Long nocturnal hemodialysis b. Automated peritoneal dialysis (APD) c. Continuous venovenous hemofiltration (CVVH) d. Continuous ambulatory peritoneal dialysis (CAPD)

b. Automated peritoneal dialysis (APD) is the type of dialysis in which the patient dialyzes during sleep and leaves the fluid in the abdomen during the day. Long nocturnal hemodialysis occurs while the patient is sleeping and is done up to six times per week. Continuous venovenous hemofiltration (CVVH) is a type of continuous renal replacement therapy used to treat AKI. Continuous ambulatory peritoneal dialysis (CAPD) is dialysis that is done with exchanges of 1.5 to 3 L of dialysate at least four times daily.

A teaching plan developed by the nurse for the patient with a new ileal conduit includes instructions to do what? a. Clean the skin around the stoma with alcohol every day. b. Use a wick to keep the skin dry during appliance changes. c. Use sterile supplies and technique during care of the stoma. d. Change the appliance every day and wash it with soap and warm water.

b. Because the stoma continuously drains urine, a wick formed of a rolled-up 4 × 4 gauze or a tampon is held against the stoma to absorb the urine while the skin is cleaned and a new appliance is attached. The skin is cleaned with warm water only because soap and other agents cause drying and irritation and clean, not sterile, technique is used. The appliance should be left in place for as long as possible before it loosens and allows leakage onto the skin, perhaps up to 14 days.

What is the most serious electrolyte disorder associated with kidney disease? a. Hypocalcemia b. Hyperkalemia c. Hyponatremia d. Hypermagnesemia

b. Hyperkalemia can lead to life-threatening dysrhythmias. Hypocalcemia leads to an accelerated rate of bone remodeling and potentially to tetany. Hyponatremia may lead to confusion. Elevated sodium levels lead to edema, hypertension, and heart failure. Hypermagnesemia may decrease reflexes, mental status, and blood pressure.

Priority Decision: A dehydrated patient is in the Injury stage of the RIFLE staging of AKI. What would the nurse first anticipate in the treatment of this patient? a. Assess daily weight b. IV administration of fluid and furosemide (Lasix) c. IV administration of insulin and sodium bicarbonate d. Urinalysis to check for sediment, osmolality, sodium, and specific gravity

b. Injury is the stage of RIFLE classification when urine output is less than 0.5 mL/kg/hr for 12 hours, the serum creatinine is increased times two or the glomerular filtration rate (GFR) is decreased by 50%. This stage may be reversible by treating the cause or, in this patient, the dehydration by administering IV fluid and a low dose of a loop diuretic, furosemide (Lasix). Assessing the daily weight will be done to monitor fluid changes but it is not the first treatment the nurse should anticipate. IV administration of insulin and sodium bicarbonate would be used for hyperkalemia. Checking the urinalysis will help to determine if the AKI has a prerenal, intrarenal, or postrenal cause by what is seen in the urine but with this patient's dehydration, it is thought to be prerenal to begin treatment.

To prevent the most common serious complication of PD, what is important for the nurse to do? a. Infuse the dialysate slowly. b. Use strict aseptic technique in the dialysis procedures. c. Have the patient empty the bowel before the inflow phase. d. Reposition the patient frequently and promote deep breathing.

b. Peritonitis is a common complication of peritoneal dialysis (PD) and may require catheter removal and termination of dialysis. Infection occurs from contamination of the dialysate or tubing or from progression of exit-site or tunnel infections and strict sterile technique must be used by health professionals as well as the patient to prevent contamination. Too-rapid infusion may cause shoulder pain and pain may be caused if the catheter tip touches the bowel. Difficulty breathing, atelectasis, and pneumonia may occur from pressure of the fluid on the diaphragm, which may be prevented by elevating the head of the bed and promoting repositioning and deep breathing.

The female patient with a UTI also has renal calculi. The nurse knows that these are most likely which type of stone? a. Cystine b. Struvite c. Uric acid d. Calcium phosphate

b. Struvite calculi are most common in women and always occur with UTIs. They are also usually large staghorn type.

A female patient with a UTI has a nursing diagnosis of risk for infection related to lack of knowledge regarding prevention of recurrence. What should the nurse include in the teaching plan instructions for this patient? a. Empty the bladder at least 4 times a day. b. Drink at least 2 quarts of water every day. c. Wait to urinate until the urge is very intense. d. Clean the urinary meatus with an antiinfective agent after voiding.

b. The bladder should be emptied at least every 3 to 4 hours. Fluid intake should be increased to about 2000 mL/ day without caffeine, alcohol, citrus juices, and chocolate drinks, because they are potential bladder irritants. Cleaning the urinary meatus with an antiinfective agent after voiding will irritate the meatus but the perineal area should be wiped from front to back after urination and defecation to prevent fecal contamination of the meatus.

A 68-year-old man with a history of heart failure resulting from hypertension has AKI as a result of the effects of nephrotoxic diuretics. Currently his serum potassium is 6.2 mEq/L (6.2 mmol/L) with cardiac changes, his BUN is 108 mg/dL (38.6 mmol/L), his serum creatinine is 4.1 mg/dL (362 mmol/L), and his serum HCO3 − is 14 mEq/L (14 mmol/L). He is somnolent and disoriented. Which treatment should the nurse expect to be used for him? a. Loop diuretics b. Renal replacement therapy c. Insulin and sodium bicarbonate d. Sodium polystyrene sulfonate (Kayexalate)

b. This patient has at least three of the six common indications for renal replacement therapy (RRT), including (1) high potassium level, (2) metabolic acidosis, and (3) changed mental status. The other indications are (4) volume overload, resulting in compromised cardiac status (this patient has a history of hypertension), (5) BUN greater than 120 mg/dL, and (6) pericarditis, pericardial effusion, or cardiac tamponade. Although the other treatments may be used, they will not be as effective as RRT for this older patient. Loop diuretics and increased fluid are used if the patient is dehydrated. Insulin and sodium bicarbonate can be used to temporarily drive the potassium into the cells. Sodium polystyrene sulfonate (Kayexalate) is used to actually decrease the amount of potassium in the body.

*3. The immunologic mechanisms involved in acute poststreptococal glomerulonephritis include:* a. tubular blocking by precipitates of bacteria and antibody reactions b. deposition of immune complexes and complement along the GBM c. thickening of the GBM from autoimmune microangiopathic changes d. destruction of glomeruli by proteolytic enzymes contained in the GBM

b. deposition of immune complexes and complement along the GBM

*2. The nurse teaches the female paitent who has frequent UTIs that she should* a. take tub baths with bubble bath b.urinate before and after sexual intercourse c.take prophylactic sufonamides for the rest of her life d. restrict fluid intake to prevent the need for frequent voiding

b.urinate before and after sexual intercourse

A nurse is assessing the nutritional status of a client with anemia. How does the nurse obtain information about the client's diet? 1.Asks the client to rate his or her diet on a scale of 1 (poor) to 10 (excellent) 2. Determines who prepares the client's meals and plans an interview with him or her 3. From a prepared list, finds out the client's food preferences 4. Has the client write down everything he or she has eaten for the past week

4. This method is the most accurate way to find out what the client likes and dislikes, as well as what the client has been eating. It will provide information about "junk" food intake, as well as about the client's protein, vitamin, and mineral intake. the third option method of dietary analysis provides a list of what the client enjoys eating, not necessarily what the client has been eating. The client may like steak but may be unable to afford it.

A client is scheduled for a bone marrow aspiration. What does the client's nurse do before taking the client to the treatment room for the biopsy? 1. Cleans the biopsy site with an antiseptic or povidone-iodine (Betadine) 2. Holds the client's hand and asks about concerns 3. Reviews the client's platelet (thrombocyte) count 4. Verifies that the client has given informed consent

4. Verifying informed consent must be done before the procedure can be performed. A signed permit must be on the client's chart. Cleaning the biopsy site is done before the procedure but is not the first thing that should be done. It is not done until consent is verified. It will be done just before the procedure is performed.

The community health nurse is planning health promotion teaching targeted at preventing coronary artery disease (CAD). Which of the following ethnic groups would the nurse select as the highest priority for this intervention? A) a. White male B) b. Hispanic male C) c. African American male D) d. Native American female

A

The nurse is caring for a patient with hypertension who is scheduled to receive a dose of atenolol (Tenormin). The nurse should withhold the dose and consult the prescribing physician for which of the following vital signs taken just before administration? A) a. Pulse 48 B) b. Respirations 24 C) c. Blood pressure 118/74 D) d. Oxygen saturation 93%

A

The nurse is providing care for a patient who has decreased cardiac output related to heart failure. The nurse recognizes that cardiac output is A) a. Calculated by multiplying the patient's stroke volume by the heart rate. B) b. The average amount of blood ejected during one complete cardiac cycle. C) c. Determined by measuring the electrical activity of the heart and the patient's heart rate. D) d. The patient's average resting heart rate multiplied by the patient's mean arterial blood pressure.

A

Two risk factors for coronary artery disease that increase the workload of the heart and increase myocardial oxygen demand are a. hypertension and cigarette smoking. b. obesity and smokeless tobacco use. c. elevated serum lipids and diabetes mellitus. d. physical inactivity and elevated homocysteine levels.

A

When a person's blood pressure rises, the homeostatic mechanism to compensate for an elevation involves stimulation of a. baroreceptors that inhibit the sympathetic nervous system, causing vasodilation b. chemoreceptors that inhibit the sympathethic nervous system causing vasodilation c. baroreceptors that inhibit the parasympathetic nervous system, causing vasodilation d. chemoreceptors that stimulate the sympathetic nervous system, causing an increased heart rate.

A

When planning emergent care for a patient with a suspected MI, the nurse will anticipate administration of A) a. Oxygen, nitroglycerin, aspirin, and morphine. B) b. Oxygen, furosemide (Lasix), nitroglycerin, and meperidine. C) c. Aspirin, nitroprusside (Nipride), dopamine (Intropin), and oxygen. D) d. Nitroglycerin, lorazepam (Ativan), oxygen, and warfarin (Coumadin).

A

Which of the following individuals would the nurse identify as having the highest risk for CAD? A) a. A 45-year-old depressed male with a high-stress job B) b. A 60-year-old male with below normal homocysteine levels C) c. A 54-year-old female vegetarian with elevated high-density lipoprotein (HDL) levels D) d. A 62-year-old female who has a sedentary lifestyle and body mass index (BMI) of 23 kg/m2

A

A patient has sought care following a syncopal episode of unknown etiology. Which of the following nursing actions should the nurse prioritize in the patient's subsequent diagnostic workup? A) a. Preparing to assist with a head-up tilt-test B) b. Assessing the patient's knowledge of pacemakers C) c. Preparing an intravenous dose of a b-adrenergic blocker D) d. Teaching the patient about the role of antiplatelet aggregators

A A head-up tilt-test is a common component of the diagnostic workup following episodes of syncope. IV b-blockers are not indicated and addressing pacemakers is premature and inappropriate at this stage of diagnosis. Patient education surrounding antiplatelet aggregators is not directly relevant to the patient's syncope.

For which of the following dysrhythmias is defibrillation primarily indicated? A) a. Ventricular fibrillation B) b. Third-degree AV block C) c. Uncontrolled atrial fibrillation D) d. Ventricular tachycardia with a pulse

A Defibrillation is always indicated in the treatment of ventricular fibrillation. Drug treatments are normally used in the treatment of uncontrolled atrial fibrillation and for ventricular tachycardia with a pulse (if the patient is stable). Otherwise, synchronized cardioversion is used (as long as the patient has a pulse). Pacemakers are the treatment of choice for third-degree heart block.

In preparing a patient for an intravenous pyelogram (IVP), the nurse would expect to: A administer a cathartic or enema. B assess patient for allergies to penicillin. C keep the patient NPO for 4 hours preprocedure. D advise the patient that a metallic taste may occur during procedure.

A Nursing responsibilities in caring for a patient undergoing an IVP include administration of a cathartic or enema to empty the colon of feces and gas. The nurse will also assess the patient for iodine sensitivity, keep the patient NPO for 8 hours preprocedure, and advise the patient that warmth, a flushed face, and a salty taste during injection of contrast material may occur.

Which of the following statements by the nurse regarding continuous ambulatory peritoneal dialysis (CAPD) would be of highest priority when teaching a patient new to this procedure? A "It is essential that you maintain aseptic technique to prevent peritonitis." B "You will be allowed a more liberal protein diet once you complete CAPD." C "It is important for you to maintain a daily written record of blood pressure and weight." D "You will need to continue regular medical and nursing follow-up visits while performing CAPD."

A Peritonitis is a potentially fatal complication of peritoneal dialysis, and thus it is imperative to teach the patient methods of preventing this from occurring. Although the other teaching statements are accurate, they do not have the potential for mortality as does the peritonitis, thus making that nursing action of highest priority.

A patient with a history of recurrent urinary tract infections has been scheduled for a cystoscopy. Which of the following teaching points should the nurse emphasize before the procedure? A "You might have pink-tinged urine and burning after your cystoscopy." B "You'll need to refrain from eating or drinking after midnight the day before the test." C "You'll require a urinary catheter inserted before the cystoscopy and it will be in place for a few days." D "The morning of the test, the nurse will ask you to drink some water that contains a contrast solution."

A Pink-tinged urine, burning, and frequency are common following a cystoscopy. The patient does not need to be NPO prior to the test and a contrast solution is unnecessary. A cystoscopy does not always necessitate catheterization before or after the procedure.

6. A patient in the intensive care unit with ADHF complains of severe dyspnea and is anxious, tachypneic, and tachycardic. All these medications have been ordered for the patient. The first action by the nurse will be to a. administer IV morphine sulfate 2 mg. b. give IV diazepam (Valium) 2.5 mg. c. increase dopamine (Intropin) infusion by 2 mcg/kg/min. d. increase nitroglycerin (Tridil) infusion by 5 mcg/min.

A Rationale: Morphine improves alveolar gas exchange, improves cardiac output by reducing ventricular preload and afterload, decreases anxiety, and assists in reducing the subjective feeling of dyspnea. Diazepam may decrease patient anxiety, but it will not improve the cardiac output or gas exchange. Increasing the dopamine may improve cardiac output but will also increase the heart rate and myocardial oxygen consumption. Nitroglycerin will improve cardiac output and may be appropriate for this patient, but it will not directly reduce anxiety and will not act as quickly as morphine to decrease dyspnea. Cognitive Level: Analysis Text Reference: pp. 828-829 Nursing Process: Implementation NCLEX: Physiological Integrity

21. A patient with ADHF who is receiving nesiritide (Natrecor) asks the nurse how the medication will work to help improve the symptoms of dyspnea and orthopnea. The nurse's reply will be based on the information that nesiritide will a. dilate arterial and venous blood vessels, decreasing ventricular preload and afterload. b. improve the ability of the ventricular myocardium to contract, strengthening contractility. c. enhance the speed of impulse conduction through the heart, increasing the heart rate. d. increase calcium sensitivity in vascular smooth muscle, boosting systemic vascular resistance.

A Rationale: Nesiritide, a recombinant form of BNP, causes both arterial and venous vasodilation, leading to reductions in preload and afterload. Inotropic medications, such as dopamine and dobutamine, may be used in ADHF to improve ventricular contractility. Nesiritide does not increase impulse conduction or calcium sensitivity in the heart. Cognitive Level: Application Text Reference: p. 829 Nursing Process: Implementation NCLEX: Physiological Integrity

1. A patient with a history of chronic heart failure is admitted to the emergency department with severe dyspnea and a dry, hacking cough. The patient has pitting edema in both ankles, blood pressure (BP) of 170/100, an apical pulse rate of 92, and respirations 28. The most important assessment for the nurse to accomplish next is to a. auscultate the lung sounds. b. assess the orientation. c. check the capillary refill. d. palpate the abdomen.

A Rationale: When caring for a patient with severe dyspnea, the nurse should use the ABCs to guide initial care. This patient's severe dyspnea and cough indicate that acute decompensated heart failure (ADHF) is occurring. ADHF usually manifests as pulmonary edema, which should be detected and treated immediately to prevent ongoing hypoxemia and cardiac/respiratory arrest. The other assessments will provide useful data about the patient's volume status and should also be accomplished rapidly, but detection (and treatment) of fluid-filled alveoli is the priority. Cognitive Level: Application Text Reference: pp. 824-825 Nursing Process: Assessment NCLEX: Physiological Integrity

At a clinic visit, the nurse provides dietary teaching for a 56-year-old woman who was recently hospitalized with an exacerbation of chronic heart failure. The nurse determines that teaching is successful if the patient makes which statement? A "I will limit the amount of milk and cheese in my diet." B "I can add salt when cooking foods but not at the table." C "I will take an extra diuretic pill when I eat a lot of salt." D "I can have unlimited amounts of foods labeled as reduced sodium ."

A "I will limit the amount of milk and cheese in my diet." Milk products should be limited to 2 cups per day for a 2500-mg sodium-restricted diet. Salt should not be added during food preparation or at the table. Diuretics should be taken as prescribed (usually daily) and not based on sodium intake. Foods labeled as reduced sodium contain at least 25% less sodium than regular.

A70-year-old woman with chronic heart failure and atrial fibrillation asks the nurse why warfarin (Coumadin) has been prescribed for her to continue at home. Which response by the nurse is accurate? A "The medication prevents blood clots from forming in your heart." B "The medication dissolves clots that develop in your coronary arteries." C "The medication reduces clotting by decreasing serum potassium levels." D "The medication increases your heart rate so that clots do not form in your heart."

A "The medication prevents blood clots from forming in your heart." Chronic heart failure causes enlargement of the chambers of the heart and an altered electrical pathway, especially in the atria. When numerous sites in the atria fire spontaneously and rapidly, atrial fibrillation occurs. Atrial fibrillation promotes thrombus formation within the atria with an increased risk of stroke and requires treatment with cardioversion, antidysrhythmics, and/or anticoagulants. Warfarin is an anticoagulant that interferes with hepatic synthesis of vitamin K-dependent clotting factors.

A patient admitted with heart failure appears very anxious and complains of shortness of breath. Which nursing actions would be appropriate to alleviate this patient's anxiety (select all that apply)? A Administer ordered morphine sulfate. B Position patient in a semi-Fowler's position. C Position patient on left side with head of bed flat. D Instruct patient on the use of relaxation techniques. E Use a calm, reassuring approach while talking to patient.

A Administer ordered morphine sulfate. B Position patient in a semi-Fowler's position. D Instruct patient on the use of relaxation techniques. E Use a calm, reassuring approach while talking to patient. Morphine sulfate reduces anxiety and may assist in reducing dyspnea. The patient should be positioned in semi-Fowler's position to improve ventilation that will reduce anxiety. Relaxation techniques and a calm reassuring approach will also serve to reduce anxiety.

The patient has heart failure (HF) with an ejection fraction of less than 40%. What core measures should the nurse expect to include in the plan of care for this patient (select all that apply)? A Left ventricular function is documented. B Controlling dysrhythmias will eliminate HF. C Prescription for digoxin (Lanoxin) at discharge D Prescription for angiotensin-converting enzyme (ACE) inhibitor at discharge E Education materials about activity, medications, weight monitoring, and what to do if symptoms worsen

A Left ventricular function is documented. D Prescription for angiotensin-converting enzyme (ACE) inhibitor at discharge E Education materials about activity, medications, weight monitoring, and what to do if symptoms worsen The Joint Commission has identified these three core measures for heart failure patients. Although controlling dysrhythmias will improve CO and workload, it will not eliminate HF. Prescribing digoxin for all HF patients is no longer done because there are newer effective drugs and digoxin toxicity occurs easily related to electrolyte levels and the therapeutic range must be maintained.

The patient with chronic heart failure is being discharged from the hospital. What information should the nurse emphasize in the patient's discharge teaching to prevent progression of the disease to ADHF? A Take medications as prescribed. B Use oxygen when feeling short of breath. C Only ask the physician's office questions. D Encourage most activity in the morning when rested.

A Take medications as prescribed. The goal for the patient with chronic HF is to avoid exacerbations and hospitalization. Taking the medications as prescribed along with nondrug therapies such as alternating activity with rest will help the patient meet this goal. If the patient needs to use oxygen at home, it will probably be used all the time or with activity to prevent respiratory acidosis. Many HF patients are monitored by a care manager or in a transitional program to assess the patient for medication effectiveness and monitor for patient deterioration and encourage the patient. This nurse manager can be asked questions or can contact the health care provider if there is evidence of worsening HF.

A patient with heart failure has a new order for captopril (Capoten) 12.5 mg PO. After administering the first dose and teaching the patient about captopril, which statement by the patient indicates that teaching has been effective? A) a. "I will call for help when I need to get up to use the bathroom." B) b. "I will be sure to take the medication after eating something." C) c. "I will need to include more high-potassium foods in my diet." D) d. "I will expect to feel more short of breath for the next few days."

A) "I will call for help when I need to get up to use the bathroom."

Which action by a new nurse who is caring for a patient who has just had an implantable cardioverter-defibrillator (ICD) inserted indicates a need for more education about care of patients with ICDs? A) The nurse assists the patient to do active range of motion exercises for all extremities. B) The nurse assists the patient to fill out the application for obtaining a Medic Alert ID and bracelet. C) The nurse gives atenolol (Tenormin) to the patient without consulting first with the health care provider. D) The nurse teaches the patient that sexual activity usually can be resumed once the surgical incision is healed.

A) The nurse assists the patient to do active range of motion exercises for all extremities. The patient should avoid moving the arm on the ICD insertion site until healing hasoccurred in order to prevent displacement of the ICD leads. The other actions by the newnurse are appropriate for this patient.

When providing nutritional counseling for patients at risk for CAD, which of the following foods would the nurse encourage patients to include in their diet (select all that apply)? A) a. Tofu B) b. Walnuts C) c. Tuna fish D) d. Whole milk E) e. Orange juice

A,B,C Tuna fish, tofu, and walnuts are all rich in omega-3 fatty acids, which have been shown to reduce the risks associated with CAD when consumed regularly

A nurse is caring for a patient immediately following a transesophageal echocardiogram (TEE). Which of the following assessments are appropriate for this patient (select all that apply)? a. Assess for return of gag reflex. b. Assess groin for hematoma or bleeding. c. Monitor vital signs and oxygen saturation. d. Position patient supine with head of bed flat. e. Assess lower extremities for circulatory compromise.

A,C

A patient is admitted to the CCU with a diagnosis of unstable angina. Which of the following medications would the nurse expect the patient to receive? (select all that apply) a. antiplatelet therapy b. fibrinolytic therapy c. B-adrenergic blockers d. prophylactic antibiotics e. intravenous nitroglycerin

A,C,E

Your plan for care of a patient with AKI includes which goal of dietary management? A. Provide sufficient calories while preventing nitrogen excess. B. Deliver adequate calories while restricting fat and protein intake. C. Replace protein intake with enough fat intake to sustain metabolism. D. Restrict fluids, increase potassium intake, and regulate sodium intake.

A. Provide sufficient calories while preventing nitrogen excess. The challenge of nutrition management in AKI is to provide adequate calories to prevent catabolism despite the restrictions required to prevent electrolyte and fluid disorders and azotemia (accumulation of nitrogen and wastes in blood).

What information should the nurse include in discharge education for this patient? (select all that apply) A. Salt restriction B. Weight reduction C. Smoking Cessation D. Avoid truck driving

A. Salt restriction B. Weight reduction C. Smoking Cessation

Which mechanism for blood pressure regulation has the fastest action? A. Sympathetic stimulation B. Aldosterone C. Epinephrine D. Antidiuretic Hormone

A. Sympathetic stimulation Fight or flight, once activated BP and HR will rise

Which of the following patients is most likely to experience anemia with an etiology of increased destruction of red blood cells? A. An African American man who has a diagnosis of sickle cell disease B. A 59-year-old man whose alcoholism has precipitated folic acid deficiency C. A 30-year-old woman with a history of "heavy periods" accompanied by anemia D. A 3-year-old child whose impaired growth and development is attributable to thalassemia

A. The etiology of sickle cell anemia involves increased hemolysis. Thalassemias and folic acid deficiencies cause a decrease in erythropoiesis whereas the anemia surrounding menstruation is a direct result of blood loss.

The nurse receives a physician's order to transfuse fresh frozen plasma to a patient suffering from an acute blood loss. Which of the following procedures is most appropriate for infusing this blood product? A. Infuse the fresh frozen plasma as rapidly as the patient will tolerate. B. Hang the fresh frozen plasma as a piggyback to the primary IV solution. C. Infuse the fresh frozen plasma as a piggyback to a primary solution of normal saline. D. Hang the fresh frozen plasma as a piggyback to a new bag of primary IV solution without KCl.

A. The fresh frozen plasma should be administered as rapidly as possible and should be used within 2 hours of thawing. Fresh frozen plasma is infused using any straight-line infusion set. Any existing IV should be interrupted while the fresh frozen plasma is infused, unless a second IV line has been started for the transfusion.

Which patient has the greatest risk for prerenal AKI? A. The patient is hypovolemic because of hemorrhage. B. The patient relates a history of chronic urinary tract obstruction. C. The patient has vascular changes related to coagulopathies. D. The patient is receiving antibiotics such as gentamicin.

A. The patient is hypovolemic because of hemorrhage. Prerenal causes of AKI are factors external to the kidneys. These factors reduce systemic circulation, causing a reduction in renal blood flow, and they lead to decreased glomerular perfusion and filtration of the kidneys.

Important nursing interventions for the patient with AKI are (select all that apply) A. careful monitoring of intake and output. B. daily patient weights. C. meticulous aseptic technique. D. increase intake of vitamin A and D. E. frequent mouth care.

A. careful monitoring of intake and output. B. daily patient weights. C. meticulous aseptic technique. E. frequent mouth care. You have an important role in managing fluid and electrolyte balance during the oliguric and diuretic phases of AKI. Observing and recording accurate intake and output are essential. Measure daily weights with the same scale at the same time each day to assess excessive gains or losses of body fluids. Mouth care is important to prevent stomatitis, which develops when ammonia (produced by bacterial breakdown of urea) in saliva irritates the mucous membrane.

When developing a health teaching plan for a patient, you should focus on his: A. high serum LDL B. age C. gender D. family history of CAD

A. high serum LDL it's the only thing that can change!

Patients with CKD have an increased incidence of cardiovascular disease related to (select all that apply) A. hypertension. B. vascular calcifications. C. a genetic predisposition. D. hyperinsulinemia causing dyslipidemia. E. increased high-density lipoproteins levels.

A. hypertension. B. vascular calcifications. D. hyperinsulinemia causing dyslipidemia. Traditional cardiovascular risk factors, such as hypertension and elevated lipid levels, are common in CKD patients. Hyperinsulinemia stimulates hepatic production of triglycerides. Most patients with uremia develop dyslipidemia. Much of the cardiovascular disease may be related to nontraditional risk factors such as vascular calcification and arterial stiffness. Vascular calcification and arterial stiffness are major contributors to cardiovascular disease in CKD. Calcium deposits in the vascular medial layer are associated with stiffening of the blood vessels. The mechanisms involved are multifactorial and incompletely understood, but they include (1) vascular smooth muscle cells that change into a chondrocyte or osteoblast-like cell, (2) high total body calcium and phosphate levels due to abnormal bone metabolism, (3) impaired renal excretion, and (4) drug therapies to treat the bone disease (e.g., calcium phosphate binders).

The advantage of continuous replacement therapy over hemodialysis is its ability to A. remove fluid without the use of a dialysate. B. remove fluid in less than 24 hours. C. allow the patient to receive the therapy at the work site. D. be administered through a peripheral line.

A. remove fluid without the use of a dialysate. Several features of continuous replacement therapy are different from those of hemodialysis. Solute removal can occur by convection (no dialysate required) in addition to osmosis and diffusion. The process can take days or weeks. The patient cannot receive the therapy at work and a vascular access device is required.

A patient has sought care after an episode of syncope of unknown origin. Which nursing action should you prioritize in the patient's subsequent diagnostic workup? A. Preparing to assist with a head-up tilt test B. Assessing the patient's knowledge of pacemakers C. Preparing an intravenous dose of a β-adrenergic blocker D. Teaching the patient about the role of antiplatelet aggregators

ANS: A A head-up tilt test is a common component of the diagnostic workup after episodes of syncope. Intravenous β-blockers are not indicated, and addressing pacemakers is premature and inappropriate at this stage of diagnosis. Patient education about antiplatelet aggregators is not directly relevant to the patient's syncope. Reference: 839

What is a priority nursing intervention for a patient during the acute phase of rheumatic fever? A. Administration of antibiotics as ordered B. Management of pain with opioid analgesics C. Encouragement of fluid intake for hydration D. Performance of frequent, active range of motion exercises

ANS: A Antibiotic therapy does not modify the course of the acute disease or the development of carditis. It does eliminate residual group A streptococci remaining in the tonsils and pharynx and prevent the spread of organisms to

When a person's blood pressure rises, the homeostatic mechanism that compensates for the elevation involves stimulation of A. baroreceptors that inhibit the sympathetic nervous system, causing vasodilation. B. chemoreceptors that inhibit the sympathetic nervous system, causing vasodilation. C. baroreceptors that inhibit the parasympathetic nervous system, causing vasodilation. D. chemoreceptors that stimulate the sympathetic nervous system, increasing the heart rate.

ANS: A Baroreceptors in the aortic arch and carotid sinus are sensitive to stretch or pressure within the arterial system. Stimulation of these receptors sends information to the vasomotor center in the brainstem. This results in temporary inhibition of the sympathetic nervous system and enhancement of the parasympathetic influence, decreasing the heart rate and peripheral vasodilation. Reference: 719

You are providing care for a patient who has decreased cardiac output related to heart failure. You recognize that cardiac output is A. calculated by multiplying the patient's stroke volume by the heart rate. B. the average amount of blood ejected during one complete cardiac cycle. C. determined by measuring the electrical activity of the heart and the patient's heart rate. D. the patient's average resting heart rate multiplied by the patient's mean arterial blood pressure.

ANS: A Cardiac output is determined by multiplying the patient's stroke volume by heart rate, identifying how much blood is pumped by the heart over a 1-minute period. Electrical activity of the heart and blood pressure are not direct components of cardiac output. Reference: 718

A70-year-old woman with chronic heart failure and atrial fibrillation asks the nurse why warfarin (Coumadin) has been prescribed for her to continue at home. Which response by the nurse is accurate? A. "The medication prevents blood clots from forming in your heart." B. "The medication dissolves clots that develop in your coronary arteries." C. "The medication reduces clotting by decreasing serum potassium levels." D. "The medication increases your heart rate so that clots do not form in your heart."

ANS: A Chronic heart failure causes enlargement of the chambers of the heart and an altered electrical pathway, especially in the atria. When numerous sites in the atria fire spontaneously and rapidly, atrial fibrillation occurs. Atrial fibrillation promotes thrombus formation within the atria with an increased risk of stroke and requires treatment with cardioversion, antidysrhythmics, and/or anticoagulants. Warfarin is an anticoagulant that interferes with hepatic synthesis of vitamin K-dependent clotting factors.

On physical assessment of the patient with hypertrophic cardiomyopathy, you expect to find A. displaced point of maximal impulse (PMI). B. ascites. C. friction rub. D. subcutaneous emphysema.

ANS: A Clinical findings of hypertrophic cardiomyopathy on examination may be unremarkable. However, on palpation of the chest, the apical impulse can be exaggerated and displaced laterally. Auscultation may reveal an S4 and a systolic murmur between the apex and the sternal border at the fourth intercostal space. The patient does not have ascites, which is associated with liver abnormalities, and the patient does not have a friction rum or subcutaneous emphysema. Reference: 862

What is a classic sign of unstable angina and coronary artery disease (CAD) in women? A. Fatigue B. Hypotension C. Atrial fibrillation D. Headache

ANS: A Common symptoms that women experience are fatigue (most prominent symptom), shortness of breath, indigestion, and anxiety. Reference: 778

Which is accurate related to factors affecting cardiac output? A. Contractility increases with epinephrine administration. B. Preload refers to the resistance from vasoconstriction. C. Afterload is the amount of blood in the ventricles at diastole. D. Cardiac output is mainly affected by absolute refractory period.

ANS: A Contractility is increased by epinephrine or norepinephrine, which are normally released by the sympathetic nervous system, but each can be administered as a drug. Preload is the volume of blood in the ventricles at the end of diastole, before the next contraction. Preload determines the amount of stretch placed on myocardial fibers. Afterload is the peripheral resistance against which the left ventricle must pump. It is affected by the size of the ventricle, wall tension, and arterial blood pressure. The absolute refractory period is a time in which cardiac muscles do not respond to any stimuli to recover excitability. Reference: 718

The charge nurse is explaining the concept of pacemaker failure to capture to the new graduate. What information should the charge nurse give? A. It occurs when the electrical charge is insufficient. B. It occurs when the pacemaker does not recognize spontaneous heart activity. C. A complication is ventricular tachycardia. D. First-line treatment when this occurs is to turn down the electrical charge.

ANS: A Failure to capture occurs when the electrical charge to the myocardium is insufficient to produce atrial or ventricular contraction. It can result in serious bradycardia, and treatment includes increasing the electrical charge. Failure to sense occurs when the pacemaker fails to recognize spontaneous atrial or ventricular activity and fires inappropriately. This can result in ventricular tachycardia. Reference: 836

A fibrinolytic agent is administered in the emergency department to the patient diagnosed with an acute myocardial infarction (MI). Which is the best indicator that the drug has achieved its therapeutic effect? A. Return of ST segment to baseline on the ECG B. Early, rapid decline in the CK-MB enzyme levels C. Increase in the prothrombin time, international normalized ratio, and partial thromboplastin time laboratory values D. Vital signs within normal range

ANS: A Fibrinolytics are given to produce an open artery by lysis of the thrombus in the coronary artery. The most reliable marker that this has occurred is the return of the ST segment to baseline on the ECG. Other markers include a resolution of chest pain and an early, rapid rise of the CK-MB enzyme levels within 3 hours of therapy, because the necrotic myocardial cells release CK-MB enzymes into the circulation after perfusion is restored to the area. Reperfusion dysrhythmias are a less reliable maker. The drug is not given specifically to accomplish prolonged clotting time or to alter vital signs. Reference: 781-782

A patient with chronic heart failure and atrial fibrillation is treated with a digitalis glycoside and a loop diuretic. To prevent possible complications of this combination of drugs, you need to A. monitor serum potassium levels. B. keep an accurate measure of intake and output. C. teach the patient about dietary restriction of potassium. D. withhold the digitalis and notify the health care provider if the heart rate is irregular.

ANS: A Hypokalemia due to use of potassium-depleting diuretics (e.g., thiazides, loop diuretics) is one of the most common causes of digitalis toxicity. Low serum levels of potassium enhance the action of digitalis, causing a therapeutic dose to achieve toxic levels. Hypokalemia can also precipitate the development of dysrhythmias. Monitoring the serum potassium levels of patients receiving digitalis preparations and potassium-depleting diuretics is essential. Reference: 807

The patient has a heart rate of 40 beats/minute. The P waves are regular, and the Q waves are regular, but there is no relationship between the P wave and QRS complex. What treatment do you anticipate? A. Pacemaker B. Continue to monitor C. Carotid massage D. Defibrillation

ANS: A In third-degree atrioventricular (AV) block, there is no correlation between the impulse from the atrium to the ventricles and the ventricular rhythm seen. A pacemaker eventually is required. Action must be taken because this usually results in reduced cardiac output with subsequent ischemia if untreated. Carotid massage is vagal stimulation, and it can cause bradycardia. There is a problem in conduction, not abnormal contraction, and defibrillation is not used. Reference: 830

At a clinic visit, the nurse provides dietary teaching for a 56-year-old woman who was recently hospitalized with an exacerbation of chronic heart failure. The nurse determines that teaching is successful if the patient makes which statement? A. "I will limit the amount of milk and cheese in my diet." B. "I can add salt when cooking foods but not at the table." C. "I will take an extra diuretic pill when I eat a lot of salt." D. "I can have unlimited amounts of foods labeled as reduced sodium ."

ANS: A Milk products should be limited to 2 cups per day for a 2500-mg sodium-restricted diet. Salt should not be added during food preparation or at the table. Diuretics should be taken as prescribed (usually daily) and not based on sodium intake. Foods labeled as reduced sodium contain at least 25% less sodium than regular.

Assessment of the patient with chronic aortic regurgitation reveals a A. water-hammer pulse. B. diminished pulses in the legs. C. third heart sound (S3). D. widening pulse pressure.

ANS: A Patients with chronic, severe aortic regurgitation develop a water-hammer pulse (strong, quick beat that collapses immediately). Heart sounds may include a soft or absent S1, presence of S3 or S4, and a soft, high-pitched diastolic murmur. A low-pitched diastolic murmur, known as an Austin-Flint murmur, may be heard in severe aortic regurgitation. Reference: 855

While admitting a patient with pericarditis, you assess for which manifestation of this disorder? A. Pulsus paradoxus B. Prolonged PR intervals C. Widened pulse pressure D. Clubbing of the fingers

ANS: A Pericarditis can lead to cardiac tamponade, an emergency situation. Pulsus paradoxus greater than 10 mm Hg is a sign of cardiac tamponade that should be assessed at least every 4 hours in a patient with pericarditis. Reference: 847

When teaching a patient about the long-term consequences of rheumatic fever, you should discuss the possibility of A. valvular heart disease. B. pulmonary hypertension. C. superior vena cava syndrome. D. hypertrophy of the right ventricle.

ANS: A Rheumatic heart disease is a chronic condition resulting from rheumatic fever that is characterized by scarring and deformity of the heart valves. Reference: 850

After teaching about ways to decrease risk factors for CAD, you recognize that additional instruction is needed when the patient says the following: A. "I would like to add weight lifting to my exercise program." B. "I can keep my blood pressure normal only with medication." C. "I can change my diet to decrease my intake of saturated fats." D. "I will change my lifestyle to reduce activities that increase my stress."

ANS: A Risk factors for CAD include elevated serum lipids, elevated blood pressure, tobacco use, physical inactivity, obesity, diabetes, metabolic syndrome, psychologic states, and elevated homocysteine levels. Weight lifting is not a cardiac-protective exercise. An example of health-promoting regular physical activity is brisk walking (3 to 4 miles/hr) for at least 30 minutes five or more times per week. Reference: 761-763

Which individual would you identify as having the highest risk for CAD? A. A 45-year-old, depressed man with a high-stress job B. A 60-year-old man with low homocysteine levels C. A 54-year-old female vegetarian with elevated high-density lipoprotein (HDL) cholesterol levels D. A 62-year-old woman who has a sedentary lifestyle and BMI of 23 kg/m2

ANS: A Studies demonstrate that depression and stressful states can contribute to the development of CAD. Elevated HDL cholesterol levels and low homocysteine levels help to prevent CAD. Although a sedentary lifestyle is a risk factor, a BMI of 23 kg/m2 depicts normal weight, and the patient with two risk factors is at greatest risk for developing CAD. Reference: 766

You recognize that primary manifestations of systolic failure include A. ↓ ejection fraction (EF) and ↑ pulmonary artery wedge pressure (PAWP). B. ↓ PAWP and ↑ EF. C. ↓ pulmonary hypertension associated with normal EF. D. ↓ afterload and ↓ left ventricular end-diastolic pressure.

ANS: A Systolic heart failure results in left ventricle (LV) systolic dysfunction. The LV loses its ability to generate enough pressure to eject blood forward through the aorta. This results in increased PAWP. The hallmark of systolic dysfunction is decreased left ventricular EF. Reference: 798

When planning emergent care for a patient with a suspected MI, you anticipate administration of A. oxygen, nitroglycerin, aspirin, and morphine. B. oxygen, furosemide (Lasix), nitroglycerin, and meperidine. C. aspirin, nitroprusside (Nipride), dopamine (Intropin), and oxygen. D. nitroglycerin, lorazepam (Ativan), oxygen, and warfarin (Coumadin).

ANS: A The American Heart Association's guidelines for emergency care of the patient with chest pain include the administration of oxygen, nitroglycerin, aspirin, and morphine. These interventions relieve chest pain, improve oxygenation, decrease myocardial workload, and prevent further platelet aggregation. Reference: 781, 782

The patient asks you what an ejection fraction (EF) is. Which statement is the appropriate explanation? A. It provides information about left ventricular function during heart contraction. B. It helps to determine electrical impulse conduction through the heart. C. It allows visualization of the heart anatomy and coronary circulation. D. Provides information on cardiac wall movement and valves.

ANS: A The EF is the percentage of end-diastolic blood volume that is ejected during systole. It provides information about the function of the left ventricle during systole. Reference: 733

The nurse is teaching a community group about preventing rheumatic fever. What information should the nurse include? A. Prompt recognition and treatment of streptococcal pharyngitis B. Completion of 4 to 6 days of antibiotic therapy for infective endocarditis of respiratory infections in children born with heart defects C. Avoidance of respiratory infections in children who have rheumatoid arthritis D. Requesting antibiotics before dental surgery for individuals with rheumatoid arthritis.

ANS: A The nurse should emphasize the need for prompt and adequate treatment of streptococcal pharyngitis infection, which can lead to the complication of rheumatic fever.

A patient with newly diagnosed hypertension has a blood pressure of 158/98 mm Hg after 6 months of exercise and diet modifications. Which management strategy is a priority for this patient? A. Medication is required because the blood pressure is still not at the goal. B. Continued blood pressure monitoring for another 3 months is all that is necessary at this time. C. Lifestyle modifications are no longer important because they were not effective, and medications will be started. D. More vigorous changes in the patient's lifestyle are needed for a longer period before starting medications.

ANS: A The patient has stage 1 hypertension. Lifestyle modifications will continue, but drug therapy initiation is a priority. Reduction of blood pressure will aid in the prevention of serious complications related to hypertension. Reference: 745, 747

Which factor in the patient's history alerts you to the possibility of infective endocarditis? A. Dental work 2 months earlier B. Rheumatoid arthritis for 2 years C. Vitamin B12 deficiency D. Fractured humerus 3 weeks earlier

ANS: A The patient's recent health history is important in assessing infective endocarditis. Ask patients if they have had dental, urologic, surgical, or gynecologic procedures, including normal or abnormal obstetric delivery, within the past 3 to 6 months. Reference: 843

On an admission assessment, you notice clubbing of the fingers. Based on this finding, you question the patient about which disease process? A. Endocarditis B. Acute renal failure C. Myocardial infarction D. Chronic thrombophlebitis

ANS: A Clubbing of the fingers is a loss of the normal angle between the base of the nail and the skin. This finding can be found in cases of endocarditis, congenital defects, and prolonged oxygen deficiency. Reference: 843

Which information about a patient who has been receiving fibrinolytic therapy for an acute myocardial infarction (AMI) is most important for the nurse to communicate to the health care provider? a. No change in the patient's chest pain b. A large bruise at the patient's IV insertion site c. A decrease in ST segment elevation on the electrocardiogram (ECG) d. An increase in cardiac enzyme levels since admission

ANS: A Continued chest pain suggests that the fibrinolytic therapy is not effective and that other interventions such as percutaneous coronary intervention (PCI) may be needed. Bruising is a possible side effect of fibrinolytic therapy, but it is not an indication that therapy should be discontinued. The decrease of the ST segment elevation indicates that fibrinolysis is occurring and perfusion is returning to the injured myocardium. An increase in cardiac enzyme levels is expected with reperfusion and is related to the washout of enzymes into the circulation as the blocked vessel is opened. DIF: Cognitive Level: Application REF: 782-783

When the nurse is taking a history for a patient who is a possible candidate for a kidney transplant, which information about the patient indicates that the patient is not an appropriate candidate for transplantation? a. The patient has metastatic lung cancer. b. The patient has poorly controlled type 1 diabetes. c. The patient has a history of chronic hepatitis C infection. d. The patient is infected with the human immunodeficiency virus.

ANS: A Disseminated malignancies are a contraindication to transplantation. The conditions of the other patients are not contraindications for kidney transplant.

The nurse hears a murmur between the S1 and S2 heart sounds at the patient's left 5th intercostal space and midclavicular line. How will the nurse record this information? a. "Systolic murmur heard at mitral area." b. "Diastolic murmur heard at aortic area." c. "Systolic murmur heard at Erb's point." d. "Diastolic murmur heard at tricuspid area."

ANS: A The S1 signifies the onset of ventricular systole. S2 signifies the onset of diastole. A murmur occurring between these two sounds is a systolic murmur. The mitral area is the intersection of the left 5th intercostal space and the midclavicular line. The other responses describe murmurs heard at different landmarks on the chest and/or during the diastolic phase of the cardiac cycle. DIF: Cognitive Level: Application REF: 725 | 726

A patient who is being admitted to the emergency department with severe chest pain gives the following list of medications taken at home to the nurse. Which of the medications has the most immediate implications for the patient's care? a. sildenafil (Viagra) b. furosemide (Lasix) c. diazepam (Valium) d. captopril (Capoten)

ANS: A The nurse will need to avoid giving nitrates to the patient because nitrate administration is contraindicated in patients who are using sildenafil because of the risk of sudden death caused by vasodilation. The other home medications also should be documented and reported to the health care provider but do not have as immediate an impact on decisions about the patient's treatment. DIF: Cognitive Level: Application REF: 773 | 775-776

Four days after having a myocardial infarction (MI), a patient who is scheduled for discharge asks for assistance with all the daily activities, saying, "I am too nervous to take care of myself." Based on this information, which nursing diagnosis is appropriate? a. Ineffective coping related to anxiety b. Activity intolerance related to weakness c. Denial related to lack of acceptance of the MI d. Social isolation related to lack of support system

ANS: A The patient data indicates that ineffective coping after the MI caused by anxiety about the impact of the MI is a concern. The other nursing diagnoses may be appropriate for some patients after an MI, but the data for this patient do not support denial, activity intolerance, or social isolation. DIF: Cognitive Level: Application REF: 788-789

In a patient with acute kidney injury (AKI) who requires hemodialysis, a temporary vascular access is obtained by placing a catheter in the left femoral vein. Which intervention will be included in the plan of care? a. Place the patient on bed rest. b. Start continuous pulse oximetry. c. Discontinue the retention catheter. d. Restrict the patient's oral protein intake.

ANS: A The patient with a femoral vein catheter must be on bed rest to prevent trauma to the vein. Protein intake is likely to be increased when the patient is receiving dialysis. The retention catheter is likely to remain in place because accurate measurement of output will be needed. There is no indication that the patient needs continuous pulse oximetry.

When caring for a patient with a left arm arteriovenous fistula, which action will the nurse include in the plan of care to maintain the patency of the fistula? a. Check the fistula site for a bruit and thrill. b. Assess the rate and quality of the left radial pulse. c. Compare blood pressures in the left and right arms. d. Irrigate the fistula site with saline every 8 to 12 hours.

ANS: A The presence of a thrill and bruit indicates adequate blood flow through the fistula. Pulse rate and quality are not good indicators of fistula patency. Blood pressures should never be obtained on the arm with a fistula. Irrigation of the fistula might damage the fistula, and typically only dialysis staff would access the fistula.

A patient who has chest pain is admitted to the emergency department (ED), and all the following diagnostic tests are ordered. Which one will the nurse arrange to be completed first? a. Electrocardiogram (ECG) b. Computed tomography (CT) scan c. Chest x-ray d. Troponin level

ANS: A The priority for the patient is to determine whether an acute myocardial infarction (AMI) is occurring so that reperfusion therapy can begin as quickly as possible. ECG changes occur very rapidly after coronary artery occlusion. Troponin levels will increase after about 3 hours. Data from the CT scan and chest x-ray may impact the patient's care but are not helpful in determining whether the patient is experiencing a myocardial infarction (MI). DIF: Cognitive Level: Application REF: 782

When providing nutritional counseling for patients at risk for CAD, which foods do you encourage patients to include in their diet (select all that apply)? A. Tofu B. Walnuts C. Tuna fish D. Whole milk E. Orange juice

ANS: A, B, C Tuna fish, tofu, and walnuts are all rich in omega-3 fatty acids, which have been shown to reduce the risks associated with CAD when consumed regularly. Reference: 768

During the nursing assessment of a patient with an aortic stenosis, you expect to find (select all that apply) A. syncope. B. angina. C. dyspnea on exertion. D. brisk, hammering pulses.

ANS: A,B,C Symptoms of aortic stenosis develop when the valve orifice becomes about one third of its normal size. Symptoms include the classic triad of angina, syncope, and exertional dyspnea, reflecting left ventricular failure. Reference: 855

Assessment of an intravenous cocaine user with infective endocarditis should focus on which signs and symptoms (select all that apply)? A. Retinal hemorrhages B. Splinter hemorrhages C. Presence of Osler's nodes D. Painless nodules over bony prominences E. Painless erythematous macules on the palms and soles

ANS: A,B,C,E Clinical manifestations of infective endocarditis may include hemorrhagic retinal lesions (Roth's spots), splinter hemorrhages (black longitudinal streaks) that may occur in the nail beds, Osler's nodes (painful, tender, red or purple, pea-size lesions) on the fingertips or toes, and Janeway's lesions (flat, painless, small, red spots) on the palms and soles. Reference: 843

Which blood pressure-regulating mechanisms, if defective, can result in hypertension (select all that apply)? A. Release of norepinephrine (NE) B. Secretion of prostaglandins C. Stimulation of the sympathetic nervous system D. Stimulation of the parasympathetic nervous system E. Activation of the renin-angiotensin-aldosterone system

ANS: A,B,C,E NE is released from the sympathetic nervous system nerve endings and activates receptors located in the vascular smooth muscle. When the α-adrenergic receptors in smooth muscle of the blood vessels are stimulated by NE, vasoconstriction results. Increased sympathetic nervous system stimulation produces increased vasoconstriction and increased renin release. Increased renin levels activate the renin-angiotensin-aldosterone system, leading to elevated blood pressure. Prostaglandins secreted by the renal medulla have a vasodilator effect on the systemic circulation. This results in decreased systemic vascular resistance and lower blood pressure. If prostaglandin synthesis is altered, the blood pressure can be increased. Reference: 739-741

A patient is admitted with myocarditis. While performing your initial assessment, which clinical signs and symptoms might you find (select all that apply)? A. Angina B. Pleuritic chest pain C. Splinter hemorrhages D. Pericardial friction rub E. Presence of Osler's nodes

ANS: A,B,D Clinical manifestations of myocarditis may include early systemic manifestations (fever, fatigue, malaise, myalgias, pharyngitis, dyspnea, lymphadenopathy, nausea, and vomiting), early cardiac manifestations (pleuritic chest pain with a pericardial friction rub and effusion), and late cardiac signs (S3 heart sound, crackles, jugular venous distention, syncope, peripheral edema, and angina). Reference: 849

Which manifestations do you expect to find in the patient with chronic mitral regurgitation (select all that apply)? A. Presence of an audible third heard sound S3 B. Fatigue C. Dyspnea D. Ventricular dysrhythmias E. Pitting edema

ANS: A,B,D,E Patients with chronic mitral regurgitation may remain asymptomatic for many years. Initial symptoms of left ventricular failure may include weakness, fatigue, palpitations, and dyspnea that gradually progress to orthopnea, paroxysmal nocturnal dyspnea, and peripheral edema. Increased left ventricular filling leads to an audible third heart sound (S3), even with normal left ventricular function. Reference: 854

Which are considered risk factors for coronary artery disease (CAD) (select all that apply)? A. Cholesterol level of 240 mg/dL B. Elevated serum level of high-density lipoproteins (HDLs) C. Blood pressure of 150/92 mm Hg D. Smokes 2 packs/day E. Body mass index (BMI) of 25 kg/m2

ANS: A,C,D A cholesterol level higher than 200 mg/dL is one of the four most firmly established risk factors. Hypertension (blood pressure greater than 140/90 mm Hg or greater than 130/80 mm Hg if the patient has diabetes or chronic kidney disease) is another major risk factor. Tobacco use (proportional to the number of cigarettes smoked) is major risk factor. High HDL levels are desirable because they transport lipid that accumulates within the arterial walls. The higher the HDL levels in the blood, the lower the risk of CAD. A BMI greater than 30 kg/m2 is a risk factor. Reference: 761-763

What are considered significant findings related to cardiac disease (select all that apply)? A. Paroxysmal nocturnal dyspnea B. Body mass index (BMI) of 22 kg/m2 C. History of streptococcal throat infections D. Nocturia E. Otitis media

ANS: A,C,D Attacks of shortness of breath, especially at night, that awaken the patient are associated with heart failure. History of improperly treated streptococcal sore throat can cause heart valve damage. Nocturia is a common finding with cardiovascular patients. A BMI of 22 kg/m2 is normal. There is no relationship between otitis media and cardiac disease. Reference: 720-721

A patient admitted with heart failure appears very anxious and complains of shortness of breath. Which nursing actions are appropriate to alleviate this patient's anxiety (select all that apply)? A. Administer the ordered morphine sulfate. B. Position the patient on the left side with the head of the bed flat. C. Instruct the patient on the use of relaxation techniques. D. Use a calm, reassuring approach while talking to the patient.

ANS: A,C,D Morphine sulfate reduces anxiety and may assist in reducing dyspnea. Relaxation techniques and a calm, reassuring approach also can reduce anxiety. The patient should be positioned in a semi-Fowler position to improve ventilation. Reference: 805

Which characteristics describe restrictive cardiomyopathy (select all that apply)? A. Systolic function is normal. B. Cardiac output is increased. C. Diastolic function is impaired. D. Ventricular hypertrophy is present. E. Lungs remain clear and function normally.

ANS: A,C,D Systolic function is normal, diastolic function is impaired, and ventricular hypertrophy is present. Cardiac output is normal or decreased. Evaluation of the lungs finds pleural effusion and congestion. Reference: 861-862

Your patient is diagnosed with mitral stenosis and new-onset atrial fibrillation. Which interventions could you delegate to nursing assistive personnel (NAP) (select all that apply)? A. Obtain and record daily weight. B. Determine the apical-radial pulse rate. C. Observe for overt signs of bleeding. D. Obtain and record vital signs, including pulse oximetry values. E. Teach the patient how to purchase a Medic Alert bracelet.

ANS: A,C,D You may delegate routine procedures such as obtaining weights and vital signs. You also may give specific directions to the NAP to observe and report obvious signs of bleeding. You cannot delegate teaching, assessment, or activities that require clinical judgment. Obtaining an apical-radial pulse rate is an assessment. Reference: 856

A patient is admitted to the coronary care unit with a diagnosis of unstable angina. Which medications do you expect the patient to receive (select all that apply)? A. Antiplatelet therapy B. Fibrinolytic therapy C. β-Adrenergic blockers D. Prophylactic antibiotics E. Intravenous nitroglycerin

ANS: A,C,E Oxygen, nitroglycerin, aspirin (chewable), and morphine may be used to treat unstable angina. For patients with unstable angina with negative cardiac markers and ongoing angina, a combination of aspirin, heparin, and a glycoprotein IIb/IIIa inhibitor (e.g., eptifibatide [Integrilin]) is recommended. β-Adrenergic blockers decrease myocardial oxygen demand by reducing heart rate, blood pressure, and contractility. Reference: 781

The patient with ACS had balloon angioplasty and a drug-eluding stent placed. What is required nursing care in the first 6 hours after the procedure (select all that apply)? A. Assess the pedal pulse in the affected leg. B. Ambulate the patient to prevent thrombus. C. Provide oxygen by mask. D. Administer a glycoprotein IIb/IIIa inhibitor. E. Regularly obtain vital signs.

ANS: A,D,E Neurovascular assessments are performed in the affected leg, a IIb/IIIa inhibitor is given to prevent abrupt closure of the stented vessel, and regularly obtained vital signs help monitor cardiac function. The patient is kept on bed rest with an extended leg and a pressure dressing on the insertion site. Oxygen, if needed, usually is administered in lower concentrations by nasal cannula. Reference: 777

The female patient presents to the emergency department just after a syncope episode. What should you assess first? A. History of syncope B. Capillary glucose level C. Last menstrual period D. Allergies

ANS: B A change in the level of consciousness should always have glucose and oxygen (and cardiac) assessed first. Hypoglycemia is a noncardiovascular cause that can be easily treated. It takes priority over the other assessments. Although those who have syncope have a 30% chance of recurrence, ruling out a simple treatable cause should be done first. Reference: 839

.You conduct a complete physical assessment on a patient admitted with infective endocarditis. Which findings are significant? A. Respiratory rate of 18 breaths/minute and heart rate of 90 beats/minute B. Regurgitant murmur at the mitral valve area C. Heart rate of 94 beats/minute and capillary refill time of 2 seconds D. Point of maximal impulse palpable in the fourth intercostal space

ANS: B A regurgitant murmur indicates valvular disease, which can be a complication of endocarditis. All the other findings are within normal limits. Reference: 853

You are watching the cardiac monitor, and a patient's rhythm suddenly changes. There are no P waves. Instead, there are fine, wavy lines between the QRS complexes. The QRS complexes each measure 0.08 second (narrow), but they occur irregularly with a rate of 120 beats/minute. You correctly interpret that this rhythm is A. sinus tachycardia. B. atrial fibrillation. C. ventricular fibrillation. D. ventricular tachycardia.

ANS: B Atrial fibrillation is represented on the cardiac monitor by irregular R-R intervals and small fibrillatory (F) waves. There are no normal P waves because the atria are not contracting, just fibrillating. Reference: 827

Which patient assessment is most likely to have a negative effect on cardiac output? A. The heart rate is 104 beats/minute. B. The ECG indicates left ventricle ischemia. C. The T wave is peaked. D. A U wave is present.

ANS: B Cardiac output is the amount of blood ejected from the ventricle with each heartbeat. This is a result of the stroke volume times the heart rate (beats per minute). An ischemic left ventricle does not pump effectively. A heart rate of 100 beats/minute is within the normal range. A peaked T wave indicates hyperkalemia, but cardiac output should remain within normal limits if the heart rate and stroke volume are within normal limits. A U wave, if seen, may represent repolarization of the Purkinje fibers, or it may be associated with hypokalemia. Reference: 718

A hospitalized patient with a history of chronic stable angina tells you that she is having chest pain. You base your actions on the knowledge that ischemia A. will always progress to myocardial infarction. B. will be relieved by rest or nitroglycerin, or both. C. indicates that irreversible myocardial damage is occurring. D. is frequently associated with vomiting and extreme fatigue.

ANS: B Chronic stable angina refers to chest pain that occurs intermittently over a long period with the same pattern of onset, duration, and intensity of symptoms. The chest pain is relieved by rest or by rest and medication (e.g., nitroglycerin). Ischemia does not cause myocardial damage. Reference: 772, 773

The body attempts to compensate for the heart's failure by A. decreasing afterload and decreasing heart rate. B. increasing heart rate and dilating ventricular chambers. C. increasing blood pressure and decreasing cardiac preload. D. inhibiting epinephrine and maintaining minimal venous return.

ANS: B Heart failure can have an abrupt onset, as with acute myocardial infarction and rapid atrial fibrillation, or it can be an insidious process resulting from slow, progressive changes. The overloaded heart resorts to compensatory mechanisms to try to maintain adequate cardiac output. The main compensatory mechanisms include sympathetic nervous system activation, neurohormonal responses, ventricular dilation, and ventricular hypertrophy. Reference: 799

You are caring for a patient admitted with a history of hypertension. The patient's medication history includes hydrochlorothiazide (HydroDIURIL) daily for the past 10 years. Which parameter indicates the optimal intended effect of this drug therapy? A. Weight loss of 2 lb B. Blood pressure 128/86 mm Hg C. Absence of ankle edema D. Urine output of 600 mL per shift

ANS: B Hydrochlorothiazide may be used alone as monotherapy to manage hypertension or in combination with other medications if not effective alone. After the first few weeks of therapy, the diuretic effect diminishes, but the antihypertensive effect remains. Because the patient has been taking this medication for 10 years, the most direct measurement of its intended effect is the blood pressure. Reference: 748

Which patient finding is most likely a result of long-term uncontrolled hypertension? A. Bilateral cataracts B. Stroke C. Blood urea nitrogen (BUN) level of 20 mg/dL and creatinine level of 0.9 mg/dL D. Platelet level of 140,000/μL

ANS: B Hypertension is a major risk factor for cerebral atherosclerosis and stroke. Even for mildly hypertensive people, the risk of stroke is four times higher than for normotensive people. Hypertension can cause retinal damage, but cataracts are cloudy of the lens due to aging. Hypertension is a leading cause for end-stage renal disease, but these laboratory values are normal range and ratio. Hypertension is not directly linked to thrombocytopenia. Reference: 738

A patient in the emergency department with chest pain that is unrelieved by nitroglycerin is diagnosed with an acute MI. Why is IV morphine now prescribed? A. To relieve the chest pain B. To decrease cardiac workload C. To depress respirations D. To prevent ventricular remodeling

ANS: B Morphine is given as a vasodilator to decrease the cardiac workload by lowering myocardial oxygen consumption, reducing contractility, and decreasing blood pressure and heart rate. It also helps to reduce anxiety and fear. The pain from an MI is from ischemia, and efforts are directed at improving perfusion and oxygenation of tissue rather than at masking the pain. Respiration depression is a side effect that you monitor for. Oxygenation is of prime importance. Ventricular remodeling is part of the healing process later on and a reason for an angiotensin-converting enzyme (ACE) inhibitor to be prescribed. Reference: 775

Which manifestation would you expect to find in the patient with a mitral valve prolapse (MVP)? A. An audible third heard sound S3 B. Systolic murmur C. Dyspnea D. Ventricular dysrhythmias

ANS: B Most patients are asymptomatic and remain so for their entire lives. About 10% of those with MVP become symptomatic. A characteristic of MVP is a murmur from regurgitation that gets more intense through systole. Reference: 854

When caring for a patient with myocarditis, you identify the priority nursing diagnosis of A. deficient knowledge. B. decreased cardiac output. C. pain. D. activity intolerance.

ANS: B Myocarditis results in cardiac dysfunction and has been linked to dilated cardiomyopathy. Decreased cardiac output is an ongoing nursing diagnosis. Although all the nursing diagnoses identified may be appropriate for a patient with myocarditis, according to the airway, breathing, and circulation (ABCs) guidelines for prioritization, cardiovascular compromise is the priority. Reference: 850

The emergency department patient is in paroxysmal supraventricular tachycardia (PSVT) at a rate of 170 beats/minute. Which treatment do you anticipate first? A. Sotalol (Betapace) by slow IVP B. Adenosine (Adenocard) by fast IVP C. Defibrillation D. Digoxin (Lanoxin)

ANS: B PSVT is a dysrhythmia originating in an ectopic focus anywhere above the bifurcation of the bundle of His. Treatment includes vagal stimulation (e.g., Valsalva maneuver, coughing) and intravenous (IV) adenosine as the first drug of choice. The drug has a short half-life and is given rapid IVP. Other drugs are β-adrenergic blockers, calcium channel blockers, and amiodarone. Defibrillation is used if the vagal stimulation and drug therapy are ineffective and the patient becomes hemodynamically unstable. Digoxin is not used for this dysrhythmia but typically is used for atrial fibrillation. Reference: 826

The home care nurse is visiting a patient with chronic heart failure who is taking 0.25 mg of digoxin (Lanoxin) orally each day with 60 mg of oral furosemide (Lasix) daily. At this visit, the patient is complaining of nausea and vomiting. Your priority is to A. instruct the patient to increase intake of high-potassium foods. B. notify the health care provider. C. perform a dipstick urine test for protein. D. weigh the patient.

ANS: B Patients receiving a digitalis preparation are prone to develop digitalis toxicity (Table 35-9). Early symptoms of toxicity include anorexia, nausea, and vomiting. Visual disturbances, such as yellow vision, can also occur. Dysrhythmias are a common but often late indication of digitalis toxicity. Reference: 807

The nurse performs discharge teaching for a 68-year-old man who is newly diagnosed with infective endocarditis with a history of IV substance abuse. Which statement by the patient indicates to the nurse that teaching was successful? A. "I will need antibiotics before having any invasive procedure or surgery." B. "I will inform my dentist about my hospitalization for infective endocarditis." C. "I should not be alarmed if I have difficulty breathing or pink-tinged sputum." D. "An elevated temperature is expected and can be managed by taking acetaminophen." ANS:

ANS: B Patients with infective endocarditis should inform their dental providers of their health history. Antibiotic prophylaxis is recommended for patients with a history of infective endocarditis who have certain dental procedures performed. Antibiotics are not indicated before genitourinary or gastrointestinal procedures unless an infection is present. Patients should immediately report the presence of fever or clinical manifestations indicating heart failure to their health care provider.

The patient asks how Prinzmetal's angina (variant angina) is different from chronic stable angina. What is the best response? A. Stable angina occurs at a higher incidence among people with a history of migraine headaches. B. Prinzmetal's angina is not precipitated by increased activity but is a strong spasm. C. Chronic stable angina has hypertension, and Prinzmetal's angina has hypotension. D. Prinzmetal's angina is not influenced by tobacco use, but chronic stable angina is.

ANS: B Prinzmetal's angina is not usually precipitated by increased physical demand. The strong contraction (spasm) of smooth muscle in the coronary artery results from an increase in intracellular calcium levels. It can occur in the absence of coronary artery disease (CAD). Prinzmetal's angina is more frequently seen in a patient with a history of migraine headaches and Raynaud's phenomenon, but stable angina also can occur in patients with this medical history. Nitrates and calcium channel blockers are used by patients with Prinzmetal's angina. Tobacco smoke increases myocardial oxygen demand and is an influence in both conditions. Reference: 772

What specific diet changes are encouraged to reduce the risks associated with CAD? A. Reduce complex carbohydrates. B. Consume forms of soybeans. C. Increase intake of organ meats. D. Avoid fatty fish such as salmon.

ANS: B The American Heart Association (AHA) recommends eating tofu, other forms of soybean, canola, walnuts, and flaxseed because these products contain α-linolenic acid, which becomes omega-3 fatty acid in the body. Complex carbohydrates (e.g., whole grains, fruit, vegetables) are recommended. Organ meats are not recommended due to high-fat content. Two servings per week of fatty fish (e.g., salmon, tuna) are recommended because they contain two types of omega-3 fatty acids. Reference: 768

During a cardiac catheterization into the right side of the heart, a pulmonary artery wedge pressure is obtained. What is the purpose of this measurement? A. Determine efficiency of the right heart contraction B. Assess function of the left side of the heart C. Identify coronary lesions D. Measure the heart's afterload

ANS: B The catheter is advanced into the vena cava, the right atrium, and the right ventricle. The catheter is further inserted into the pulmonary artery, and pressures are recorded. The catheter is then advanced until it is wedged in position and looks forward through the pulmonary capillary bed to the pressure in the left side of the heart (wedge pressure). The wedge pressure is used to assess the function of the left side of the heart. Reference: 735

When teaching a patient about dietary management of stage 1 hypertension, which instruction is most appropriate? A. Restrict all caffeine. B. Restrict sodium intake. C. Increase protein intake. D. Use of calcium supplements.

ANS: B The patient should decrease intake of sodium to help control hypertension. Excessive salt intake can cause fluid retention. Reference: 746

A patient is hospitalized with severe dyspnea; a dry, hacking cough; and pitting edema in both ankles. The patient has a history of heart failure, and current vital signs are blood pressure of 170/100 mm Hg, pulse of 92 beats/minute, and respiratory rate of 28 breaths/minute. You recognize that the patient's symptoms indicate A. the venous return to the heart is impaired, causing a decrease in afterload. B. impaired emptying of the right and left ventricles with decreased output. C. right-sided failure leading to a decrease in systemic oxygenation. D. decreased oxygenation to the myocardium.

ANS: B The patient with chronic heart failure probably has manifestations of biventricular failure. Reference: 802

You are examining the ECG of a patient who has just been admitted with a suspected MI. Which ECG change is most indicative of prolonged or complete coronary occlusion? A. Sinus tachycardia B. Pathologic Q wave C. Fibrillatory P waves D. Prolonged PR interval

ANS: B The presence of a pathologic Q wave, as often accompanies STEMI, indicates complete coronary occlusion. Sinus tachycardia, fibrillatory P waves (e.g., atrial fibrillation), or a prolonged PR interval (first-degree heart block) are not direct indicators of extensive occlusion. Reference: 780

After teaching a patient with chronic stable angina about nitroglycerin, you recognize the need for further teaching when the patient states the following: A. "I will replace my nitroglycerin supply every 6 months." B. "I can take up to five tablets every 3 minutes for relief of my chest pain." C. "I will take acetaminophen (Tylenol) to treat the headache caused by nitroglycerin." D. "I will take the nitroglycerin 10 minutes before planned activity that usually causes chest pain."

ANS: B The recommended dose of nitroglycerin is one tablet taken sublingually (SL) or one metered spray for symptoms of angina. If symptoms are unchanged or worse after 5 minutes, the patient should be instructed to activate the EMS system. Reference: 775

You obtain a 6-second rhythm strip, and document the following analysis: atrial rate of 70 beats/minute, regular; ventricular rate of 40 beats/minute, regular; QRS of 0.04 second; no relationship between P waves and QRS complexes; and atria and ventricles beating independently of each other. What is the correct interpretation of this rhythm strip? A. Sinus dysrhythmias B. Third-degree heart block C. Wenckebach phenomenon D. Premature ventricular contractions

ANS: B Third-degree heart block represents a loss of communication between the atrium and ventricles. This is depicted on the rhythm strip because there is no relationship between the P waves, representing atrial contraction, and QRS complexes, representing ventricular contraction. The atria are beating totally on their own at 70 beats/minute, whereas the ventricles are pacing themselves at 40 beats/minute. Reference: 830

A 59-year-old man has presented to the emergency department with chest pain. Which component of his subsequent blood work most clearly indicates a myocardial infarction (MI)? A. CK-MB B. Troponin C. Myoglobin D. C-reactive protein (CRP)

ANS: B Troponin is the biomarker of choice in the diagnosis of MI, with sensitivity and specificity that exceed those of CK-MB and myoglobin. CRP levels are not used to diagnose acute MI. Reference: 727

Auscultation of a patient's heart reveals a murmur. This assessment finding is a result of A. increased viscosity of the patient's blood. B. turbulent blood flow across a heart valve. C. friction between the heart the myocardium. D. a deficit in heart conductivity that impairs normal contractility.

ANS: B Turbulent blood flow across the affected valve results in a murmur. A murmur is not a direct result of variances in blood viscosity, conductivity, or friction between the heart and myocardium. Reference: 726

Which is a correct aspect of a cardiac assessment? A. Auscultate the carotid artery to hear a thrill. B. The point of maximal impulse is at the fifth left intercostal space. C. Erb's point is located at the right second intercostals space. D. S1 and S2 cardiac sounds are best heard with the bell of the stethoscope.

ANS: B When the patient is supine, the mitral valve area is the point of maximal impulse (PMI), which is also known as the apical pulse. It reflects the pulsation of the apex of the heart and is located at the left midclavicular line in the fifth intercostal space. A thrill is assessed by touch, a bruit is heard by auscultation, and Erb's point is located at the third left intercostal space, near the sternum. It is where the S2 heart sound is normally heard best. S1 and S2 are best heard with the diaphragm of the stethoscope because they are high-pitched sounds. Reference: 726

Which information collected by the nurse who is admitting a patient with chest pain suggests that the pain is caused by an acute myocardial infarction (AMI)? a. The pain increases with deep breathing. b. The pain has persisted longer than 30 minutes. c. The pain worsens when the patient raises the arms. d. The pain is relieved after the patient takes nitroglycerin.

ANS: B Chest pain that lasts for 20 minutes or more is characteristic of AMI. Changes in pain that occur with raising the arms or with deep breathing are more typical of pericarditis or musculoskeletal pain. Stable angina is usually relieved when the patient takes nitroglycerin. DIF: Cognitive Level: Application REF: 779

The nurse is assessing a patient who is receiving peritoneal dialysis with 2 L inflows. Which information should be reported immediately to the health care provider? a. The patient has an outflow volume of 1800 mL. b. The patient's peritoneal effluent appears cloudy. c. The patient has abdominal pain during the inflow phase. d. The patient complains of feeling bloated after the inflow. .

ANS: B Cloudy appearing peritoneal effluent is a sign of peritonitis and should be reported immediately so that treatment with antibiotics can be started. The other problems can be addressed through nursing interventions such as slowing the inflow and repositioning the patient

The nurse is assessing a patient who is receiving peritoneal dialysis with 2 L inflows. Which information should be reported immediately to the health care provider? a. The patient has an outflow volume of 1800 mL. b. The patient's peritoneal effluent appears cloudy. c. The patient has abdominal pain during the inflow phase. d. The patient complains of feeling bloated after the inflow.

ANS: B Cloudy appearing peritoneal effluent is a sign of peritonitis and should be reported immediately so that treatment with antibiotics can be started. The other problems can be addressed through nursing interventions such as slowing the inflow and repositioning the patient.

For a patient who has been admitted the previous day to the coronary care unit with an acute myocardial infarction (AMI), the nurse will anticipate teaching about a. typical emotional responses to AMI. b. when patient cardiac rehabilitation will begin. c. discharge drugs such as aspirin and -blockers. d. the pathophysiology of coronary artery disease.

ANS: B Early after an AMI, the patient will want to know when resumption of usual activities can be expected. At this time, the patient's anxiety level or denial will prevent good understanding of complex information such as coronary artery disease (CAD) pathophysiology. Teaching about discharge medications should be done when the time for discharge is closer. The nurse should support the patient by decreasing anxiety rather than discussing the typical emotional response to myocardial infarction (MI). DIF: Cognitive Level: Application REF: 788-789

A patient with ST segment elevation in several electrocardiographic (ECG) leads is admitted to the emergency department (ED) and diagnosed as having an ST-segment-elevation myocardial infarction (STEMI). Which question should the nurse ask to determine whether the patient is a candidate for fibrinolytic therapy? a. "Do you take aspirin on a daily basis?" b. "What time did your chest pain begin?" c. "Is there any family history of heart disease?" d. "Can you describe the quality of your chest pain?"

ANS: B Fibrinolytic therapy should be started within 6 hours of the onset of the myocardial infarction (MI), so the time at which the chest pain started is a major determinant of the appropriateness of this treatment. The other information also will be needed, but it will not be a factor in the decision about fibrinolytic therapy. DIF: Cognitive Level: Application REF: 782-783

When caring for a patient who has survived a sudden cardiac death (SCD) event and has no evidence of an acute myocardial infarction (AMI), the nurse will anticipate teaching the patient a. that sudden cardiac death events rarely reoccur. b. about the purpose of outpatient Holter monitoring. c. how to self-administer low-molecular-weight heparin. d. to limit activities after discharge to prevent future events.

ANS: B Holter monitoring is used to determine whether the patient is experiencing dysrhythmias such as ventricular tachycardia during normal daily activities. SCD is likely to recur. Heparin will not have any effect on the incidence of SCD, and SCD can occur even when the patient is resting. DIF: Cognitive Level: Application REF: 793-794

A patient with chronic kidney disease (CKD) brings all home medications to the clinic to be reviewed by the nurse. Which medication being used by the patient indicates that patient teaching is required? a. Multivitamin with iron b. Milk of magnesia 30 mL c. Calcium phosphate (PhosLo) d. Acetaminophen (Tylenol) 650 mg

ANS: B Magnesium is excreted by the kidneys, and patients with CKD should not use over-the-counter products containing magnesium. The other medications are appropriate for a patient with CKD.

A patient who has had an acute myocardial infarction (AMI) asks the nurse about when sexual intercourse can be resumed. Which response by the nurse is best? a. "Most patients are able to enjoy intercourse without any complications." b. "Sexual activity uses about as much energy as climbing two flights of stairs." c. "The doctor will discuss sexual intercourse when your heart is strong enough." d. "Holding and cuddling are good ways to maintain intimacy after a heart attack."

ANS: B Sexual activity places about as much physical stress on the cardiovascular system as climbing two flights of stairs. The other responses do not directly address the patient's question, or may not be accurate for this patient. DIF: Cognitive Level: Application REF: 792

When caring for a dehydrated patient with acute kidney injury who is oliguric, anemic, and hyperkalemic, which of the following prescribed actions should the nurse take first? a. Insert a urinary retention catheter. b. Place the patient on a cardiac monitor. c. Administer epoetin alfa (Epogen, Procrit). d. Give sodium polystyrene sulfonate (Kayexalate).

ANS: B Since hyperkalemia can cause fatal cardiac dysrhythmias, the initial action should be to monitor the cardiac rhythm. Kayexalate and Epogen will take time to correct the hyperkalemia and anemia. The catheter allows monitoring of the urine output, but does not correct the cause of the renal failure.

When the nurse is caring for a patient who has been admitted with a severe crushing injury after an industrial accident, which laboratory result will be most important to report to the health care provider? a. Serum creatinine level 2.1 mg/dL b. Serum potassium level 6.5 mEq/L c. White blood cell count 11,500/µL d. Blood urea nitrogen (BUN) 56 mg/dL

ANS: B The hyperkalemia associated with crushing injuries may cause cardiac arrest and should be treated immediately. The nurse also will report the other laboratory values, but abnormalities in these are not immediately life threatening.

A patient with a history of benign prostatic hyperplasia (BPH) is admitted with acute urinary retention and an elevated blood urea nitrogen (BUN) and creatinine. Which of these prescribed therapies should the nurse implement first? a. Obtain renal ultrasound. b. Insert retention catheter. c. Infuse normal saline at 50 mL/hour. d. Draw blood for complete blood count.

ANS: B The patient's elevation in BUN is most likely associated with hydronephrosis caused by the acute urinary retention, so the insertion of a retention catheter is the first action to prevent ongoing postrenal failure for this patient. The other actions also are appropriate, but should be implemented after the retention catheter

A few days after experiencing a myocardial infarction (MI), the patient states, "I just had a little chest pain. As soon as I get out of here, I'm going for my vacation as planned." Which response should the nurse make? a. "Where are you planning to go for your vacation?" b. "What do you think caused your chest pain episode?" c. "Sometimes plans need to change after a heart attack." d. "Recovery from a heart attack takes at least a few weeks."

ANS: B When the patient is experiencing denial, the nurse should assist the patient in testing reality until the patient has progressed beyond this step of the emotional adjustment to MI. Asking the patient about vacation plans reinforces the patient's plan, which is not appropriate in the immediate post-MI period. Reminding the patient in denial about the MI is likely to make the patient angry and lead to distrust of the nursing staff. DIF: Cognitive Level: Application REF: 788-789

What sign or symptom is the most important for you to teach the patient who is taking gemfibrozil (Lipid) or simvastatin (Zocor)? A. Nightmares B. Severe muscle aching C. Diarrhea D. Thrombocytopenia

ANS: B The risk is rhabdomyolysis. Clinical manifestations of rhabdomyolysis are increased creatinine kinase levels and muscle tenderness. Nightmares are a side effect but not as ominous as rhabdomyolysis. Reference: 770

Which teaching points should you include when providing discharge instructions to a patient with a new permanent pacemaker and to the caregiver (select all that apply)? A. Avoid or limit air travel. B. Take and record a pulse rate daily. C. Obtain and wear a Medic Alert ID or bracelet at all times. D. Avoid lifting the arm on the side of the pacemaker above the shoulder. E. Avoid microwave ovens because they interfere with pacemaker function.

ANS: B,C,D Air travel is not restricted. Inform airport security about the pacemaker because it may set off the metal detector. If a hand-held screening wand is used, it should not be placed directly over the pacemaker. Manufacturer information may vary regarding the effect of metal detectors on the function of the pacemaker. The patient should monitor the pulse and inform the primary care provider if it drops below predetermined rate. A Medic Alert ID or bracelet should be worn at all times. The patient should avoid lifting the arm on the pacemaker side above the shoulder until it is approved by the primary care provider. Microwave ovens are safe to use and do not interfere with pacemaker function. Table 36-13 provides additional discharge teaching guidelines for a patient with a pacemaker. Reference: 837

When collecting subjective data related to the cardiovascular system, which data should be obtained from the patient (select all that apply)? A. Annual income B. Smoking history C. Religious preference D. Number of pillows used to sleep E. Blood for basic laboratory studies

ANS: B,C,D The health history should include assessment of tobacco use. The patient should be asked about cultural or religious beliefs that may influence management of the cardiovascular problem. Patients with heart failure may need to sleep with the head elevated on pillows or in a chair. Reference: 721-723

You assess a patient with complaints of chest pain for which clinical manifestations associated with an MI (select all that apply)? A. Flushing B. Ashen skin C. Diaphoresis D. Nausea and vomiting E. S3 or S4 heart sounds

ANS: B,C,D,E During the initial phase of an MI, catecholamines are released from the ischemic myocardial cells, causing increased sympathetic nervous system (SNS) stimulation. This results in the release of glycogen, diaphoresis, and vasoconstriction of peripheral blood vessels, and the patient's skin may be ashen, cool, and clammy (not flushed). Nausea and vomiting may result from reflex stimulation of the vomiting center by severe pain. Ventricular dysfunction resulting from the MI may produce abnormal S3 and S4 heart sounds. Reference: 779

Which assessment finding in a patient with hypertensive crisis is most concerning to you? A. Blood pressure of 220/140 mm Hg B. History of hypertension for 20 years C. New-onset confusion D. History of cocaine use

ANS: C A hypertensive emergency often manifests as hypertensive encephalopathy, a syndrome in which a sudden rise in blood pressure is associated with severe headache, nausea, vomiting, seizures, confusion, and coma. The blood pressure level alone is a poor indicator of the seriousness of the patient's condition; instead, it is the association between elevated blood pressure and signs of new or progressive target organ damage. Patients with chronic hypertension can tolerate much higher blood pressures than previously normotensive people. Cocaine use can cause a hypertensive emergency, but the new onset of confusion and related symptoms is the more serious indicator. The finding does not indicate how recent the cocaine use was. Reference: 756

The patient's hypertension is being managed with enalapril (Vasotec). What finding is most important for you to follow-up? A. Blood pressure of 140/90 mm Hg B. Dry, hacking cough C. Use of salt substitutes D. Puffy ankles

ANS: C A potential side effect of angiotensin-converting enzyme (ACE) inhibitors is hyperkalemia. Salt substitutes use potassium and should not be used with ACE inhibitors; this takes priority over the other findings. Reference: 750-751

The priority nursing assessment of a patient receiving intravenous (IV) nesiritide (Natrecor) to treat heart failure would be A. urine output. B. lung sounds. C. blood pressure. D. respiratory rate.

ANS: C Although all identified assessments are appropriate for a patient receiving IV nesiritide, the priority assessment is monitoring for hypotension, the main adverse effect of nesiritide. Reference: 805

A 72-year-old man with a history of aortic stenosis is admitted to the emergency department. He reports severe left-sided chest pressure radiating to the jaw. Which medication, if ordered by the health care provider, should the nurse question? A. Aspirin B. Oxygen C. Nitroglycerin D. Morphine sulfate

ANS: C Aspirin, oxygen, nitroglycerin, and morphine sulfate are all commonly used to treat acute chest pain suspected to be caused by myocardial ischemia. However, nitroglycerin should be used cautiously or avoided in patients with aortic stenosis as a significant reduction in blood pressure may occur. Chest pain can worsen because of a drop in blood pressure.

You are caring for a patient admitted with chronic obstructive pulmonary disorder (COPD), angina, and hypertension. Before administering the prescribed daily dose of atenolol (100 mg PO), you assess the patient carefully. Which adverse effect is this patient at risk for given the health history? A. Hypocapnia B. Tachycardia C. Bronchospasm D. Nausea and vomiting

ANS: C Atenolol is a cardioselective β1-adrenergic blocker that reduces blood pressure and can affect the β2-adrenergic receptors in the lungs with larger doses or with drug accumulation. Although the risk of bronchospasm is less with cardioselective β-blockers than nonselective β-blockers, atenolol should be used cautiously in patients with COPD. Reference: 749

When computing a heart rate from the ECG tracing, you count 15 of the small blocks between the R waves of a patient whose rhythm is regular. From these data, you calculate the patient's heart rate to be what? A. 60 beats/minute B. 75 beats/minute C. 100 beats/minute D. 150 beats/minute

ANS: C Because each small block on the ECG paper represents 0.04 second, 1500 of them represent 1 minute. By dividing the number of small blocks (15 in this case) into 1500, you can calculate the heart rate in a patient whose rhythm is regular (in this case, 100). Reference: 821

You are caring for a patient with chronic constrictive pericarditis. Which assessment finding reflects a more serious complication of this condition? A. Fatigue B. Peripheral edema C. Jugular venous distention D. Thickened pericardium on echocardiography

ANS: C Cardiac tamponade is a serious complication of pericarditis. As the compression of the heart increases, decreased left atrial filling causes decreased cardiac output. Neck veins usually are markedly distended because of jugular venous pressure elevation. Reference: 847

Which is the main difference between a hypertension emergency and hypertension urgency? A. Class of medication used B. Blood pressure level C. Signs of target organ damage D. Length of onset

ANS: C Hypertensive crisis refers to a hypertension emergency or hypertension urgency. An emergency has severely elevated blood pressure with evidence of acute target organ damage, especially to the central nervous system. Urgency refers to blood pressure that is severely elevated but with no clinical evidence of target organ damage. Target organ damage is a more significant indicator than an absolute blood pressure reading. Reference: 756

What is considered a risk factor for sudden cardiac death (SCD) without having any manifestations of an acute MI? A. Female gender B. Family history of heart attacks C. Diabetes mellitus D. Asthma

ANS: C It is difficult to predict who is at risk for SCD. However, left ventricular dysfunction (ejection fraction less than 30%) and ventricular dysrhythmias after an MI have been found to be the strongest predictors. Other risk factors for SCD include male gender (especially African American men), family history of premature atherosclerosis, tobacco use, diabetes mellitus, hypercholesterolemia, hypertension, and cardiomyopathy. Reference: 793

What term is applied to a pacemaker that is implanted for the purpose of terminating atrial tachycardias? A. Temporary pacemaker B. Antitachycardia pacing C. Overdriving pacing D. Cardiac resynchronization therapy

ANS: C Overdrive pacing involves pacing the atrium at a rate of 200 to 500 impulses per minute in an attempt to terminate atrial tachycardias (e.g., atrial flutter with a rapid ventricular response). A temporary pacemaker is a category of pacemakers used temporarily with the power source outside the body. Antitachycardia pacing delivers a stimulus to the ventricle to terminate tachydysrhythmias. Cardiac resynchronization therapy is a technique to resynchronize the cardiac cycle by pacing both ventricles. Reference: 835

The patient is positioned sitting upright and learning forward. After exhalation, you auscultate a high-pitched scratchy heart sound intermittently at the apex. What is the best interpretation of this sound? A. The patient has a I/VI heart murmur. B. An S4 atrial gallop is heard. C. Pericardial friction rub is caused by pericarditis. D. Normal splitting of the S2 cardiac sound is heard.

ANS: C Pericardial friction rubs are sounds caused by friction that occurs when inflamed surfaces of the pericardium (pericarditis) move against each other. They are high-pitched, scratchy sounds that are heard best at the apex with the patient upright and leaning forward and after expiration. A murmur is caused by turbulent blood flow across diseased heart values; a I/VI murmur is barely audible. An S4 heart sound is a low-frequency vibration that precedes the S1. Normal splitting of S2 is best heard at the pulmonic area during inspiration. Reference: 726

Your patient is recovering from an uncomplicated MI. Which rehabilitation guideline is a priority to include in your teaching? A. Refrain from sexual activity for a minimum of 3 weeks. B. Plan a diet program that aims for a 1- to 2-pound weight loss per week. C. Begin an exercise program that aims for at least five 30-minute sessions per week. D. Consider the use of erectile agents and prophylactic nitroglycerin before engaging in sexual activity.

ANS: C Physical activity should be regular, rhythmic, and repetitive, using large muscles to build up endurance (e.g., walking, cycling, swimming, rowing). Physical activity sessions should be at least 30 minutes long. Instruct the patient to begin slowly at personal tolerance levels (perhaps only 5 to 10 minutes) and build up to 30 minutes. Reference: 792

Important teaching for the patient scheduled for a radiofrequency catheter ablation procedure includes explaining that A. ventricular bradycardia may be induced and treated during the procedure. B. catheters will be placed in both femoral arteries to allow double-catheter use. C. the procedure will destroy areas of the conduction system that are causing rapid heart rhythms. D. a general anesthetic will be given to prevent the awareness of any "sudden cardiac death" experiences.

ANS: C Radiofrequency catheter ablation therapy uses electrical energy to "burn" or ablate areas of the conduction system as definitive treatment for tachydysrhythmias. Reference: 837

The patient says, "The doctor said my heart was enlarged. Isn't that a good thing, like bigger muscles?" What is your response? A. Your heart will become more effective as a pump. B. Your chambers can now hold more blood to be pumped into the circulation. C. Your heart will lose some of its effectiveness as a pump and require more oxygen. D. Your heart will store more blood in its chambers, but you could be at risk for clots.

ANS: C Remodeling involves hypertrophy of the ventricular myocytes, resulting in large, abnormally shaped contractile cells. The altered geometric shape of the ventricles eventually leads to increased ventricular mass, increased wall tension, increased oxygen consumption, and impaired contractility. Although the ventricles become larger, they become less effective pumps. Ventricular remodeling is a risk factor for life-threatening dysrhythmias and sudden cardiac death (SCD). Reference: 799

The nurse prepares to administer digoxin (Lanoxin) 0.125 mg to an 82-year-old man admitted with influenza and a history of chronic heart failure. What should the nurse assess before giving the medication? A. Prothrombin time B. Urine specific gravity C. Serum potassium level D. Hemoglobin and hematocrit

ANS: C Serum potassium should be monitored because hypokalemia increases the risk for digoxin toxicity. Changes in prothrombin time, urine specific gravity, and hemoglobin or hematocrit would not require holding the digoxin dose.

Which statement best describes the electrical activity of the heart represented by measuring the PR interval on the ECG? A. The length of time it takes to depolarize the atrium B. The length of time it takes for the atria to depolarize and repolarize C. The length of time for the electrical impulse to travel from the sinoatrial (SA) node to the Purkinje fibers D. The length of time it takes for the electrical impulse to travel from the SA node to the AV node

ANS: C The electrical impulse in the heart must travel from the SA node through the AV node and into the Purkinje fibers for synchronous atrial and ventricular contraction to occur. The P wave represents atrial contraction, and the R wave is part of the QRS complex that represents ventricular contraction. When measuring the time from the beginning of the P wave to the beginning of the QRS (PR interval), you are identifying the length of time it takes for the electrical impulse to travel from the SA node to the Purkinje fibers. Reference: 818-819

What is the purpose of the head-up tilt test? A. To determine whether a fluid volume deficit exists B. To assess for dysrhythmias when under stress C. To determine whether there is positional decreased venous return to the heart D. To evaluate for peripheral vascular disease

ANS: C The head-up tilt test is used to see whether there is cardioneurogenic syncope with increased venous pooling that occurs in the upright position. This reduces the venous return to the heart. Reference: 839

You are preparing to administer digoxin to a patient with heart failure. In preparation, you review the following laboratory results: sodium level of 139 mEq/L, potassium level of 3.0 mEq/L, chloride level of 103 mEq/L, and glucose level 106 mg/dL. What should you do at this time? A. Give the digoxin with a salty snack, such as crackers. B. Give the digoxin with extra fluids to dilute the sodium level. C. Withhold the dose and report the potassium level. D. Withhold the daily dose until the next day.

ANS: C The normal potassium level is 3.5 to 5.0 mEq/L. The patient is hypokalemic, which makes the patient more prone to digoxin toxicity. For this reason, you should withhold the dose and report the potassium level. The physician may order the digoxin to be given after the potassium level has been corrected and rises to within normal range. Reference: 807

The patient presents to the emergency department with crushing chest pain. The electrocardiogram (ECG) is completed within 5 minutes and is normal. What future action is most important? A. Administer 30 mL of antacid. B. Reassure the patient. C. Repeat the ECG later. D. Obtain blood for a complete blood cell (CBC) count.

ANS: C The normal progression is ischemia, injury, infarction, and resolution of the infarction. Initial ECG results can be negative, but serial ECGs may show the injury and infarction. When an initial 12-lead ECG is nondiagnostic, serial 12-lead ECGs are done every 2 to 4 hours. Reference: 780

In caring for a patient admitted with poorly controlled hypertension, you understand that which laboratory test result indicates the presence of target organ damage resulting from the primary diagnosis? A. BUN level of 15 mg/dL B. Serum uric acid level of 3.8 mg/dL C. Serum creatinine level of 2.6 mg/dL D. Serum potassium level of 3.5 mEq/L

ANS: C The normal serum creatinine level is 0.6 to 1.3 mg/dL. This elevated level indicates damage to the kidneys. Reference: 743

The patient has an electrocardiographic (ECG) tracing that is 50 beats/minute, the rhythm is regular, and there is a P wave before every QRS complex. The QRS has a normal shape and duration, and the PR interval is normal. What is you response? A. Administer atropine by intravenous push (IVP). B. Administer epinephrine by IVP. C. Monitor the patient for syncope. D. Attach an external pacemaker.

ANS: C The rhythm described is sinus bradycardia. Treatment depends on the patient's response and whether adequate perfusion is occurring. If the patient tolerates the rhythm, no treatment is given. Reference: 824

A patient admitted with acute coronary syndrome (ACS) has continuous ECG monitoring. An examination of the rhythm strip reveals the following characteristics: atrial rate of 74 beats/minute and regular; ventricular rate of 62 beats/minute and irregular; P wave with a normal shape; PR interval that lengthens progressively until a P wave is not conducted; and QRS complex with a normal shape. Your priority nursing intervention involves A. performing synchronized cardioversion. B. administering 1 mg of epinephrine by IVP. C. observing for symptoms of hypotension or angina. D. preparing the patient for a transcutaneous pacemaker.

ANS: C The rhythm is a second-degree AV block, type I (Mobitz I or Wenckebach heart block). The rhythm is identified by a gradual lengthening of the PR interval. Type I AV block is usually a result of myocardial ischemia or infarction and typically is transient and well tolerated. You should assess for bradycardia, hypotension, and angina. If the patient becomes symptomatic, atropine or a temporary pacemaker may be needed. Reference: 830

The patient is prescribed propranolol (Inderal) as part of the management of chronic stable angina. What nursing assessment should be done before the drug's administration? A. Breath sounds B. Incidence of gastric esophageal reflux (GERD) C. Apical heart rate D. Presence of depression

ANS: C The β-adrenergic blockers are preferred drugs for the management of chronic stable angina. They reduce myocardial contractility, heart rate, and blood pressure, all which reduce the myocardial oxygen demand. They decrease morbidity and mortality in patients with coronary artery disease (CAD). Side effects include bradycardia and hypotension, and the patient should be assessed, with the drug withheld, if either value is too low. Reference: 776

Which statement is accurate regarding blood work results in assessing cardiac function? A. C-reactive protein (CRP) is an independent risk factor for autoimmune diseases. B. Homocysteine is acquired from high dietary consumption of cholesterol. C. B-type natriuretic peptide (BNP) helps to differentiate between cardiac and respiratory causes of dyspnea. D. Myoglobin is the biomarker of choice in the diagnosis of myocardial infarction (MI).

ANS: C There are three natriuretic peptides. BNP is found in ventricles. BNP has emerged as the marker of choice for determining whether there is a cardiac or respiratory cause of dyspnea. CRP is a protein produced by the liver during periods of acute inflammation. It is emerging as an independent risk factor for coronary artery disease (CAD). Homocysteine is an amino acid produced during protein catabolism. Elevated levels can be hereditary or acquired from dietary deficiencies of B6, B12, or folate. Troponin is the biomarker of choice in the diagnosis of MI. Myoglobin is found in cardiac and skeletal muscles, and it is used as an indicator in early myocardial injury. Reference: 727

What is the main reason a patient is instructed to take the nitroglycerin patch off for 8 hours every night? A. Hypotension will occur. B. It is not needed because the patient is inactive. C. The interruption prevents tolerance to nitroglycerin. D. It causes nightmares.

ANS: C Unless there is nocturnal angina, an 8-hour nitrate-free period is suggested because tolerance to nitroglycerin-induced vasodilation can develop. Reference: 776

Which ECG characteristics are consistent with a diagnosis of ventricular tachycardia (VT)? A. Unmeasurable rate and rhythm B. Rate of 150 beats/minute; inverted P wave C. Rate of 200 beats/minute; P wave not visible D. Rate of 125 beats/minute; normal QRS complex

ANS: C VT is associated with a rate of 150 to 250 beats/minute, and the P wave is not normally visible. P-wave inversion and a normal QRS complex are not associated with VT. Rate and rhythm are not measurable in ventricular fibrillation. Reference: 831

Following an acute myocardial infarction (AMI), a patient ambulates in the hospital hallway. When the nurse is evaluating the patient's response, which of these assessment data would indicate that the exercise level should be decreased? a. BP changes from 118/60 to 126/68 mm Hg. b. Oxygen saturation drops from 100% to 98%. c. Heart rate increases from 66 to 90 beats/minute. d. Respiratory rate goes from 14 to 22 breaths/minute.

ANS: C A change in heart rate of more than 20 beats or more indicates that the patient should stop and rest. The increases in BP and respiratory rate, and the slight decrease in oxygen saturation, are normal responses to exercise. DIF: Cognitive Level: Application REF: 792

Which data obtained when assessing a patient who had a kidney transplant 8 years ago and who is receiving the immunosuppressants tacrolimus (Prograf), cyclosporine (Sandimmune), and prednisone (Deltasone) will be of most concern to the nurse? a. The blood glucose is 144 mg/dL. b. The patient's blood pressure is 150/92. c. There is a nontender lump in the axilla. d. The patient has a round, moonlike face.

ANS: C A nontender lump suggests a malignancy such as a lymphoma, which could occur as a result of chronic immunosuppressive therapy. The elevated glucose, moon face, and hypertension are possible side effects of the prednisone and should be addressed, but they are not as great a concern as the possibility of a malignancy

Which data obtained when assessing a patient who had a kidney transplant 8 years ago and who is receiving the immunosuppressants tacrolimus (Prograf), cyclosporine (Sandimmune), and prednisone (Deltasone) will be of most concern to the nurse? a. The blood glucose is 144 mg/dL. b. The patient's blood pressure is 150/92. c. There is a nontender lump in the axilla. d. The patient has a round, moonlike face.

ANS: C A nontender lump suggests a malignancy such as a lymphoma, which could occur as a result of chronic immunosuppressive therapy. The elevated glucose, moon face, and hypertension are possible side effects of the prednisone and should be addressed, but they are not as great a concern as the possibility of a malignancy.

A patient admitted to the coronary care unit (CCU) with an ST-segment-elevation myocardial infarction (STEMI) is restless and anxious. The blood pressure is 86/40 and heart rate is 110. Based on this information, which nursing diagnosis is a priority for the patient? a. Acute pain related to myocardial ischemia b. Anxiety related to perceived threat of death c. Decreased cardiac output related to cardiogenic shock d. Activity intolerance related to decreased cardiac output

ANS: C All the nursing diagnoses may be appropriate for this patient, but the hypotension indicates that the priority diagnosis is decreased cardiac output, which will decrease perfusion to all vital organs (e.g., brain, kidney, heart). DIF: Cognitive Level: Application REF: 786

Which of these statements made by a patient with coronary artery disease after the nurse has completed teaching about the therapeutic lifestyle changes (TLC) diet indicates that further teaching is needed? a. "I will switch from whole milk to 1% or nonfat milk." b. "I like fresh salmon and I will plan to eat it more often." c. "I will miss being able to eat peanut butter sandwiches." d. "I can have a cup of coffee with breakfast if I want one."

ANS: C Although only 30% of the daily calories should come from fats, most of the fat in the TLC diet should come from monosaturated fats such as are found in nuts, olive oil, and canola oil. The patient can include peanut butter sandwiches as part of the TLC diet. The other patient comments indicate a good understanding of the TLC diet. DIF: Cognitive Level: Application REF: 768

Which parameter will be most important for the nurse to consider when titrating the IV fluid infusion rate immediately after a patient has had kidney transplantation? a. Heart rate b. Blood urea nitrogen (BUN) level c. Urine output d. Creatinine clearance

ANS: C Fluid volume is replaced based on urine output after transplant because the urine output can be as high as a liter an hour. The other data will be monitored but are not the most important determinants of fluid infusion rate.

To assist the patient with coronary artery disease (CAD) in making appropriate dietary changes, which of these nursing interventions will be most effective? a. Instruct the patient that a diet containing no saturated fat and minimal sodium will be necessary. b. Emphasize the increased risk for cardiac problems unless the patient makes the dietary changes. c. Assist the patient to modify favorite high-fat recipes by using monosaturated oils when possible. d. Provide the patient with a list of low-sodium, low-cholesterol foods that should be included in the diet.

ANS: C Lifestyle changes are more likely to be successful when consideration is given to the patient's values and preferences. The highest percentage of calories from fat should come from monosaturated fats. Although low-sodium and low-cholesterol foods are appropriate, providing the patient with a list alone is not likely to be successful in making dietary changes. Removing saturated fat from the diet completely is not a realistic expectation; up to 7% of calories in the therapeutic lifestyle changes (TLC) diet can come from saturated fat. Telling the patient about the increased risk without assisting further with strategies for dietary change is unlikely to be successful. DIF: Cognitive Level: Application REF: 767-768

A patient who has acute glomerulonephritis is hospitalized with acute kidney injury (AKI) and hyperkalemia. Which information will the nurse obtain to evaluate the effectiveness of the prescribed calcium gluconate IV? a. Urine output b. Calcium level c. Cardiac rhythm d. Neurologic status

ANS: C The calcium gluconate helps prevent dysrhythmias that might be caused by the hyperkalemia. The nurse will monitor the other data as well, but these will not be helpful in determining the effectiveness of the calcium gluconate.

During the administration of the fibrinolytic agent to a patient with an acute myocardial infarction (AMI), the nurse should stop the drug infusion if the patient experiences a. bleeding from the gums. b. surface bleeding from the IV site. c. a decrease in level of consciousness. d. a nonsustained episode of ventricular tachycardia.

ANS: C The change in level of consciousness indicates that the patient may be experiencing intracranial bleeding, a possible complication of fibrinolytic therapy. Bleeding of the gums and prolonged bleeding from IV sites are expected side effects of the therapy. The nurse should address these by avoiding any further injuries, but they are not an indication to stop infusion of the fibrinolytic medication. A nonsustained episode of ventricular tachycardia is a common reperfusion dysrhythmia and may indicate that the therapy is effective. DIF: Cognitive Level: Application REF: 783-784

Which assessment finding by the nurse who is caring for a patient who has had coronary artery bypass grafting using a right radial artery graft is most important to communicate to the physician? a. Complaints of incisional chest pain b. Crackles audible at both lung bases c. Pallor and weakness of the right hand d. Redness on either side of the chest incision

ANS: C The changes in the right hand indicate compromised blood flow, which requires immediate evaluation and actions such as prescribed calcium channel blockers or surgery. The other changes are expected and/or require nursing interventions. DIF: Cognitive Level: Application REF: 788

A patient with stage 2 chronic kidney disease (CKD) is scheduled for an intravenous pyelogram (IVP). Which of these orders for the patient will the nurse question? a. NPO for 6 hours before IVP procedure b. Normal saline 500 mL IV before procedure c. Ibuprofen (Advil) 400 mg PO PRN for pain d. Dulcolax suppository 4 hours before IVP procedure

ANS: C The contrast dye used in IVPs is potentially nephrotoxic, and concurrent use of other nephrotoxic medications such as the NSAIDs should be avoided. The suppository and NPO status are necessary to ensure adequate visualization during the IVP. IV fluids are used to ensure adequate hydration, which helps reduce the risk for contrast-induced renal failure.

Two hours after a kidney transplant, the nurse obtains all of the following data when assessing the patient. Which information is most important to communicate to the health care provider? a. The urine output is 900 to 1100 mL/hr. b. The blood urea nitrogen (BUN) and creatinine levels are elevated. c. The patient's central venous pressure (CVP) is decreased. d. The patient has level 8 (on a 10-point scale) incisional pain.

ANS: C The decrease in CVP suggests hypovolemia, which must be rapidly corrected to prevent renal hypoperfusion and acute tubular necrosis. The other information is not unusual in a patient after a transplant.

After the nurse has finished teaching a patient about use of sublingual nitroglycerin (Nitrostat), which patient statement indicates that the teaching has been effective? a. "I can expect indigestion as a side effect of nitroglycerin." b. "I can only take the nitroglycerin if I start to have chest pain." c. "I will call an ambulance if I still have pain 5 minutes after taking the nitroglycerin." d. "I will help slow down the progress of the plaque formation by taking nitroglycerin."

ANS: C The emergency medical services (EMS) system should be activated when chest pain or other symptoms are not completely relieved 5 minutes after taking one nitroglycerin. Nitroglycerin can be taken to prevent chest pain or other symptoms from developing (e.g., before intercourse). Gastric upset is not an expected side effect of nitroglycerin. Nitroglycerin does not impact the underlying pathophysiology of coronary artery atherosclerosis. DIF: Cognitive Level: Application REF: 775-776

Which information about a patient who was admitted 10 days previously with acute kidney injury (AKI) caused by dehydration will be most important for the nurse to report to the health care provider? a. The blood urea nitrogen (BUN) level is 67 mg/dL. b. The creatinine level is 3.0 mg/dL. c. Urine output over an 8-hour period is 2500 mL. d. The glomerular filtration rate is <30 mL/min/1.73m2.

ANS: C The high urine output indicates a need to increase fluid intake to prevent hypovolemia. The other information is typical of AKI and will not require a change in therapy.

The nurse teaches the patient being evaluated for rhythm disturbances with a Holter monitor to a. exercise more than usual while the monitor is in place. b. remove the electrodes when taking a shower or tub bath. c. keep a diary of daily activities while the monitor is worn. d. connect the recorder to a telephone transmitter once daily.

ANS: C The patient is instructed to keep a diary describing daily activities while Holter monitoring is being accomplished to help correlate any rhythm disturbances with patient activities. Patients are taught that they should not take a shower or bath during Holter monitoring and that they should continue with their usual daily activities. The recorder stores the information about the patient's rhythm until the end of the testing, when it is removed and the data are analyzed. DIF: Cognitive Level: Application REF: 729

A transesophageal echocardiogram (TEE) is ordered for a patient with possible endocarditis. Which of these actions included in the standard TEE orders will the nurse need to accomplish first? a. Administer O2 per mask. b. Start a large-gauge IV line. c. Place the patient on NPO status. d. Give lorazepam (Ativan) 1 mg IV.

ANS: C The patient will need to be NPO for 6 hours preceding the TEE, so the nurse should place the patient on NPO status as soon as the order is received. The other actions also will need to be accomplished but not until just before or during the procedure. DIF: Cognitive Level: Application REF: 730

Which statement by a patient with stage 5 chronic kidney disease (CKD) indicates that the nurse's teaching about management of CKD has been effective? a. "I need to try to get more protein from dairy products." b. "I will try to increase my intake of fruits and vegetables." c. "I will measure my urinary output each day to help calculate the amount I can drink." d. "I need to take the erythropoietin to boost my immune system and help prevent infection."

ANS: C The patient with end-stage renal disease is taught to measure urine output as a means of determining an appropriate oral fluid intake. Erythropoietin is given to increase the red blood cell count and will not offer any benefit for immune function. Dairy products are restricted because of the high phosphate level. Many fruits and vegetables are high in potassium and should be restricted in the patient with CKD.

A patient with severe heart failure develops elevated blood urea nitrogen (BUN) and creatinine levels. The nurse will plan care to meet the goal of a. replacing fluid volume. b. preventing hypertension. c. maintaining cardiac output. d. diluting nephrotoxic substances.

ANS: C The primary goal of treatment for acute kidney injury (AKI) is to eliminate the cause and provide supportive care while the kidneys recover. Because this patient's heart failure is causing AKI, the care will be directed toward treatment of the heart failure. For renal failure caused by hypertension, hypovolemia, or nephrotoxins, the other responses would be correct.

The nurse has just received change-of-shift report about the following four patients. Which patient should the nurse assess first? a. 38-year-old who has pericarditis and is complaining of sharp, stabbing chest pain b. 45-year-old who had a myocardial infarction (MI) 4 days ago and is anxious about the planned discharge c. 51-year-old with unstable angina who has just returned to the unit after having a percutaneous coronary intervention (PCI) d. 60-year-old with variant angina who is to receive a scheduled dose of nifedipine (Procardia)

ANS: C This patient is at risk for bleeding from the arterial access site for the PCI, so the nurse should assess the patient's blood pressure, pulse, and the access site immediately. The other patients also should be assessed as quickly as possible, but assessment of this patient has the highest priority.

A patient who has had severe chest pain for several hours is admitted with a diagnosis of possible acute myocardial infarction (AMI). Which of these ordered laboratory tests should the nurse monitor to help determine whether the patient has had an AMI? a. Homocysteine b. C-reactive protein c. Cardiac-specific troponin I and troponin T d. High-density lipoprotein (HDL) cholesterol

ANS: C Troponin levels increase about 4 to 6 hours after the onset of myocardial infarction (MI). The other laboratory data are useful in determining the patient's risk for developing coronary artery disease (CAD) but are not helpful in determining whether an acute MI is in progress. DIF: Cognitive Level: Comprehension REF: 780-781

Which instruction given to a patient who is about to undergo Holter monitoring is most appropriate? A. You may remove the monitor only to shower or bathe. B. You should connect the monitor whenever you feel symptoms. C. You should refrain from exercising while wearing this monitor. D. You must keep a diary of all your activities and symptoms.

ANS: D A Holter monitor is worn for at least 24 hours while a patient continues with usual activity and keeps a diary of activities and symptoms. The patient should not take a bath or shower while wearing this monitor. Reference: 729

The blood pressure of a 71-year-old patient admitted with pneumonia is 160/70 mm Hg. What age-related change contributes to this finding? A. Stenosis of the heart valves B. Decreased adrenergic sensitivity C. Increased parasympathetic activity D. Loss of elasticity in arterial vessels

ANS: D An age-related change that increases the risk of systolic hypertension is a loss of elasticity in the arterial walls. Because of the increasing resistance to flow, pressure is increased within the blood vessel, and hypertension results. Reference: 720

In reviewing medication instructions with a patient being discharged on antihypertensive medications, which statement would be most appropriate for you to make when discussing atenolol (Tenormin)? A. A fast heart rate is a side effect to watch for while taking atenolol. B. Stop the drug and notify your doctor if you experience any nausea or vomiting. C. Because this drug may affect the lungs in large doses, it may also help your breathing. D. Make position changes slowly, especially when going from a lying down to a standing position.

ANS: D Atenolol is a β1-adrenergic blocker and antihypertensive agent that can cause orthostatic hypotension. For this reason, the patient should be instructed to rise slowly, especially when moving from a recumbent to a standing position. Reference: 749

Which is accurate regarding measuring arterial blood pressure? A. Radial pulse should be used to detect Korotkoff sounds. B. Inflate cuff to previous systolic blood pressure when obtaining blood pressure values. C. When using Doppler, the point at which the sound disappears is the systolic pressure. D. Auscultatory gap is loss of sound between systolic and diastolic blood pressures.

ANS: D Auscultatory gap is the loss of sound between the systolic and diastolic blood pressures, and it occasionally is heard. Korotkoff sounds are the blood pressure measured externally by auscultating for sounds of turbulent blood flow through a compressed artery. The brachial artery is recommended. The cuff should be inflated to a pressure 20 to 30 mm Hg above the systolic pressure. With Doppler, the cuff is inflated 20 to 30 mm Hg above the point at which the sound disappears. When the sound returns during deflation, it is the systolic blood pressure. Reference: 719

A major consideration in the management of the older adult with hypertension is to A. prevent pseudohypertension from converting to true hypertension. B. recognize that the older adult is less likely to comply with the drug therapy than a younger adult. C. ensure that the patient receives larger initial doses of antihypertensive drugs because of impaired absorption. D. use careful technique in assessing the blood pressure of the patient because of a possible auscultatory gap.

ANS: D Careful technique is important in assessing blood pressure in older adults. In some older people, there is a wide gap between the first Korotkoff sound and subsequent beats, called an auscultatory gap. Failure to inflate the cuff high enough may result in seriously underestimating systolic blood pressure. Reference: 756

What is a classic manifestation of chronic stable angina? A. Pain remains during rest. B. Characteristics vary between episodes. C. Discomfort is described as sharp or stabbing. D. Discomfort often is described as squeezing or choking.

ANS: D Chronic, stable angina is described as pain, pressure, or an ache in the chest. It is an unpleasant feeling, often described as a constrictive, squeezing, heavy, choking, or suffocating sensation. It usually is relieved with rest or when the precipitating factor is removed. The symptoms and triggers tend to remain consistent. Angina is rarely sharp or stabbing and usually does not change with position or breathing.

13. Priority nursing management for a patient with myocarditis includes interventions related to A. meticulous skin care. B. antibiotic prophylaxis. C. tight glycemic control. D. oxygenation and ventilation.

ANS: D General supportive measures for management of myocarditis include interventions to improve ventilation and oxygenation (oxygen therapy, bed rest, and restricted activity). Reference: 850

Which statement is true about hypertrophic cardiomyopathy? A. It is most common in women. B. It involves enlargement of both ventricles. C. Ventricular dilation results from hypertrophy. D. It is the most common cause of sudden cardiac death in otherwise healthy young people.

ANS: D Hypertrophic cardiomyopathy is the leading cause of sudden cardiac death among healthy young people. It is asymmetric left ventricular hypertrophy without ventricular dilation. It is more common among men than women. Reference: 861

The patient had an SCD incident caused by a lethal rhythm and now has an implantable cardioverter-defibrillator (ICD). He arrives in the emergency department today unresponsive and in ventricular fibrillation. What action should you take? A. Administer amiodarone (Cordarone). B. Administer epinephrine. C. Check the ICD's battery. D. Defibrillate.

ANS: D If the patient is in ventricular fibrillation and unresponsive, the ICD is not working. Defibrillation is the only effective means to treat ventricular fibrillation initially, not drugs. Amiodarone is used in treating patients with a history of SCD after the initial episode. Reference: 793-794

In teaching a patient with hypertension about controlling the condition, you recognize that A. all patients with elevated blood pressure require medication. B. it is not necessary to limit salt in the diet if taking a diuretic. C. obese persons must achieve a normal weight to lower blood pressure. D. lifestyle modifications are indicated for all persons with elevated blood pressure.

ANS: D Lifestyle modifications are indicated for all patients with prehypertension and hypertension. Reference: 745

A drug used in the management of a patient with acute decompensated heart failure (ADHF) and pulmonary edema that decreases preload and afterload and provides relief of anxiety is A. amrinone. B. furosemide. C. dobutamine. D. morphine sulfate.

ANS: D Morphine sulfate reduces preload and afterload and is frequently used in the treatment of ADHF and pulmonary edema. It dilates the pulmonary and systemic blood vessels, a goal in decreasing pulmonary pressures and improving the gas exchange. Intravenous morphine decreases oxygen demands, which may be increased as a result of anxiety and subsequent increased musculoskeletal and respiratory activity. When morphine is used, the patient often experiences relief from dyspnea and the associated anxiety. Reference: 805

In assessing the patient with hypertrophic cardiomyopathy, the nurse expects to find A. distended neck veins. B. orthostatic hypotension. C. bilateral wheezing on inspiration. D. an exaggerated apical pulse that is displaced laterally.

ANS: D On palpation of the chest, the apical impulse can be exaggerated and displaced laterally. Auscultation may reveal an S4 heart sound and a systolic murmur between the apex and the sternal border. Syncope occurs, but it is unrelated to orthostatic hypotension. The patient is not in heart failure, and neck veins are not distended, nor is there wheezing on auscultation. Reference: 862

The patient with hypertensive urgency is treated as an outpatient with clonidine (Catapres). What is most important for you to teach this patient? A. Take the drug on an empty stomach. B. Diarrhea is a common side effect. C. Use moisturizing eyedrops. D. Change positions slowly.

ANS: D Patients should be instructed to change positions slowly to limit orthostatic hypotension. The drug can be taken with or without food. Constipation can be a side effect. The medication can cause eye dryness, but the instruction about position change is the most important. Reference: 757

The patient with chronic stable angina is prescribed propranolol (Inderal). You should question the order when noticing what in the patient's history? A. Taking sildenafil (Viagra) B. Taking an angiotensin-converting enzyme (ACE) inhibitor C. History of myocardial infarction D. Asthma

ANS: D Patients with asthma should avoid β-adrenergic blockers because they can contribute to bronchoconstriction and wheezing. Viagra is avoided in patients taking nitrates. Reference: 776

The patient is prescribed niacin to help reduce cholesterol. The patient tells you he does not take the medication regularly because it causes uncomfortable flushing in his face and neck. What should you teach the patient to do? A. Administer the medication with food. B. Take the drug at bedtime with milk. C. Administer the drug in split doses. D. Take ibuprofen 30 minutes before the medication.

ANS: D Premedicating with aspirin or a nonsteroidal antiinflammatory drug (NSAID) 30 minutes before taking niacin can reduce flushing. Taking time-released niacin can also prevent flushing. Reference: 770

You are administering the prescribed fibrinolytic agent to a patient diagnosed with an acute MI. You should stop the therapy and notify the physician when observing what patient manifestation? A. Bleeding from the intravenous (IV) line site B. Accelerated idioventricular rhythm C. Elevated ST segment on the ECG D. Sudden change in level of consciousness

ANS: D Signs and symptoms of major bleeding require the therapy to be stopped. A major complication of fibrinolytic therapy is bleeding. Signs and symptoms of major bleeding include a drop in blood pressure, an increase in heart rate, a sudden change in the patient's level of consciousness, or blood in the urine or stool. Minor bleeding can be controlled by applying a pressure dressing or ice pack. Reperfusion dysrhythmias are generally self-limiting and do not require aggressive treatment. Elevated ST segment on the ECG indicates an MI and need for the treatment. Reference: 783

The patient has chronic atrial fibrillation (AF). What action do you anticipate? A. Monitoring the PR interval B. Defibrillation with 360 joule C. Teaching the patient to monitor the pulse deficit D. Teaching the patient to take an anticoagulant daily

ANS: D The chaotic atrial activity results in blood stasis that can lead to embolic events. Patients with chronic AF are given an anticoagulant, most often warfarin (Coumadin), to prevent the formation of emboli. There is no PR interval in AF because the P wave is absent, replaced by chaotic fibrillatory waves. Defibrillation is an elective procedure in chronic AF and is performed at lower levels of electricity. Pulse deficit is a higher-level skill and is not taught to the patient. Reference: 827

Which finding is considered the hallmark finding in acute pericarditis? A. Dysphagia B. Left shoulder pain C. Bilateral lung crackles D. Pericardial friction rub

ANS: D The hallmark finding in acute pericarditis is a pericardial friction rub. The rub is a scratching, grating, high-pitched sound thought to result from friction between the roughened pericardial and epicardial surfaces. It is best heard with the stethoscope's diaphragm placed at the lower left sternal border of the chest with the patient leaning forward. The pericardial friction rub does not radiate widely or vary in timing from the heartbeat. The patient does not experience difficulty swallowing or have lung crackles. Left shoulder pain does occur but is not the hallmark. Reference: 847

A patient was admitted to the emergency department 24 hours earlier with complaints of chest pain that were subsequently attributed to ST-segment-elevation myocardial infarction (STEMI). Which complication of MI should you anticipate? A. Unstable angina B. Cardiac tamponade C. Sudden cardiac death D. Cardiac dysrhythmias

ANS: D The most common complication after an MI is dysrhythmias, which occur in c% of patients. Unstable angina is a considered a precursor to MI rather than a complication. Cardiac tamponade is a rare event, and sudden cardiac death is defined as an unexpected death from cardiac causes. Cardiac dysfunction in the period after an MI is not characterized as sudden cardiac death. Reference: 779

When caring for a patient who has just arrived on the medical-surgical unit after having cardiac catheterization, which nursing action should the nurse delegate to an LPN/LVN? a. Perform the initial assessment of the catheter insertion site. b. Teach the patient about the usual postprocedure plan of care. c. Check the rate on the infusion pump used to administer heparin. d. Administer the scheduled aspirin and lipid-lowering medication.

ANS: D Administration of oral medications is within the scope of practice for LPNs/LVNs. The initial assessment of the patient, patient teaching, and administration of intravenous anticoagulant medications should be done by the RN. DIF: Cognitive Level: Application REF: 793

When developing a health teaching plan for a 60-year-old man with the following risk factors for coronary artery disease (CAD), the nurse should focus on the a. family history of coronary artery disease. b. increased risk associated with the patient's gender. c. high incidence of cardiovascular disease in older people. d. elevation of the patient's serum low density lipoprotein (LDL) level.

ANS: D Because family history, gender, and age are nonmodifiable risk factors, the nurse should focus on the patient's LDL level. Decreases in LDL will help reduce the patient's risk for developing CAD. DIF: Cognitive Level: Application REF: 767

When the nurse is monitoring a patient who is undergoing exercise (stress) testing on a treadmill, which assessment finding requires the most rapid action by the nurse? a. Patient complaint of feeling tired. b. Pulse change from 80 to 96 beats/minute. c. BP increase from 134/68 to 150/80 mm Hg. d. Electrocardiographic (ECG) changes indicating coronary ischemia.

ANS: D ECG changes associated with coronary ischemia (such as T-wave inversions and ST segment depression) indicate that the myocardium is not getting adequate oxygen delivery and that the exercise test should be terminated immediately. Increases in BP and heart rate (HR) are normal responses to aerobic exercise. Tiredness also is normal as the intensity of exercise increases during the stress testing. DIF: Cognitive Level: Application REF: 730

Which information will be most useful to the nurse in evaluating improvement in kidney function for a patient who is hospitalized with acute kidney injury (AKI)? a. Blood urea nitrogen (BUN) level b. Urine output c. Creatinine level d. Calculated glomerular filtration rate (GFR)

ANS: D GFR is the preferred method for evaluating kidney function. BUN levels can fluctuate based on factors such as fluid volume status. Urine output can be normal or high in patients with AKI and does not accurately reflect kidney function. Creatinine alone is not an accurate reflection of renal function.

Before administration of calcium carbonate (Caltrate) to a patient with chronic kidney disease (CKD), the nurse should check the laboratory value for a. creatinine. b. potassium. c. total cholesterol. d. serum phosphate.

ANS: D If serum phosphate is elevated, the calcium and phosphate can cause soft tissue calcification. The calcium carbonate should not be given until the phosphate level is lowered. Total cholesterol, creatinine, and potassium values do not affect whether calcium carbonate should be administered.

To determine the effects of therapy for a patient who is being treated for heart failure, which laboratory result will the nurse plan to review? a. Myoglobin b. Homocysteine (Hcy) c. Low-density lipoprotein (LDL) d. B-type natriuretic peptide (BNP)

ANS: D Increased levels of BNP are a marker for heart failure. The other laboratory results would be used to assess for myocardial infarction (myoglobin) or risk for coronary artery disease (Hcy and LDL). DIF: Cognitive Level: Application REF: 727 | 733

Which action by a patient who is using peritoneal dialysis (PD) indicates that the nurse should provide more teaching about PD? a. The patient slows the inflow rate when experiencing pain. b. The patient leaves the catheter exit site without a dressing. c. The patient plans 30 to 60 minutes for a dialysate exchange. d. The patient cleans the catheter while taking a bath every day.

ANS: D Patients are encouraged to take showers rather than baths to avoid infections at the catheter insertion side. The other patient actions indicate good understanding of peritoneal dialysis.

The nurse will suspect that the patient with stable angina is experiencing a side effect of the prescribed metoprolol (Lopressor) if a. the patient is restless and agitated. b. the blood pressure is 190/110 mm Hg. c. the patient complains about feeling anxious. d. the cardiac monitor shows a heart rate of 45.

ANS: D Patients taking -blockers should be monitored for bradycardia. Because this category of medication inhibits the sympathetic nervous system, restlessness, agitation, hypertension, and anxiety will not be side effects. DIF: Cognitive Level: Application REF: 776

The nurse obtains the following data when caring for a patient who experienced an acute myocardial infarction (AMI) 2 days previously. Which information is most important to report to the health care provider? a. The patient denies ever having a heart attack. b. The cardiac-specific troponin level is elevated. c. The patient has occasional premature atrial contractions (PACs). d. Crackles are auscultated bilaterally in the mid-lower lobes.

ANS: D The crackles indicate that the patient may be developing heart failure, a possible complication of myocardial infarction (MI). The health care provider may need to order medications such as diuretics or angiotensin-converting enzyme (ACE) inhibitors for the patient. Elevation in cardiac troponin level at this time is expected. PACs are not life-threatening dysrhythmias. Denial is a common response in the immediate period after the MI. DIF: Cognitive Level: Application REF: 779-780

When administering IV nitroglycerin (Tridil) to a patient with a myocardial infarction (MI), which action will the nurse take to evaluate the effectiveness of the medication? a. Check blood pressure. b. Monitor apical pulse rate. c. Monitor for dysrhythmias. d. Ask about chest discomfort.

ANS: D The goal of IV nitroglycerin administration in MI is relief of chest pain by improving the balance between myocardial oxygen supply and demand. The nurse also will monitor heart rate and BP and observe for dysrhythmias, but these parameters will not indicate whether the medication is effective. DIF: Cognitive Level: Application REF: 784-785

After noting lengthening QRS intervals in a patient with acute kidney injury (AKI), which action should the nurse take first? a. Document the QRS interval. b. Notify the patient's health care provider. c. Look at the patient's current blood urea nitrogen (BUN) and creatinine levels. d. Check the chart for the most recent blood potassium level.

ANS: D The increasing QRS interval is suggestive of hyperkalemia, so the nurse should check the most recent potassium and then notify the patient's health care provider. The BUN and creatinine will be elevated in a patient with AKI, but they would not directly affect the electrocardiogram (ECG). Documentation of the QRS interval also is appropriate, but interventions to decrease the potassium level are needed to prevent life-threatening bradycardia.

Which is the best method to document a patient's tobacco use and risk of heart disease? A. Number of years smoked B. Number of cigarettes daily C. Number of forms of tobacco use D. Number of pack-years of tobacco use

ANS: D The most informative method is pack-years, which is the number of packs smoked per day multiplied by the number of years the patient has smoked. Reference: 721

Which electrocardiographic (ECG) change is most important for the nurse to communicate to the health care provider when caring for a patient with chest pain? a. Frequent premature atrial contractions (PACs) b. Inverted P wave c. Sinus tachycardia d. ST segment elevation

ANS: D The patient is likely to be experiencing an ST-segment-elevation myocardial infarction (STEMI) and immediate therapy with percutaneous coronary intervention (PCI) or fibrinolytic medications is indicated to minimize the amount of myocardial damage. The other ECG changes also may suggest a need for therapy, but not as rapidly. DIF: Cognitive Level: Application REF: 780-781

After the nurse teaches a patient with chronic stable angina about how to use the prescribed short-acting and long-acting nitrates, which statement by the patient indicates that the teaching has been effective? a. "I will put on the nitroglycerin patch as soon as I develop any chest pain." b. "I will check the pulse rate in my wrist just before I take any nitroglycerin." c. "I will be sure to remove the nitroglycerin patch before using any sublingual nitroglycerin." d. "I will stop what I am doing and sit down before I put the nitroglycerin under my tongue."

ANS: D The patient should sit down before taking the nitroglycerin to decrease cardiac workload and prevent orthostatic hypotension. Transdermal nitrates are used prophylactically rather than to treat acute pain and can be used concurrently with sublingual nitroglycerin. Although the nurse should check blood pressure before giving nitroglycerin, patients do not need to check the pulse rate before taking nitrates. DIF: Cognitive Level: Application REF: 775-776

A new order for IV gentamicin (Garamycin) 60 mg BID is received for a patient with diabetes who has pneumonia. When evaluating for adverse effects of the medication, the nurse will plan to monitor the patient's a. urine osmolality. b. serum potassium. c. blood glucose level. d. blood urea nitrogen (BUN) and creatinine. .

ANS: D When a patient at risk for chronic kidney disease (CKD) receives a nephrotoxic medication, it is important to monitor renal function with BUN and creatinine levels. The other laboratory values would not be useful in determining the effect of the gentamicin

For which dysrhythmia is defibrillation primarily indicated? A. Ventricular fibrillation B. Third-degree AV block C. Uncontrolled atrial fibrillation D. Ventricular tachycardia with a pulse

ANS:A Defibrillation is always indicated in the treatment of ventricular fibrillation. Drug treatments are normally used in the treatment of uncontrolled atrial fibrillation and for ventricular tachycardia with a pulse (if the patient is stable). Otherwise, synchronized cardioversion is used (as long as the patient has a pulse). Pacemakers are the treatment of choice for third-degree heart block. Reference: 833

What is important to teach a patient taking hydrochlorothiazide (HydroDIURIL) for hypertension? A. Eat bananas and oranges. B. Take the medication at bedtime. C. Notify the doctor if erectile dysfunction occurs. D. Increase consumption of dairy products.

ANS:A Thiazide diuretics can cause hypokalemia, and eating potassium-rich foods should be encouraged. Diuretics should be taken in the morning. Erectile dysfunction is a potential side effect and can lead to noncompliance, but the potassium level is more important. Hypercalcemia is a risk, and increasing dairy is not indicated. Reference: 748

Classic symptoms of restrictive cardiomyopathy include (select all that apply) A. fatigue. B. dyspnea. C. oliguria. D. dizziness. E. exercise intolerance.

ANS:A,B,E Classic manifestations of restrictive cardiomyopathy are fatigue, exercise intolerance, and dyspnea because the heart cannot increase cardiac output by increasing the heart rate without further compromising ventricular filling. Reference: 862

A patient is admitted to the hospital in hypertensive crisis (blood pressure of 243/142 mm Hg). Sodium nitroprusside is started to treat the elevated blood pressure. Which management strategies are appropriate for this patient (select all that apply)? A. Measuring hourly urine output B. Decreasing the mean arterial pressure by 50% within the first hour C. Continuous blood pressure monitoring with an intraarterial line D. Maintaining bed rest and providing tranquilizers to lower the blood pressure E. Assessing the patient for signs and symptoms of heart failure and changes in mental status

ANS:A,C,E Measure urine output hourly to assess renal perfusion. It is recommended that patients being treated with intravenous sodium nitroprusside have continuous intraarterial blood pressure monitoring. Hypertensive crisis can cause encephalopathy, intracranial or subarachnoid hemorrhage, acute left ventricular failure, myocardial infarction, renal failure, dissecting aortic aneurysm, and retinopathy. The initial treatment goal is to decrease mean arterial pressure by no more than 25% within minutes to 1 hour. Patients receiving intravenous antihypertensive drugs may be restricted to bed. Getting up (e.g., to use the toilet) may cause severe cerebral ischemia and fainting. Reference: 757

The nurse identifies which ethnic group as having a greater incidence of dilated cardiomyopathy? A. Native Americans B. African Americans C. Hispanic Americans D. Whites

ANS:B Dilated cardiomyopathy occurs more frequently in middle-aged African Americans and in men. Reference: 86

The telemetry nurse notes eight premature ventricular contractions per minute, often with two occurring together, on the patient's electrocardiograph tracing. Why will lidocaine be administered? A. To limit cardiac tissue remodeling B. To prevent ventricular fibrillation C. To promote cardiac conduction D. To stimulate atrial contractility

ANS:B Premature ventricular contractions may precede ventricular tachycardia and fibrillation. These ventricular dysrhythmias are often lethal and need to be prevented. Ventricular remodeling is normal myocardium compensation after an MI. The normal cardiac tissue hypertrophies and dilates after an MI to compensate for the damaged tissue. Reference: 780

The most common symptom of hypertrophic cardiomyopathy is A. cough. B. dyspnea. C. weight gain. D. palpitations and S3 heart sound.

ANS:B The most common symptom is dyspnea, which is caused by an elevated left ventricular diastolic pressure. Cough does not occur, nor does weight gain. An S4 heart sound may be heard. Reference: 862

You are admitting a patient who is scheduled to undergo a cardiac catheterization. Which allergies are most important for you to assess before this procedure? A. Iron B. Iodine C. Aspirin D. Penicillin

ANS:B The physician typically uses iodine-based contrast to perform this procedure. You must assess whether the patient is allergic to iodine or shellfish. Reference: 721, 732

Heart transplant recipients are at risk for which complications in the first year after transplantation (select all that apply)? A. Cancer B. Infection C. Rejection D. Vasculopathy E. Sudden cardiac death

ANS:B,C,E A variety of complications can occur after heart transplantation. In the first year after transplantation, the major causes of death are acute rejection and infection. There is also a risk of sudden cardiac death. Later, malignancy (especially lymphoma) and cardiac vasculopathy (accelerated coronary artery disease) are major causes of death. Reference: 815

You are caring for a patient newly admitted with heart failure due to dilated cardiomyopathy. Which intervention is a priority? A. Encourage caregivers to learn cardiopulmonary resuscitation (CPR). B. Consider a consultation with hospice for palliative care. C. Monitor the patient's response to prescribed medications. D. Arrange for the patient to enter a cardiac rehabilitation program.

ANS:C Observing for signs and symptoms of worsening heart failure, dysrhythmias, and embolic formation is paramount for patients with dilated cardiomyopathy, as is monitoring drug responsiveness. The goal of therapy is to keep the patient at an optimal level of functioning and out of the hospital. The priority intervention is to manage the acute symptoms with medications. Caregivers should learn CPR before hospital discharge, and the patient may be referred to cardiac rehabilitation. Patients with dilated cardiomyopathy with progression to class IV, stage D heart failure are candidates for palliative care. Reference: 862

Which nursing action should you prioritize during the care of a patient who has recently recovered from rheumatic fever? A. Teach the patient how to manage his or her physical activity. B. Teach the patient about the need for ongoing anticoagulation. C. Teach the patient about his or her need for continuous antibiotic prophylaxis. D. Teach the patient about the need to maintain standard infection control procedures.

ANS:C Patients with a history of rheumatic fever frequently require ongoing antibiotic prophylaxis, an intervention that necessitates education. This consideration is more important than activity management in preventing recurrence. Anticoagulation and standard precautions are not indicated for this patient population. Reference: 851

The leading cause of death of patients with idiopathic dilated cardiomyopathy is A. shock. B. heart failure. C. ventricular dysrhythmias. D. disseminated intravascular coagulation (DIC).

ANS:C Sudden cardiac death from ventricular dysrhythmias is a leading cause of death of patients with idiopathic dilated cardiomyopathy. Shock is not associated with dilated cardiomyopathy. Heart failure may occur, but it is not the leading cause of death. DIC is not associated with dilated cardiomyopathy. Reference: 860

Which clinical findings most likely indicate decreased cardiac output in a patient with aortic valve regurgitation? A. Reduction in peripheral edema and weight B. Carotid venous distention and new-onset atrial fibrillation C. Significant pulsus paradoxus and diminished peripheral pulses D. Shortness of breath on minimal exertion and a diastolic murmur

ANS:D Clinical manifestations of chronic aortic regurgitation that indicate decreased cardiac output include severe dyspnea, chest pain, and hypotension. Other manifestations include water-hammer pulse (a strong, quick beat that collapses immediately), soft or absent S1, presence of S3 or S4, and soft, high-pitched diastolic murmur. A low-pitched diastolic murmur may be heard in severe aortic regurgitation. Early manifestations may include exertional dyspnea, orthopnea, and paroxysmal nocturnal dyspnea. Reference: 854

Residual and chronic cardiac damage from acute rheumatic fever most often includes A myocardial scarring. B. chronic constrictive pericarditis. C. blockade of the conduction system. D. scarring and contractures of the mitral valve.

ANS:D Rheumatic endocarditis can result in fibrous tissue growth in valve leaflets and chordae tendineae, with scarring and contractures. The mitral valve is most frequently involved, but the aortic and tricuspid valves also may be affected. Reference: 851

You are alert for which dysrhythmia in the patient with mitral stenosis? A. Ventricular tachycardia B. Ventricular fibrillation C. Atrial tachycardia D. Atrial fibrillation

ANS:D The primary symptom of mitral stenosis is exertional dyspnea due to reduced lung compliance. Fatigue and palpitations from atrial fibrillation may also occur. Ventricular dysrhythmias are not associated with mitral stenosis. Tachycardia affects the whole heart, not just the atria. Reference: 853

31. Which drugs are used to treat overflow incontinence (select all that apply)? a. Baclofen (Lioresal) b. Anticholinergic drugs c. α-Adrenergic blockers d. 5α-reductase inhibitors e. Bethanechol (Urecholine)

c, d, e. α-Adrenergic blockers block the stimulation of the smooth muscle of the bladder, 5α-reductase inhibitors decrease outlet resistance, and bethanechol enhances bladder contractions. Baclofen or diazepam is used to relax the external sphincter for reflex incontinence. Anticholinergics are used to relax bladder tone and increase sphincter tone with urge incontinence.

The patient with chronic kidney disease is considering whether to use peritoneal dialysis (PD) or hemodialysis (HD). What are advantages of PD when compared to HD (select all that apply)? a. Less protein loss b. Rapid fluid removal c. Less cardiovascular stress d. Decreased hyperlipidemia e. Requires fewer dietary restrictions

c, e. Peritoneal dialysis is less stressful for the cardiovascular system and requires fewer dietary restrictions. Peritoneal dialysis actually contributes to more protein loss and increased hyperlipidemia. The fluid and creatinine removal are slower with peritoneal dialysis than hemodialysis.

A patient has a junctional escape rhythm on the monitor. The nurse will expect the patientto have a heart rate of how many beats/minute? a. 15 to 20 b. 20 to 40 c. 40 to 60 d. 60 to 100

c. 40 to 60 If the sinoatrial (SA) node fails to discharge, the atrioventricular (AV) node willautomatically discharge at the normal rate of 40 to 60. The slower rates are typical of the bundle of His and the Purkinje system and may be seen with failure of both the SA andAV node to discharge. The normal SA node rate is 60 to 100 beats/minute

On assessment of the patient with a renal calculus passing down the ureter, what should the nurse expect the patient to report? a. A history of chronic UTIs b. Dull, costovertebral flank pain c. Severe, colicky back pain radiating to the groin d. A feeling of bladder fullness with urgency and frequency

c. A classic sign of the passage of a calculus down the ureter is intense, colicky back pain that may radiate into the testicles, labia, or groin and may be accompanied by mild shock with cool, moist skin. Many patients with renal stones do not have a history of chronic UTIs. Stones obstructing a calyx or at the ureteropelvic junction may produce dull costovertebral flank pain and large bladder stones may cause bladder fullness and lower obstructive symptoms.

What is included in nursing care that applies to the management of all urinary catheters in hospitalized patients? a. Measuring urine output every 1 to 2 hours to ensure patency b. Turning the patient frequently from side to side to promote drainage c. Using strict sterile technique during irrigation and obtaining culture specimens d. Daily cleaning of the catheter insertion site with soap and water and application of lotion

c. All urinary catheters in hospitalized patients pose a very high risk for infection, especially antibiotic-resistant, health care-associated infections, and scrupulous aseptic technique is essential in the insertion and maintenance of all catheters. Routine irrigations are not performed. Turning the patient to promote drainage is recommended only for suprapubic catheters. Cleaning the insertion site with soap and water should be performed for urethral and suprapubic catheters but lotion or powder should be avoided and site care for other catheters may require special interventions.

What is the most common cause of acute pyelonephritis resulting from an ascending infection from the lower urinary tract? a. The kidney is scarred and fibrotic. b. The organism is resistant to antibiotics. c. There is a preexisting abnormality of the urinary tract. d. The patient does not take all of the antibiotics for treatment of a UTI.

c. Ascending infections from the bladder to the kidney are prevented by the normal anatomy and physiology of the urinary tract unless a preexisting condition, such as vesicoureteral reflux or lower urinary tract dysfunction (bladder tumors, prostatic hyperplasia, strictures, or stones), is present. Resistance to antibiotics and failure to take a full prescription of antibiotics for a UTI usually result in relapse or reinfection of the lower urinary tract.

What can patients at risk for renal lithiasis do to prevent the stones in many cases? a. Lead an active lifestyle b. Limit protein and acidic foods in the diet c. Drink enough fluids to produce dilute urine d. Take prophylactic antibiotics to control UTIs

c. Because crystallization of stone constituents can precipitate and unite to form a stone when in supersaturated concentrations, one of the best ways to prevent stones of any type is by drinking adequate fluids to keep the urine dilute and flowing (e.g., an output of about 2 L of urine a day). Sedentary lifestyle is a risk factor for renal stones but exercise also causes fluid loss and a need for additional fluids. Protein foods high in purine should be restricted only for the small percentage of patients with uric acid stones and although UTIs contribute to stone formation, prophylactic antibiotics are not indicated.

Which type of urinary tract calculi are the most common and frequently obstruct the ureter? a. Cystine b. Uric acid c. Calcium oxalate d. Calcium phosphate

c. Calcium oxalate calculi are most common and small enough to get trapped in the ureter.

A patient with AKI is a candidate for continuous renal replacement therapy (CRRT). What is the most common indication for use of CRRT? a. Azotemia b. Pericarditis c. Fluid overload d. Hyperkalemia

c. Continuous renal replacement therapy (CRRT) is indicated for the patient with AKI as an alternative or adjunct to hemodialysis to slowly remove solutes and fluid in the hemodynamically unstable patient. It is especially useful for treatment of fluid overload, but hemodialysis is indicated for treatment of hyperkalemia, pericarditis, or other serious effects of uremia.

What does the dialysate for PD routinely contain? a. Calcium in a lower concentration than in the blood b. Sodium in a higher concentration than in the blood c. Dextrose in a higher concentration than in the blood d. Electrolytes in an equal concentration to that of the blood

c. Dextrose or icodextrin or amino acid is added to dialysate fluid to create an osmotic gradient across the membrane to remove excess fluid from the blood. The dialysate fluid has no potassium so that potassium will diffuse into the dialysate from the blood. Dialysate also usually contains higher calcium to promote its movement into the blood. Dialysate sodium is usually less than or equal to that of blood to prevent sodium and fluid retention.

Glomerulonephritis is characterized by glomerular damage caused by a. growth of microorganisms in the glomeruli. b. release of bacterial substances toxic to the glomeruli. c. accumulation of immune complexes in the glomeruli. d. hemolysis of red blood cells circulating in the glomeruli.

c. Glomerulonephritis is not an infection but rather an antibody-induced injury to the glomerulus, where either autoantibodies against the glomerular basement membrane (GBM) directly damage the tissue or antibodies reacting with nonglomerular antigens are randomly deposited as immune complexes along the GBM. Prior infection by bacteria or viruses may stimulate the antibody production but is not present or active at the time of glomerular damage.

The patient with CKD is brought to the emergency department with Kussmaul respirations. What does the nurse know about CKD that could cause this patient's Kussmaul respirations? a. Uremic pleuritis is occurring. b. There is decreased pulmonary macrophage activity. c. They are caused by respiratory compensation for metabolic acidosis. d. Pulmonary edema from heart failure and fluid overload is occurring.

c. Kussmaul respirations occur with severe metabolic acidosis when the respiratory system is attempting to compensate by removing carbon dioxide with exhalations. Uremic pleuritis would cause a pleural friction rub. Decreased pulmonary macrophage activity increases the risk of pulmonary infection. Dyspnea would occur with pulmonary edema.

To assist the patient with stress incontinence, what is the best thing the nurse should teach the patient to do? a. Void every 2 hours to prevent leakage. b. Use absorptive perineal pads to contain urine. c. Perform pelvic floor muscle exercises 40 to 50 times per day. d. Increase intraabdominal pressure during voiding to empty the bladder completely.

c. Pelvic floor exercises (Kegel exercises) increase the tone of the urethral sphincters and should be done in sets of 10 or more contractions four to five times a day (total of 40 to 50 per day). Frequent bladder emptying is recommended for patients with urge incontinence and an increase in pressure on the bladder is recommended for patients with overflow incontinence. Absorptive perineal pads should be only a temporary measure because longterm use discourages continence and can lead to skin problems.

What results in the edema associated with nephrotic syndrome? a. Hypercoagulability b. Hyperalbuminemia c. Decreased plasma oncotic pressure d. Decreased glomerular filtration rate

c. The massive proteinuria that results from increased glomerular membrane permeability in nephrotic syndrome leaves the blood without adequate proteins (hypoalbuminemia) to create an oncotic colloidal pressure to hold fluid in the vessels. Without oncotic pressure, fluid moves into the interstitial, causing severe edema. Hypercoagulability occurs in nephrotic syndrome but is not a factor in edema formation and glomerular filtration rate (GFR) is not necessarily affected in nephrotic syndrome.

During the nursing assessment of the patient with renal insufficiency, the nurse asks the patient specifically about a history of a. angina. b. asthma. c. hypertension. d. rheumatoid arthritis.

c. The most common causes of CKD in the United States are diabetes mellitus and hypertension. The nurse should obtain information on long-term health problems that are related to kidney disease. The other disorders are not closely associated with renal disease.

While caring for a 77-year-old woman who has a urinary catheter, the nurse monitors the patient for the development of a UTI. What clinical manifestations is the patient most likely to experience? a. Cloudy urine and fever b. Urethral burning and bloody urine c. Vague abdominal discomfort and disorientation d. Suprapubic pain and slight decline in body temperature

c. The usual classic manifestations of UTI are often absent in older adults, who tend to experience nonlocalized abdominal discomfort and cognitive impairment characterized by confusion or decreased level of consciousness rather than dysuria and suprapubic pain.

Thirty percent of patients with kidney cancer have metastasis at the time of diagnosis. Why does this occur? a. The only treatment modalities for the disease are palliative. b. Diagnostic tests are not available to detect tumors before they metastasize. c. Classic symptoms of hematuria and palpable mass do not occur until the disease is advanced. d. Early metastasis to the brain impairs the patient's ability to recognize the seriousness of symptoms.

c. There are no early characteristic symptoms of cancer of the kidney and gross hematuria, flank pain, and a palpable mass do not occur until the disease is advanced. The treatment of choice is a partial or radical nephrectomy, which can be successful in early disease. Many kidney cancers are diagnosed as incidental imaging findings. Targeted therapy is the preferred treatment for metastatic kidney cancer. Radiation is palliative. The most common sites of metastases are the lungs, liver, and long bones.

A woman with no history of UTIs who is experiencing urgency, frequency, and dysuria comes to the clinic, where a dipstick and microscopic urinalysis indicate bacteriuria. What should the nurse anticipate for this patient? a. Obtaining a clean-catch midstream urine specimen for culture and sensitivity b. No treatment with medication unless she develops fever, chills, and flank pain c. Empirical treatment with trimethoprim-sulfamethoxazole (TMP-SMX, Bactrim) for 3 days d. Need to have a blood specimen drawn for a complete blood count (CBC) and kidney function tests

c. Unless a patient has a history of recurrent UTIs or a complicated UTI, trimethoprim-sulfamethoxazole (TMPSMX) or nitrofurantoin (Macrodantin) is usually used to empirically treat an initial UTI without a culture and sensitivity or other testing. Asymptomatic bacteriuria does not justify treatment but symptomatic UTIs should always be treated.

What causes the gastrointestinal (GI) manifestation of stomatitis in the patient with CKD? a. High serum sodium levels b. Irritation of the GI tract from creatinine c. Increased ammonia from bacterial breakdown of urea d. Iron salts, calcium-containing phosphate binders, and limited fluid intake

c. Uremic fetor, or the urine odor of the breath, is caused by high urea content in the blood. Increased ammonia from bacterial breakdown of urea leads to stomatitis and mucosal ulcerations. Irritation of the gastrointestinal (GI) tract from urea in CKD contributes to anorexia, nausea, and vomiting. Ingestion of iron salts and calcium-containing phosphate binders, limited fluid intake, and limited activity cause constipation.

A patient experiences dizziness and shortness of breath for several days. During cardiacmonitoring in the emergency department (ED), the nurse obtains the followingelectrocardiographic (ECG) tracing. <<see image>> 6 irregular P waves 2 irregular QRS The nurse interprets this cardiac rhythm as a. sinus rhythm with premature ventricular contractions (PVCs). b. junctional escape rhythm. c. third-degree atrioventricular (AV) block. d. sinus rhythm with premature atrial contractions (PACs).

c. third-degree atrioventricular (AV) block. The inconsistency between the atrial and ventricular rates and the variable P-R intervalindicate that the rhythm is third-degree AV block. Sinus rhythm with PACs or PVCs willhave a normal rate and consistent P-R intervals with occasional PACs or PVCs. A junctional escape rhythm will not have P waves.

A coronary care unit (CCU) nurse is caring for a client admitted with acute myocardial infarction (MI). The nurse monitors for which most common complication of MI?

cardiac dysrhythmias

A patient who has a diagnosis of atrial fibrillation as a heart rate of 152 beats per minute. The healthcare provider should assess for which of these problems related to the dysrhythmia? Chest pain Hypotension Headache Pulse deficit Dizziness

chest pain, hypotension, pulse deficit, dizziness Atrial kick is lost and cardiac output is diminished. The result is less blood supplying the brain (dizziness), hypotension, less blood perfusing the coronary arteries, and a pulse deficit because the stroke volume is not sufficient to produce a palpable peripheral pulse.

What are intrarenal causes of acute kidney injury (AKI) (select all that apply)? a. Anaphylaxis b. Renal stones c. Bladder cancer d. Nephrotoxic drugs e. Acute glomerulonephritis f. Tubular obstruction by myoglobin

d, e, f. Intrarenal causes of acute kidney injury (AKI) include conditions that cause direct damage to the kidney tissue, including nephrotoxic drugs, acute glomerulonephritis, and tubular obstruction by myoglobin, or prolonged ischemia. Anaphylaxis and other prerenal problems are frequently the initial cause of AKI. Renal stones and bladder cancer are among the postrenal causes of AKI.

The nurse has received change-of-shift report about the following patients on thetelemetry unit. Which patient should the nurse see first? a. A patient with atrial fibrillation, rate 88, who has a new order for warfarin(Coumadin) b. A patient with type 1 second-degree atrioventricular (AV) block, rate 60, who isdizzy when ambulating c. A patient who is in a sinus rhythm, rate 98, after having electrical cardioversion 2hours ago d. A patient whose implantable cardioverter-defibrillator (ICD) fired three timestoday who has a dose of amiodarone (Cordarone) due

d. A patient whose implantable cardioverter-defibrillator (ICD) fired three timestoday who has a dose of amiodarone (Cordarone) due The frequent firing of the ICD indicates that the patient's ventricles are very irritable, andthe priority is to assess the patient and administer the amiodarone. The other patients may be seen after the amiodarone is administered.

Which test is required for a diagnosis of pyelonephritis? a. Renal biopsy b. Blood culture c. Intravenous pyelogram (IVP) d. Urine for culture and sensitivity

d. A urine specimen specifically obtained for culture and sensitivity is required to diagnose pyelonephritis because it will show pyuria, the specific bacteriuria, and what drug the bacteria is sensitive to for treatment. The renal biopsy is used to diagnose chronic pyelonephritis or cancer. Blood cultures would be done if bacteremia is suspected. Intravenous pyelogram (IVP) would increase renal irritation, but CT urograms may be used to assess for signs of infection in the kidney and complications of pyelonephritis.

What manifestation in the patient will indicate the need for restriction of dietary protein in management of acute post streptococcal glomerulonephritis (APSGN)? a. Hematuria b. Proteinuria c. Hypertension d. Elevated blood urea nitrogen (BUN)

d. An elevated blood urea nitrogen (BUN) indicates that the kidneys are not clearing nitrogenous wastes from the blood and protein may be restricted until the kidney recovers. Proteinuria indicates loss of protein from the blood and possibly a need for increased protein intake. Hypertension is treated with sodium and fluid restriction, diuretics, and antihypertensive drugs. The hematuria is not specifically treated.

Which serum laboratory value indicates to the nurse that the patient's CKD is getting worse? a. Decreased BUN b. Decreased sodium c. Decreased creatinine d. Decreased calculated glomerular filtration rate (GFR)

d. As GFR decreases, BUN and serum creatinine levels increase. Although elevated BUN and creatinine indicate that waste products are accumulating, the calculated GFR is considered a more accurate indicator of kidney function than BUN or serum creatinine.

A patient rapidly progressing toward end-stage kidney disease asks about the possibility of a kidney transplant. In responding to the patient, the nurse knows that what is a contraindication to kidney transplantation? a. Hepatitis C infection b. Coronary artery disease c. Refractory hypertension d. Extensive vascular disease

d. Extensive vascular disease is a contraindication for renal transplantation, primarily because adequate blood supply is essential for the health of the new kidney. Other contraindications include disseminated malignancies, refractory or untreated cardiac disease, chronic respiratory failure, chronic infection, or unresolved psychosocial disorders. Coronary artery disease (CAD) may be treated with bypass surgery before transplantation and transplantation can relieve hypertension. Hepatitis B or C infection is not a contraindication.

While caring for the patient in the oliguric phase of AKI, the nurse monitors the patient for associated collaborative problems. When should the nurse notify the health care provider? a. Urine output is 300 mL/day. b. Edema occurs in the feet, legs, and sacral area. c. Cardiac monitor reveals a depressed T wave and elevated ST segment. d. The patient experiences increasing muscle weakness and abdominal cramping.

d. Hyperkalemia is a potentially life-threatening complication of AKI in the oliguric phase. Muscle weakness and abdominal cramping are signs of the neuromuscular impairment that occurs with hyperkalemia. In addition, hyperkalemia can cause the cardiac conduction abnormalities of peaked T wave, prolonged PR interval, prolonged QRS interval, and depressed ST segment. Urine output of 300 mL/day is expected during the oliguric phase, as is the development of peripheral edema.

What indicates to the nurse that a patient with oliguria has prerenal oliguria? a. Urine testing reveals a low specific gravity. b. Causative factor is malignant hypertension. c. Urine testing reveals a high sodium concentration. d. Reversal of oliguria occurs with fluid replacement.

d. In prerenal oliguria, the oliguria is caused by a decrease in circulating blood volume and there is no damage yet to the renal tissue. It can be reversed by correcting the precipitating factor, such as fluid replacement for hypovolemia. Prerenal oliguria is characterized by urine with a high specific gravity and a low sodium concentration, whereas oliguria of intrarenal failure is characterized by urine with a low specific gravity and a high sodium centration. Malignant hypertension causes damage to renal tissue and intrarenal oliguria.

What accurately describes the care of the patient with CKD? a. A nutrient that is commonly supplemented for the patient on dialysis because it is dialyzable is iron. b. The syndrome that includes all of the signs and symptoms seen in the various body systems in CKD is azotemia. c. The use of morphine is contraindicated in the patient with CKD because accumulation of its metabolites may cause seizures. d. The use of calcium-based phosphate binders in the patient with CKD is contraindicated when serum calcium levels are increased.

d. In the patient with CKD, when serum calcium levels are increased, calcium-based phosphate binders are not used. The nutrient supplemented for patients on dialysis is folic acid. The various body system manifestations occur with uremia, which includes azotemia. Meperidine is contraindicated in patients with CKD related to possible seizures.

In caring for the patient with AKI, what should the nurse be aware of? a. The most common cause of death in AKI is irreversible metabolic acidosis. b. During the oliguric phase of AKI, daily fluid intake is limited to 1000 mL plus the prior day's measured fluid loss. c. Dietary sodium and potassium during the oliguric phase of AKI are managed according to the patient's urinary output. d. One of the most important nursing measures in managing fluid balance in the patient with AKI is taking accurate daily weights.

d. Measuring daily weights with the same scale at the same time each day allows for the evaluation and detection of excessive body fluid gains or losses. Infection is the leading cause of death in AKI, so meticulous aseptic technique is critical. The fluid limitation in the oliguric phase is 600 mL plus the prior day's measured fluid loss. Dietary sodium and potassium intake are managed according to the plasma levels.

Prevention of calcium oxalate stones would include dietary restriction of which foods or drinks? a. Milk and milk products b. Dried beans and dried fruits c. Liver, kidney, and sweetbreads d. Spinach, cabbage, and tomatoes

d. Oxalate-rich foods should be limited to reduce oxalate excretion. Foods high in oxalate include spinach, rhubarb, asparagus, cabbage, and tomatoes, in addition to chocolate, coffee, and cocoa. Currently, it is believed that high dietary calcium intake may actually lower the risk for renal stones by reducing the intestinal oxalate absorption and therefore the urinary excretion of oxalate. Milk, milk products, dried beans, and dried fruits are high sources of calcium. Organ meats are high in purine, which contributes to uric acid lithiasis.

A patient with bladder cancer undergoes cystectomy with formation of an ileal conduit. During the patient's first postoperative day, what should the nurse plan to do? a. Measure and fit the stoma for a permanent appliance. b. Encourage high oral intake to flush mucus from the conduit. c. Teach the patient to self-catheterize the stoma every 4 to 6 hours. d. Empty the drainage bag every 2 to 3 hours and measure the urinary output.

d. Urine drains continuously from an ileal conduit and the drainage bag must be emptied every 2 to 3 hours and measured to ensure adequate urinary output. Fitting for a permanent appliance is not done until the stoma shrinks to its normal size in a few weeks. With an ileal conduit, mucus is present in the urine because it is secreted by the ileal segment as a result of the irritating effect of the urine but the surgery causes paralytic ileum and the patient will be NPO for several days postoperatively. Self-catheterization is performed when patients have formation of a continent Kock pouch.

A hospitalized patient with a history of chronic stable angina tells the nurse that she is having chest pain. The nurse bases his actions on the knowledge that ischemia a. will always progress to myocardial infarction b. will be relieved by rest, nitroglycerin, or both c. indicates that irreversible myocardial damage is occurring d. is frequently associated with vomiting and extreme fatigue

B

After teaching a patient with chronic stable angina about nitroglycerin, the nurse recognizes the need for further teaching when the patient states, A) a. "I will replace my nitroglycerin supply every 6 months." B) b. "I can take up to five tablets every 3 minutes for relief of my chest pain." C) c. "I will take acetaminophen (Tylenol) to treat the headache caused by nitroglycerin." D) d. "I will take the nitroglycerin 10 minutes before planned activity that usually causes chest pain."

B

Auscultation of a patient's heart reveals the presence of a murmur. This assessment finding is a result of A) a. Increased viscosity of the patient's blood. B) b. Turbulent blood flow across a heart valve. C) c. Friction between the heart and the myocardium. D) d. A deficit in heart conductivity that impairs normal contractility.

B

The nurse is caring for a patient who is two days post-MI. The patient reports that she is experiencing chest pain. She states "it hurts when I take a deep breath." Which of the following actions would be a priority? a. Notify the physician STAT and obtain a 12-lead ECG b. obtain vital signs and auscultate for a pericardial friction rub c. Apply high-flow oxygen by face mask and auscultate breath sounds d. Medicate the patient with PRN analgesic and reevaluate in 30 minutes

B

The nurse is providing teaching to a patient recovering from an MI. Discussion regarding resumption of sexual activity should be A) a. Delegated to the primary care provider. B) b. Discussed along with other physical activities. C) c. Avoided because it is embarrassing to the patient. D) d. Accomplished by providing the patient with written material.

B

The nurse is watching the cardiac monitor, and a patient's rhythm suddenly changes. There are no P waves. Instead there are fine, wavy lines between the QRS complexes. The QRS complexes measure 0.08 sec (narrow), but they occur irregularly with a rate of 120 beats/min. The nurse correctly interprets that this rhythm is which of the following? A) a. Sinus tachycardia B) b. Atrial fibrillation C) c. Ventricular fibrillation D) d. Ventricular tachycardia

B

The portion of the vascular system responsible for hemostasis is the a. thin capillary vessels b. endothelial layer of the arteries c. elastic middle layer of the veins d. smooth muscle of the arterial wall

B

While obtaining subjective assessment data from a patient with hypertension, the nurse recognizes that a modifiable risk factor for the development of hypertension is a. a low-calcium diet b. excessive alcohol consumption c. a family history of hypertension d. consumption of a high-protein diet

B

A patient with a history of end-stage renal disease secondary to diabetes mellitus has presented to the outpatient dialysis unit for his scheduled hemodialysis. Which of the following assessments should the nurse prioritize before, during, and after his treatment? A Level of consciousness B Blood pressure and fluid balance C Temperature, heart rate, and blood pressure D Assessment for signs and symptoms of infection

B Although all of the assessments are relevant to the care of a patient receiving hemodialysis, the nature of procedure indicates a particular need to monitor patients' blood pressure and fluid balance.

The nurse is caring for a patient admitted with a history of hypertension. The patient's medication history includes hydrochlorothiazide (Hydrodiuril) daily for the past 10 years. Which of the following parameters would indicate the optimal intended effect of this drug therapy? A) a. Weight loss of 2 lb B) b. Blood pressure 128/86 C) c. Absence of ankle edema D) d. Output of 600 ml per shift

B Hydrochlorothiazide may be used alone as monotherapy to manage hypertension or in combination with other medications if not effective alone. After the first few weeks of therapy, the diuretic effect diminishes, but the antihypertensive effect remains. Since the patient has been taking this medication for 10 years, the most direct measurement of its intended effect would be the blood pressure.

*7. The nurse recommends genetic counseling for the children of a patient with* a.nephrotic syndrome b.chronic pyelonephritis c. malignant nephrosclerosis d.adult onset polycystic kidney disease

d.adult onset polycystic kidney disease

*5.The edema that occurs in nephrotic syndrome is due to* a. increased hydrostatic pressure caused by sodium retention. b. decreased aldosterone secretion from adrenal insufficiency. c. increased fluid retention caused by decreased glomerular filtration d.decreased colloidal osmotic pressure caused by loss of serum albumin

d.decreased colloidal osmotic pressure caused by loss of serum albumin

Which of the following nursing interventions is appropriate in providing care for an adult patient with newly diagnosed adult onset polycystic kidney disease (PKD)? A Help the patient cope with the rapid progression of the disease. B Suggest genetic counseling resources for the children of the patient. C Expect the patient to have polyuria and poor concentration ability of the kidneys. D Implement appropriate measures for the patient's deafness and blindness in addition to the renal problems.

B PKD is one of the most common genetic diseases. The adult form of PKD may range from a relatively mild disease to one that progresses to chronic kidney disease. Polyuria, deafness, and blindness are not associated with PKD.

A 70-year-old male patient has sought care because of recent difficulties in establishing and maintaining a urine stream as well as pain that occasionally accompanies urination. The nurse would document which of the following abnormal assessment findings? A Anuria B Dysuria C Oliguria D Enuresis

B Painful and difficult urination is characterized as dysuria. Anuria is an absence of urine production, whereas oliguria is diminished urine production. Enuresis is involuntary nocturnal urination.

When providing dietary instruction to a patient with hypertension, the nurse would advise the patient to restrict intake of which of the following meats? A) a. Broiled fish B) b. Roasted duck C) c. Roasted turkey D) d. Baked chicken breast

B Roasted duck is high in fat, which should be avoided by the patient with hypertension. The other meats are lower in fat and are therefore acceptable in the diet.

The nurse is examining the ECG of a patient who has just been admitted with a suspected MI. Which of the following ECG changes is most indicative of prolonged or complete coronary occlusion? A) a. Sinus tachycardia B) b. Pathologic Q wave C) c. Fibrillatory P waves D) d. Prolonged PR interval

B The presence of a pathologic Q wave, as often accompanies STEMI, is indicative of complete coronary occlusion. Sinus tachycardia, fibrillatory P waves (e.g., atrial fibrillation), or a prolonged PR interval (first-degree heart block) are not direct indicators of extensive occlusion.

The nurse obtains a 6-second rhythm strip and charts the following analysis: atrial rate 70, regular; ventricular rate 40, regular; QRS 0.04 sec; no relationship between P waves and QRS complexes; atria and ventricles beating independently of each other. Which of the following would be a correct interpretation of this rhythm strip? A) a. Sinus dysrhythmias B) b. Third-degree heart block C) c. Wenckebach phenomenon D) d. Premature ventricular contractions

B Third-degree heart block represents a loss of communication between the atrium and ventricles. This is depicted on the rhythm strip as no relationship between the P waves, representing atrial contraction, and QRS complexes, representing ventricular contraction. The atrium are beating totally on their own at 70 beats/min, whereas the ventricles are pacing themselves at 40 beats/min.

As a component of the head-to-toe assessment of a patient who has been recently transferred, the nurse is preparing to palpate the patient's kidneys. The nurse should position the patient: A Prone. B Supine. C Seated at the edge of the bed. D Standing, facing away from the nurse.

B To palpate the right kidney, the patient is positioned supine and the nurse's left hand is placed behind and supports the patient's right side between the rib cage and the iliac crest. The right flank is elevated with the left hand, and the right hand is used to palpate deeply for the right kidney.

A 59-year-old man has presented to the emergency department with chest pain. Which of the following components of his subsequent blood work is most clearly indicative of a myocardial infarction (MI)? A) a. CK-MB B) b. Troponin C) c. Myoglobin D) d. C-reactive protein

B Troponin is the biomarker of choice in the diagnosis of MI, with sensitivity and specificity that exceed those of CK-MB and myoglobin. CRP levels are not used to diagnose acute MI.

14. The nurse identifies the collaborative problem of potential complication: pulmonary edema for a patient in ADHF. When assessing the patient, the nurse will be most concerned about a. an apical pulse rate of 106 beats/min. b. an oxygen saturation of 88% on room air. c. weight gain of 1 kg (2.2 lb) over 24 hours. d. decreased hourly patient urinary output.

B Rationale: A decrease in oxygen saturation to less than 92% indicates hypoxemia. The nurse should administer supplemental oxygen immediately to the patient. An increase in apical pulse rate, 1-kg weight gain, and decreases in urine output also indicate worsening heart failure and require rapid nursing actions, but the low oxygen saturation rate requires the most immediate nursing action. Cognitive Level: Analysis Text Reference: pp. 829-830 Nursing Process: Assessment NCLEX: Physiological Integrity

22. A patient who is receiving dobutamine (Dobutrex) for the treatment of ADHF has all of the following nursing actions included in the plan of care. Which action will be best for the RN to delegate to an experienced LPN/LVN? a. Teach the patient the reasons for remaining on bed rest. b. Monitor the patient's BP every hour. c. Adjust the drip rate to keep the systolic BP >90 mm Hg. d. Call the health care provider about a decrease in urine output.

B Rationale: An experienced LPN/LVN would be able to monitor BP and would know to report significant changes to the RN. Teaching patients and making adjustments to the drip rate for vasoactive medications are RN-level skills. Because the health care provider may order changes in therapy based on the decrease in urine output, the RN should call the health care provider about the decreased urine output. Cognitive Level: Application Text Reference: pp. 827-829 Nursing Process: Planning NCLEX: Safe and Effective Care Environment

5. When the nurse is developing a teaching plan to prevent the development of heart failure in a patient with stage 1 hypertension, the information that is most likely to improve compliance with antihypertensive therapy is that a. hypertensive crisis may lead to development of acute heart failure in some patients. b. hypertension eventually will lead to heart failure by overworking the heart muscle. c. high BP increases risk for rheumatic heart disease. d. high systemic pressure precipitates papillary muscle rupture.

B Rationale: Hypertension is a primary cause of heart failure because the increase in ventricular afterload leads to ventricular hypertrophy and dilation. Hypertensive crisis may precipitate acute heart failure is some patients, but this patient with stage 1 hypertension may not be concerned about a crisis that happens only to some patients. Hypertension does not directly cause rheumatic heart disease (which is precipitated by infection with group A -hemolytic streptococcus) or papillary muscle rupture (which is caused by myocardial infarction/necrosis of the papillary muscle). Cognitive Level: Application Text Reference: p. 822 Nursing Process: Planning NCLEX: Health Promotion and Maintenance

8. A patient admitted to the hospital with an exacerbation of chronic heart failure tells the nurse, "I felt fine when I went to bed, but I woke up in the middle of the night feeling like I was suffocating!" The nurse can best document this assessment information as a. pulsus alternans. b. paroxysmal nocturnal dyspnea. c. two-pillow orthopnea. d. acute bilateral pleural effusion.

B Rationale: Paroxysmal nocturnal dyspnea is caused by the reabsorption of fluid from dependent body areas when the patient is sleeping and is characterized by waking up suddenly with the feeling of suffocation. Pulsus alternans is the alternation of strong and weak peripheral pulses during palpation. Orthopnea indicates that the patient is unable to lie flat because of dyspnea. Pleural effusions develop over a longer time period. Cognitive Level: Comprehension Text Reference: p. 825 Nursing Process: Assessment NCLEX: Physiological Integrity

11. When developing a plan to decrease preload in the patient with heart failure, the nurse will include actions such as a. administering sedatives to promote rest and decrease myocardial oxygen demand. b. positioning the patient in a high-Fowler's position with the feet horizontal in the bed. c. administering oxygen per mask or nasal cannula. d. encouraging leg exercises to improve venous return.

B Rationale: Positioning the patient in a high-Fowler's position with the legs dependent will reduce preload by decreasing venous return to the right atrium. The other interventions may also be appropriate for patients with heart failure but will not help in decreasing preload. Cognitive Level: Application Text Reference: pp. 827-828 Nursing Process: Planning NCLEX: Physiological Integrity

7. Intravenous sodium nitroprusside (Nipride) is ordered for a patient with acute pulmonary edema. During the first hours of administration, the nurse will need to adjust the Nipride rate if the patient develops a. a drop in heart rate to 54 beats/min. b. a systolic BP <90 mm Hg. c. any symptoms indicating cyanide toxicity. d. an increased amount of ventricular ectopy.

B Rationale: Sodium nitroprusside is a potent vasodilator, and the major adverse effect is severe hypotension. After 48 hours of continuous use, cyanide toxicity is a possible (though rare) adverse effect. Reflex tachycardia (not bradycardia) is another adverse effect of this medication. Nitroprusside does not cause increased ventricular ectopy. Cognitive Level: Application Text Reference: p. 828 Nursing Process: Evaluation NCLEX: Physiological Integrity

10. The nurse working in the heart failure clinic will know that teaching for a 74-year-old patient with newly diagnosed heart failure has been effective when the patient a. says that the nitroglycerin patch will be used for any chest pain that develops. b. calls when the weight increases from 124 to 130 pounds in a week. c. tells the home care nurse that furosemide (Lasix) is taken daily at bedtime. d. makes an appointment to see the doctor at least once yearly.

B Rationale: Teaching for a patient with heart failure includes information about the need to weigh daily and notify the health care provider about an increase of 3 pounds in 2 days or 5 pounds in a week. Nitroglycerin patches are used primarily to reduce preload (not to prevent chest pain) in patients with heart failure and should be used daily, not on an "as necessary" basis. Diuretics should be taken earlier in the day to avoid nocturia and sleep disturbance. Heart failure is a chronic condition that will require frequent follow-up rather than an annual health care provider examination. Cognitive Level: Application Text Reference: pp. 826, 833-834, 838 Nursing Process: Evaluation NCLEX: Health Promotion and Maintenance

What should the nurse recognize as an indication for the use of dopamine (Intropin) in the care of a patient with heart failure? A Acute anxiety B Hypotension and tachycardia C Peripheral edema and weight gain D Paroxysmal nocturnal dyspnea (PND)

B Hypotension and tachycardia Dopamine is a β-adrenergic agonist whose inotropic action is used for treatment of severe heart failure accompanied by hemodynamic instability. Such a state may be indicated by tachycardia accompanied by hypotension. PND, anxiety, edema, and weight gain are common signs and symptoms of heart failure, but these do not necessarily warrant the use of dopamine.

A 55-year-old with Stage D heart failure and type 2 diabetes asks the nurse whether heart transplant is a possible therapy. Which response by the nurse is appropriate? A) a. "Since you are diabetic, you would not be a candidate for a heart transplant." B) b. "The choice of a patient for a heart transplant depends on many different factors." C) c. "Your heart failure has not reached the stage in which heart transplants are considered." D) d. "People who have heart transplants are at risk for multiple complications after surgery."

B) "The choice of a patient for a heart transplant depends on many different factors."

A patient with ST segment elevation in several electrocardiographic (ECG) leads is admitted to the emergency department (ED) and diagnosed as having an ST-segment-elevation myocardial infarction (STEMI). Which question should the nurse ask to determine whether the patient is a candidate for fibrinolytic therapy? A) "Do you take aspirin on a daily basis?" B) "What time did your chest pain begin?" C) "Is there any family history of heart disease?" D) "Can you describe the quality of your chest pain?"

B) "What time did your chest pain begin?"

A patient develops sinus bradycardia at a rate of 32 beats/minute, has a BP of 80/36 mm Hg, and is complaining of feeling faint. Which action should the nurse take? A) Continue to monitor the rhythm and BP. B) Apply the transcutaneous pacemaker (TCP). C) Have the patient perform the Valsalva maneuver. D) Give the scheduled dose of diltiazem (Cardizem).

B) Apply the transcutaneous pacemaker (TCP). The patient is experiencing symptomatic bradycardia, and treatment with TCP isappropriate. Continued monitoring of the rhythm and BP is an inadequate response.Calcium channel blockers will further decrease the heart rate, and the diltiazem should beheld. The Valsalva maneuver will further decrease the rate

Which of the following patient teaching points should the nurse include when providing discharge instructions to a patient with a new permanent pacemaker and the caregiver? (select all that apply) a. avoid or limit air travel b. take and record a daily pulse rate c. obtain and wear a medic alert ID or bracelet at all times d. avoid lifting arm on the side of the pacemaker above the shoulder e. avoid microwave ovens because they interfere with pacemaker function

B,C,D

The nurse would assess a patient with complaints of chest pain for which of the following clinical manifestations associated with a myocardial infarction (MI) (select all that apply)? A) a. Flushing B) b. Ashen skin C) c. Diaphoresis D) d. Nausea and vomiting E) e. S3 or S4 heart sounds

B,C,D,E During the initial phase of an MI, catecholamines are released from the ischemic myocardial cells, causing an increased sympathetic nervous system (SNS) stimulation. This results in the release of glycogen, diaphoresis, and vasoconstriction of peripheral blood vessels. The patient's skin may be ashen, cool, and clammy (not flushed) as a result of this response. Nausea and vomiting may result from reflex stimulation of the vomiting center by severe pain. Ventricular dysfunction resulting from the MI may lead to the presence of the abnormal S3 and S4 heart sounds.

After you finished teaching a patient about sublingual nitroglycerin (Nitrostat), which statement indicates that the teaching has been effective? A. "It will help slow down the progress of the plaque formation by taking nitroglycerin." B. "I will call an ambulance if I still have pain 5 minutes after taking the nitroglycerin." C. "I can only take the nitroglycerin if I start to have chest pain." D. "I can expect indigestion as a side effect of nitroglycerin."

B. "I will call an ambulance if I still have pain 5 minutes after taking the nitroglycerin." - after taking 3 times Can be taken before chest pain (preventive) No side effect of indigestion

The blood bank notifies the nurse that the two units of blood ordered for an anemic patient are ready for pick up. The nurse should take which of the following actions to prevent an adverse effect during this procedure? A. Immediately pick up both units of blood from the blood bank. B. Correct Infuse the blood slowly for the first 15 minutes of the transfusion. C. Regulate the flow rate so that each unit takes at least 4 hours to transfuse. D. Set up the Y-tubing of the blood set with dextrose in water as the flush solution.

B. Because a transfusion reaction is more likely to occur at the beginning of a transfusion, the nurse should initially infuse the blood at a rate no faster than 2 ml/min and remain with the patient for the first 15 minutes after hanging a unit of blood.

Which diagnostic finding indicates that LV remodeling has occurred? A. Low ejection fraction B. Enlargement of the heart C. High BNP level D. Mitral regurgitation murmur

B. Enlargement of the heart

A patient with a diagnosis of hemophilia had a fall down an escalator earlier in the day and is now experiencing bleeding in her left knee joint. The emergency nurse's immediate response to this should include A. Immediate transfusion of platelets. B. Resting the patient's knee to prevent hemarthroses. C. Assistance with intracapsular injection of corticosteroids. D. Range-of-motion exercises to prevent thrombus formation.

B. In patients with hemophilia, joint bleeding requires resting of the joint in order to prevent deformities from hemarthrosis. Clotting factors, not platelets or steroids, are administered. Thrombus formation is not a central concern in a patient with hemophilia.

What are the main advantages of peritoneal dialysis compared to hemodialysis? A. No medications are required because of the enhanced efficiency of the peritoneal membrane in removing toxins. B. The diet is less restricted and dialysis can be performed at home. C. The dialysate is biocompatible and causes no long-term consequences. D. High glucose concentrations of the dialysate cause a reduction in appetite, promoting weight loss.

B. The diet is less restricted and dialysis can be performed at home. Advantages of peritoneal dialysis include fewer dietary restrictions and home dialysis is possible.

The nurse notes a physician's order written at 10:00 am for two units of packed red blood cells to be administered to a patient who is anemic as a result of chronic blood loss. If the transfusion is picked up at 11:30 am, the nurse should plan to hang the unit no later than which of the following times? A. 11:45 am B. 12:00 noon C. 12:30 pm D. 15:30 pm

B. noon. The nurse must hang the unit of packed red blood cells within 30 minutes of signing them out from the blood bank.

A patient admitted with ACS has continuous ECG monitoring. An examination of the rhythm strip reveals the following characteristics: atrial rate- 74 and regular; ventricular rate- 62 and irregular: p wave- normal shape; PR interval-lengthens progressively until a P wave is not conducted; QRS-normal shape. The priority nursing intervention would involve a. performing synchronized cardioversion b. administering epinephrine 1 mg IV push c. observing for symptoms of hypotension or angina d. preparing the patient for a transcutaneous pacemaker

C

A patient is recovering from an uncomplicated MI. Which of the following rehabilitation guidelines is a priority to include in the teaching plan? a. refrain from sexual activity for a minimum of 3 weeks b. plan a diet program that aims for a 1-to2- pound weight loss per week c. begin an exercise program that aims for at least five 30-minute sessions per week d. consider the use of erectile agents and prophylactic NTG before engaging in sexual activity

C

Important teaching for the patient scheduled for a radiofrequency catheter ablation procedure includes explaining that a. ventricular bradycardia may be induced and treated during the procedure b. a catheter will be placed in both femoral arteries to allow double catheter use c. the procedure will destroy areas of the conduction system that are causing rapid heart rhythms d. a general anesthetic will be given to prevent the awareness of any "sudden cardiac death" experiences.

C

The most common finding in individuals at risk for sudden cardiac death is a. aortic valve disease b. mitral valve disease c. left ventricular dysfunction d. atherosclerotic heart disease

C

The nurse determines that teaching about implementing dietary changes to decrease the risk of CAD has been effective when the patient says, a, "I should not eat any red meat such as beef, pork, or lamb." b. "I should have some type of fish at least 3 times a week." c. "Most of my fat intake should be from olive oil or the oils in nuts." d. "If I reduce the fat in my diet to about 5% of my calories, I will be much healthier."

C

The nurse is caring for a patient admitted with emphysema, angina, and hypertension. Before administering the prescribed daily dose of atenolol 100 mg PO, the nurse assesses the patient carefully. Which of the following adverse effects is this patient at risk for given the patient's health history? A) a. Hypocapnia B) b. Tachycardia C) c. Bronchospasm D) d. Nausea and vomiting

C

The nurse is caring for a patient who is 24 hours postpacemaker insertion. Which of the following nursing interventions is most appropriate at this time? A) a. Reinforcing the pressure dressing as needed B) b. Encouraging range-of-motion exercises of the involved arm C) c. Assessing the incision for any redness, swelling, or discharge D) d. Applying wet-to-dry dressings every 4 hours to the insertion site

C

When assessing the cardiovascular system of a 79-year-old patient, you might expect to find a. a narrowed pulse pressure b. diminished carotid artery pulses c. difficulty in isolating the apical pulse d. an increased heart rate in response to stress

C

Which of the following statements best describes the electrical activity of the heart represented by measuring the PR interval on the ECG? A) a. The length of time it takes to depolarize the atrium B) b. The length of time it takes for the atria to depolarize and repolarize C) c. The length of time for the electrical impulse to travel from the SA node to the Purkinje fibers D) d. The length of time it takes for the electrical impulse to travel from the SA node to the AV node

C

An elderly male patient visits his primary care provider because of burning on urination and production of urine that he describes as "foul smelling." The health care provider should assess the patient for which of the following factors that may dispose him to urinary tract infections (UTIs)? A High-purine diet B Sedentary lifestyle C Benign prostatic hyperplasia (BPH) D Recent use of broad-spectrum antibiotics

C BPH causes urinary stasis, which is a predisposing factor for UTIs. A sedentary lifestyle and recent antibiotic use are unlikely to contribute to UTIs, whereas a diet high in purines is associated with renal calculi.

While assessing the cardiovascular status of a patient, the nurse performs auscultation. Which of the following practices should the nurse implement into the assessment during auscultation? A) a. Position the patient supine. B) b. Ask the patient to hold his or her breath. C) c. Palpate the radial pulse while auscultating the apical pulse. D) d. Use the bell of the stethoscope when auscultating S1 and S2.

C In order to detect a pulse deficit, simultaneously palpate the radial pulse when auscultating the apical area. The diaphragm is more appropriate than the bell when auscultating S1 and S2. A sitting or side-lying position is most appropriate for cardiac auscultation. It is not necessary to ask the patient to hold his or her breath during cardiac auscultation.

A patient is recovering in the intensive care unit (ICU) after receiving a kidney transplant approximately 24 hours ago. Which of the following is an expected assessment finding for this patient during this early stage of recovery? A Hypokalemia B Hyponatremia C Large urine output D Leukocytosis with cloudy urine output

C Patients frequently experience diuresis in the hours and days immediately following a kidney transplant. Electrolyte imbalances and signs of infection are unexpected findings that warrant prompt intervention.

Postoperative care of a patient undergoing coronary artery bypass graft (CABG) surgery includes monitoring for which of the following common complications? A) a. Dehydration B) b. Paralytic ileus C) c. Atrial dysrhythmias D) d. Acute respiratory distress syndrome

C Postoperative dysrhythmias, specifically atrial dysrhythmias, are common in the first 3 days following CABG surgery. Although the other complications could occur, they are not common complications.

In caring for a patient admitted with poorly controlled hypertension, the nurse would understand that which of the following laboratory test results would indicate the presence of target organ damage secondary to the primary diagnosis? A) a. BUN of 15 mg/dl B) b. Serum uric acid of 3.8 mg/dl C) c. Serum creatinine of 2.6 mg/dl D) d. Serum potassium of 3.5 mEq/L

C The normal serum creatinine level is 0.6-1.3 mg/dl. This elevated level indicates target organ damage to the kidneys.

Which of the following ECG characteristics is consistent with a diagnosis of ventricular tachycardia (VT)? A) a. Unmeasurable rate and rhythm B) b. Rate 150 beats/min; inverted P wave C) c. Rate 200 beats/min; P wave not visible D) d. Rate 125 beats/min; normal QRS complex

C VT is associated with a rate of 150 to 250 beats/min; the P wave is not normally visible. P wave inversion and a normal QRS complex are not associated with VT. Rate and rhythm are not measurable in ventricular fibrillation.

Delegation Decision: Which nursing interventions could be delegated to unlicensed assistive personnel (UAP) (select all that apply)? a. Assess the need for catheterization. b. Use bladder scanner to estimate residual urine. c. Teach patient pelvic floor muscle (Kegel) exercises. d. Insert indwelling catheter for uncomplicated patient. e. Assist incontinent patient to commode at regular intervals. f. Provide perineal care with soap and water around a urinary catheter.

e, f. The unlicensed assistive personnel (UAP) may assist the incontinent patient to void at regular intervals and provide perineal care. An RN should perform the assessments and teaching. In long-term care and rehabilitation facilities, UAP may use bladder scanners after they are trained.

2. A patient with chronic heart failure who has been following a low-sodium diet tells the nurse at the clinic about a 5-pound weight gain in the last 3 days. The nurse's first action will be to a. ask the patient to recall the dietary intake for the last 3 days because there may be hidden sources of sodium in the patient's diet. b. instruct the patient in a low-calorie, low-fat diet because the weight gain has likely been caused by excessive intake of inappropriate foods. c. assess the patient for clinical manifestations of acute heart failure because an exacerbation of the chronic heart failure may be occurring. d. educate the patient about the use of diuretic therapy because it is likely that the patient will need medications to reduce the hypervolemia.

C Rationale: The 5-pound weight gain over 3 days indicates that the patient's chronic heart failure may be worsening; it is important that the patient be immediately assessed for other clinical manifestations of decompensation, such as lung crackles. A dietary recall to detect hidden sodium in the diet and teaching about diuretic therapy are appropriate interventions but are not the first nursing actions indicated. There is no evidence that the patient's weight gain is caused by excessive dietary intake of fat or calories, so the answer beginning "instruct the patient in a low-calorie, low-fat diet" describes an inappropriate action. Cognitive Level: Application Text Reference: p. 826 Nursing Process: Assessment NCLEX: Physiological Integrity

20. An elderly patient with a 40-pack-year history of smoking and a recent myocardial infarction is admitted to the medical unit with acute shortness of breath; the nurse need to rule out pneumonia versus heart failure. The diagnostic test that the nurse will monitor to help in determining whether the patient has heart failure is a. 12-lead electrocardiogram (ECG). b. arterial blood gases (ABGs). c. B-type natriuretic peptide (BNP). d. serum creatine kinase (CK).

C Rationale: BNP is secreted when ventricular pressures increase, as with heart failure, and elevated BNP indicates a probable or very probable diagnosis of heart failure. 12-lead ECGs, ABGs, and CK may also be used in determining the causes or effects of heart failure but are not as clearly diagnostic of heart failure as BNP. Cognitive Level: Application Text Reference: p. 827 Nursing Process: Assessment NCLEX: Physiological Integrity

3. During assessment of a 72-year-old with ankle swelling, the nurse notes jugular venous distention (JVD) with the head of the patient's bed elevated 45 degrees. The nurse knows this finding indicates a. decreased fluid volume. b. incompetent jugular vein valves. c. elevated right atrial pressure. d. jugular vein atherosclerosis.

C Rationale: The jugular veins empty into the superior vena cava and then into the right atrium, so JVD with the patient sitting at a 45-degree angle reflects elevated right atrial pressure. JVD is an indicator of excessive fluid volume (increased preload), not decreased fluid volume; it is not caused by incompetent jugular vein valves or atherosclerosis. Cognitive Level: Comprehension Text Reference: p. 825 Nursing Process: Assessment NCLEX: Physiological Integrity

9. During a visit to an elderly patient with chronic heart failure, the home care nurse finds that the patient has severe dependent edema and that the legs appear to be weeping serous fluid. Based on these data, the best nursing diagnosis for the patient is a. activity intolerance related to venous congestion. b. disturbed body image related to massive leg swelling. c. impaired skin integrity related to peripheral edema. d. impaired gas exchange related to chronic heart failure.

C Rationale: The patient's findings of severe dependent edema and weeping serous fluid from the legs support the nursing diagnosis of impaired skin integrity. There is less evidence for the nursing diagnoses of activity intolerance, disturbed body image, and impaired gas exchange, although the nurse will further assess the patient to determine whether there are other clinical manifestations of heart failure to indicate that these diagnoses are appropriate. Cognitive Level: Application Text Reference: p. 836 Nursing Process: Diagnosis NCLEX: Physiological Integrity

A patient is admitted to the coronary care unit following a cardiac arrest and successful cardiopulmonary resuscitation. When reviewing the health care provider's admission orders, which of the following orders is most important for the nurse to question? a. Oxygen at 4 L/min per nasal cannula b. Morphine sulfate 2 mg IV every 10 minutes until the pain is relieved c. Tissue plasminogen activator (t-PA) 100 mg IV infused over 3 hours d. IV nitroglycerin at 5 mcg/min; increase by 5 mcg/min every 3 to 5 minutes

C Rationale: Traumatic or prolonged (>10 minutes) cardiopulmonary resuscitation is a relative contraindication for the administration of fibrinolytic therapy.

Which of these statements made by a patient with coronary artery disease after the nurse has completed teaching about the therapeutic lifestyle changes (TLC) diet indicates that further teaching is needed? A) "I will switch from whole milk to 1% or nonfat milk." B) "I like fresh salmon and I will plan to eat it more often." C) "I will miss being able to eat peanut butter sandwiches." D) "I can have a cup of coffee with breakfast if I want one."

C "I will miss being able to eat peanut butter sandwiches."

After the nurse has finished teaching a patient about use of sublingual nitroglycerin (Nitrostat), which patient statement indicates that the teaching has been effective? A) "I can expect indigestion as a side effect of nitroglycerin." B) "I can only take the nitroglycerin if I start to have chest pain." C) "I will call an ambulance if I still have pain 5 minutes after taking the nitroglycerin." D) "I will help slow down the progress of the plaque formation by taking nitroglycerin."

C) "I will call an ambulance if I still have pain 5 minutes after taking the nitroglycerin."

A patient has received instruction on the management of a new permanent pacemaker before discharge from the hospital. The nurse recognizes that teaching has been effective when the patient tells the nurse, A) "It will be 6 weeks before I can take a bath or return to my usual activities." B) "I will notify the airlines when I make a reservation that I have a pacemaker." C) "I won't lift the arm on the pacemaker side up very high until I see the doctor." D) "I must avoid cooking with a microwave oven or being near a microwave in use."

C) "I won't lift the arm on the pacemaker side up very high until I see the doctor." The patient is instructed to avoid lifting the arm on the pacemaker side above theshoulder to avoid displacing the pacemaker leads. The patient should notify airportsecurity about the presence of a pacemaker before going through the metal detector, butthere is no need to notify the airlines when making a reservation. Microwave oven usedoes not affect the pacemaker. The insertion procedure involves minor surgery that willhave a short recovery period.

A patient who was admitted with a myocardial infarction experiences a 50-second episode of ventricular tachycardia, then converts to sinus rhythm with a heart rate of 98 beats/minute. Which action should the nurse take next? A) Notify the health care provider. B) Perform synchronized cardioversion. C) Administer the PRN IV lidocaine (Xylocaine). D) Document the rhythm and monitor the patient.

C) Administer the PRN IV lidocaine (Xylocaine). The burst of sustained ventricular tachycardia indicates that the patient has significantventricular irritability, and antidysrhythmic medication administration is needed to prevent further episodes. The nurse should notify the health care provider after the medications are administered. Defibrillation is not indicated given that the patient is currently in a sinus rhythm. Documentation and continued monitoring are not adequate responses to this situation.

Which diagnostic test will be most useful to the nurse in determining whether a patient admitted with acute shortness of breath has heart failure? A) a. Serum creatine kinase (CK) B) b. Arterial blood gases (ABGs) C) c. B-type natriuretic peptide (BNP) D) d. 12-lead electrocardiogram (ECG)

C) B-type natriuretic peptide (BNP)

While assessing a patient who was admitted with heart failure, the nurse notes that the patient has jugular venous distention (JVD) when lying flat in bed. Which action should the nurse take next? A) Use a ruler to measure the level of the JVD. B) Document this finding in the patient's record. C) Observe for JVD with the head at 30 degrees. D) Have the patient perform the Valsalva maneuver.

C) Observe for JVD with the head at 30 degrees.

The nurse needs to estimate quickly the heart rate for a patient with a regular heart rhythm. Which method will be best to use? A) Print a 1-minute electrocardiogram (ECG) strip and count the number of QRS complexes. B) Count the number of large squares in the R-R interval and divide by 300. C) Use the 3-second markers to count the number of QRS complexes in 6 seconds and multiply by 10. D) Calculate the number of small squares between one QRS complex and the next and divide into 1500.

C) Use the 3-second markers to count the number of QRS complexes in 6 seconds and multiply by 10. This is the quickest way to determine the ventricular rate for a patient with a regular rhythm. All the other methods are accurate, but take longer.

During the administration of the fibrinolytic agent to a patient with an acute myocardial infarction (AMI), the nurse should stop the drug infusion if the patient experiences A) bleeding from the gums. B) surface bleeding from the IV site. C) a decrease in level of consciousness. D) a nonsustained episode of ventricular tachycardia.

C) a decrease in level of consciousness.

A patient with dilated cardiomyopathy has an atrial fibrillation that has been unresponsive to drug therapy for several days. The nurse anticipates that the patient may need teaching about A) electrical cardioversion. B) IV adenosine (Adenocard). C) anticoagulant therapy with warfarin (Coumadin). D) insertion of an implantable cardioverter-defibrillator (ICD).

C) anticoagulant therapy with warfarin (Coumadin). Atrial fibrillation therapy that has persisted for more than 48 hours requires anticoagulanttreatment for 3 weeks before attempting cardioversion; this is done to preventembolization of clots from the atria. Adenosine is not used to treat atrial fibrillation.Cardioversion may be done after several weeks of Coumadin therapy. ICDs are used for patients with recurrent ventricular fibrillation.

The nurse plans discharge teaching for a patient with chronic heart failure who has prescriptions for digoxin (Lanoxin) and hydrochlorothiazide (HydroDIURIL). Appropriate instructions for the patient include A) avoid dietary sources of potassium. B) take the hydrochlorothiazide before bedtime. C) notify the health care provider about any nausea. D) never take digoxin if the pulse is below 60 beats/minute

C) notify the health care provider about any nausea.

The patient in the oliguric phase of AKI excreted 300 mL of urine in addition to 100 mL of other losses during the past 24 hours. With appropriate calculations, you determine that for the next 24 hours the patient's fluid allocation is A. 600 mL. B. 800 mL. C. 1000 mL. D. 1200 mL.

C. 1000 mL. Fluid intake must be closely monitored during the oliguric phase. The rule for calculating the fluid restriction is to add all losses for the previous 24 hours to 600 mL for insensible losses.

When preparing to administer an ordered blood transfusion, the nurse selects which of the following intravenous solutions to use when priming the blood tubing? A. Lactated Ringer's B. 5% Dextrose in water C. 0.9% NaCl D. 0.45% NaCl

C. The blood set should be primed before the transfusion with 0.9% sodium chloride, also known as normal saline. It is also used to flush the blood tubing after the infusion is complete to ensure the patient receives blood that is left in the tubing when the bag is empty.

Atrial fibrillation puts the patient at risk for thrombosis. This risk can be prevented by taking what medication?

Couamdin

The blood pressure of a 71-year-old patient admitted with pneumonia is 160/70 mm Hg. Which of the following is an age-related change that contributes to this finding? A) a. Stenosis of the heart valves B) b. Decreased adrenergic sensitivity C) c. Increased parasympathetic activity D) d. Loss of elasticity in arterial vessels

D

Which of the following instructions given to a patient who is about to undergo Holter monitoring is most appropriate? A) a. "You may remove the monitor only to shower or bathe." B) b. "You should connect the monitor whenever you feel symptoms." C) c. "You should refrain from exercising while wearing this monitor." D) d. "You will need to keep a diary of all your activities and symptoms."

D A Holter monitor is worn for at least 24 hours while a patient continues with usual activity and keeps a diary of activities and symptoms. The patient should not take a bath or shower while wearing this monitor.

For which of the following antilipemic medications would the nurse question an order in a patient with cirrhosis of the liver? A) a. Niacin (Nicobid) B) b. Ezetimibe (Zetia) C) c. Gemfibrozil (Lopid) D) d. Atorvastatin (Lipitor)

D Adverse effects of atorvastatin (Lipitor), a statin drug, include liver damage and myopathy. Liver enzymes must be monitored frequently and the medication stopped if these enzymes increase. Thus liver disease is a contraindication for atorvastatin.

For which of the following is percutaneous coronary intervention (PCI) most clearly indicated? A) a. Chronic stable angina B) b. Left-sided heart failure C) c. Coronary artery disease D) d. Acute myocardial infarction

D PCI is indicated to restore coronary perfusion in cases of myocardial infarction. Chronic stable angina and CAD are normally treated with more conservative measures initially. PCI is not relevant to the pathophysiology of heart failure.

When caring for a patient during the oliguric phase of acute kidney injury, which of the following would be an appropriate nursing intervention? A Weigh patient three times weekly. B Increase dietary sodium and potassium. C Provide a low-protein, high-carbohydrate diet. D Restrict fluids according to previous daily loss.

D Patients in the oliguric phase of acute kidney injury will have fluid volume excess with potassium and sodium retention; hence, they will need to have dietary sodium, potassium, and fluids restricted. Daily fluid intake is based on the previous 24-hour fluid loss (measured output plus 600 ml for insensible loss). The diet also needs to provide adequate, not low, protein intake to prevent catabolism. The patient should also be weighed daily, not just three times a week.

The nurse preparing to administer a dose of PhosLo to a patient with chronic kidney disease would interpret that this medication should have a beneficial effect on which of the following laboratory values of the patient? A Sodium B Potassium C Magnesium D Phosphorus

D Phosphorus and calcium have inverse or reciprocal relationships, meaning that when phosphorus levels are high, calcium levels tend to be low. Therefore administration of calcium should help to reduce a patient's abnormally high phosphorus level, as seen with chronic kidney disease.

A patient was admitted to the emergency department 24 hours earlier with complaints of chest pain that were subsequently attributed to ST-segment-elevation myocardial infarction (STEMI). Which of the following complications of MI should the nurse anticipate? A) a. Unstable angina B) b. Cardiac tamponade C) c. Sudden cardiac death D) d. Cardiac dysrhythmias

D The most common complication after MI is dysrhythmias, which are present in 80% of patients. Unstable angina is considered a precursor to MI rather than a complication. Cardiac tamponade is a rare event, and sudden cardiac death is defined as an unexpected death from cardiac causes; cardiac dysfunction in the period following an MI would not be characterized as sudden cardiac death.

17. Following an acute myocardial infarction, a previously healthy 67-year-old patient develops clinical manifestations of heart failure. The nurse anticipates discharge teaching will include information about a. digitalis preparations, such as digoxin (Lanoxin). b. calcium-channel blockers, such as diltiazem (Cardizem). c. -adrenergic agonists, such as dobutamine (Dobutrex). d. angiotensin-converting enzyme (ACE) inhibitors, such as captopril (Capoten).

D Rationale: ACE-inhibitor therapy is currently recommended to prevent the development of heart failure in patients who have had a myocardial infarction and as a first-line therapy for patients with chronic heart failure. Digoxin therapy for heart failure is no longer considered a first-line measure, and digoxin is added to the treatment protocol when therapy with other medications such as ACE-inhibitors, diuretics, and -adrenergic blockers is insufficient. Calcium-channel blockers are not generally used in the treatment of heart failure. The -adrenergic agonists such as dobutamine are administered through the IV route and are not used as initial therapy for heart failure. Cognitive Level: Application Text Reference: p. 832 Nursing Process: Implementation NCLEX: Physiological Integrity

23. A hospitalized patient with heart failure has a new order for captopril (Capoten) 12.5 mg PO. After administering the first dose and teaching the patient about captopril, which statement by the patient indicates that teaching has been effective? a. "I will need to include more high-potassium foods in my diet." b. "I will expect to feel more short of breath for the next few days." c. "I will be sure to take the medication after eating something." d. "I will call for help when I need to get up to the bathroom."

D Rationale: Captopril can cause hypotension, especially after the initial dose, so it is important that the patient not get up out of bed without assistance until the nurse has had a chance to evaluate the effect of the first dose. The ACE inhibitors are potassium sparring, and the nurse should not teach the patient to increase sources of dietary potassium. Increased shortness of breath is expected with initiation of -blocker therapy for heart failure, not for ACE-inhibitor therapy. ACE inhibitors are best absorbed when taken an hour before eating. Cognitive Level: Application Text Reference: p. 832 Nursing Process: Evaluation NCLEX: Physiological Integrity

18. A 55-year-old patient with inoperable coronary artery disease and end-stage heart failure asks the nurse whether heart transplant is a possible therapy. The nurse's response to the patient will be based on the knowledge that a. heart transplants are experimental surgeries that are not covered by most insurance. b. the patient is too old to be placed on the transplant list. c. the diagnoses and symptoms indicate that the patient is not an appropriate candidate. d. candidacy for heart transplant depends on many factors.

D Rationale: Indications for a heart transplant include inoperable coronary artery disease and refractory end-stage heart failure, but other factors such as coping skills, family support, and patient motivation to follow the rigorous post-transplant regimen are also considered. Heart transplants are not considered experimental; rather, transplantation has become the treatment of choice for patients who meet the criteria. The patient is not too old for a transplant. The patient's diagnoses and symptoms indicate that the patient may be an appropriate candidate for a heart transplant. Cognitive Level: Comprehension Text Reference: p. 837 Nursing Process: Planning NCLEX: Health Promotion and Maintenance

12. When teaching the patient with heart failure about a 2000-mg sodium diet, the nurse explains that foods to be restricted include a. eggs and other high-cholesterol foods. b. canned and frozen fruits. c. fresh or frozen vegetables. d. milk, yogurt, and other milk products.

D Rationale: Milk and yogurt naturally contain a significant amount of sodium, and intake of these should be limited for patients on a diet that limits sodium to 2000 mg daily. Other milk products, such as processed cheeses, have very high levels of sodium and are not appropriate for a 2000-mg sodium diet. The other foods listed have minimal levels of sodium and can be eaten without restriction. Cognitive Level: Application Text Reference: p. 833 Nursing Process: Implementation NCLEX: Health Promotion and Maintenance

The nurse is administering a dose of digoxin (Lanoxin) to a patient with heart failure (HF). The nurse would become concerned with the possibility of digitalis toxicity if the patient reported which symptom(s)? A Muscle aches B Constipation C Pounding headache D Anorexia and nausea

D Anorexia and nausea Anorexia, nausea, vomiting, blurred or yellow vision, and cardiac dysrhythmias are all signs of digitalis toxicity. The nurse would become concerned and notify the health care provider if the patient exhibited any of these symptoms.

A stable patient with acute decompensated heart failure (ADHF) suddenly becomes dyspneic. Before positioning the patient on the bedside, what should the nurse assess first? A Urine output B Heart rhythm C Breath sounds D Blood pressure

D Blood pressure The nurse should evaluate the blood pressure before dangling the patient on the bedside because the blood pressure can decrease as blood pools in the periphery and preload decreases. If the patient's blood pressure is low or marginal, the nurse should put the patient in the semi-Fowler's position and use other measures to improve gas exchange.

Beyond the first year after a heart transplant, the nurse knows that what is a major cause of death? A Infection B Acute rejection C Immunosuppression D Cardiac vasculopathy

D Cardiac vasculopathy Beyond the first year after a heart transplant, malignancy (especially lymphoma) and cardiac vasculopathy (accelerated CAD) are the major causes of death. During the first year after transplant, infection and acute rejection are the major causes of death. Immunosuppressive therapy will be used for posttransplant management to prevent rejection and increases the patient's risk of an infection.

A male patient with a long-standing history of heart failure has recently qualified for hospice care. What measure should the nurse now prioritize when providing care for this patient? A Taper the patient off his current medications. B Continue education for the patient and his family. C Pursue experimental therapies or surgical options. D Choose interventions to promote comfort and prevent suffering.

D Choose interventions to promote comfort and prevent suffering. The central focus of hospice care is the promotion of comfort and the prevention of suffering. Patient education should continue, but providing comfort is paramount. Medications should be continued unless they are not tolerated. Experimental therapies and surgeries are not commonly used in the care of hospice patients.

After the nurse teaches a patient with chronic stable angina about how to use the prescribed short-acting and long-acting nitrates, which statement by the patient indicates that the teaching has been effective? A) "I will put on the nitroglycerin patch as soon as I develop any chest pain." B) "I will check the pulse rate in my wrist just before I take any nitroglycerin." C) "I will be sure to remove the nitroglycerin patch before using any sublingual nitroglycerin." D) "I will stop what I am doing and sit down before I put the nitroglycerin under my tongue."

D) "I will stop what I am doing and sit down before I put the nitroglycerin under my tongue."

Which laboratory result for a patient whose cardiac monitor shows multifocal premature ventricular contractions (PVCs) is most important for the nurse to communicate to the health care provider? A) Blood glucose 228 mg/dL B) Serum chloride 90 mEq/L C) Serum sodium 133 mEq/L D) Serum potassium 2.8 mEq/L

D) Serum potassium 2.8 mEq/L Hypokalemia increases the risk for ventricular dysrhythmias such as PVCs, ventricular tachycardia, and ventricular fibrillation; the health care provider will need to prescribe a potassium infusion to correct this abnormality. Although the other laboratory values alsoare abnormal, they are not likely to be the etiology of the patient's PVCs and do notrequire immediate correction.

Which information given by a patient admitted with chronic stable angina will help the nurse confirm this diagnosis? A) The patient rates the pain at a level 3 to 5 (0 to 10 scale). B) The patient states that the pain "wakes me up at night." C) The patient says that the frequency of the pain has increased over the last few weeks. D) The patient states that the pain is resolved after taking one sublingual nitroglycerin tablet.

D) The patient states that the pain is resolved after taking one sublingual nitroglycerin tablet.

A patient has a normal cardiac rhythm and a heart rate of 72 beats/minute, except that the PR interval is 0.24 seconds. The appropriate intervention by the nurse is to A) notify the patient's health care provider immediately. B) administer atropine per agency bradycardia protocol. C) prepare the patient for temporary pacemaker insertion. D) document the finding and continue to monitor the patient.

D) document the finding and continue to monitor the patient. First-degree atrioventricular (AV) block is asymptomatic and requires on going monitoring because it may progress to more serious forms of heart block. The rate isnormal, so there is no indication that atropine is needed. Immediate notification of the health care provider about an asymptomatic rhythm is not necessary.

A patient who has chronic heart failure tells the nurse, "I felt fine when I went to bed, but I woke up in the middle of the night feeling like I was suffocating!" The nurse will document this assessment information as A) pulsus alternans. B) two-pillow orthopnea. C) acute bilateral pleural effusion. D) paroxysmal nocturnal dyspnea.

D) paroxysmal nocturnal dyspnea.

Nadolol (Corgard) is prescribed for a patient with angina. To determine whether the drug is effective, the nurse will monitor for A) decreased blood pressure and apical pulse rate. B) fewer complaints of having cold hands and feet. C) improvement in the quality of the peripheral pulses. D) the ability to do daily activities without chest discomfort.

D) the ability to do daily activities without chest discomfort.

Which of the follow is not a way of administering Nitroglycerin? A. Oral B. Sublingual C. IV D. Eye drop

D. Eye drop

What risk factors for hypertension? A. Ethnicity B. Smoking C. Alcohol consumption D. Gender E. All of the Above

E. All of the Above

What assessment finding is indicative of target organ damage? A. ECG changes. B. Visual problems. C. Kidney dysfunction. D. Palpable apical pulse 5th ICS lateral to the MCL E. All of the above

E. All of the above ECG - heart failure Visual problems - i.e. glaucoma Kidney - renal failure Apical false - enlarged heart

A patient is being discharged after the insertion of a permanent pacemaker. Which statement made by the patient indicates an understanding regarding appropriate self-care? "Every morning I will perform arm and shoulder stretches." "Each day I'll take my pulse and record it in a log." "I'll have to get rid of my microwave oven." "I won't be able to use my electric blanket anymore."

Each day I'll take my pulse and record it in a log

Difference between hypertension emergency (crisis) and urgency

Emergency develops over hours or days, more about how fast the BP elevates, symptom: severe headache, seizure, nausea, vomiting, altered level of consciousness, coma Urgency develops over days weeks, no target organ damage

The nurse discusses mineral therapy with a client diagnosed with atrial fibrillation. Which of the following deficiencies will the nurse most likely discuss with the client?

Magnesium Magnesium deficiency is associated with dysrhythmias, including atrial fibrillation, premature atrial and ventricular contractions, ventricular tachycardia, and ventricular fibrillation

A, B, C, D

Priority nursing actions when caring for a hospitalized patient with a new onset temperature of 102.2*F and severe neurtopenia include: (check all that apply): A. administering the prescribed antibiotic STAT B. drawing peripheral and ventral line blood cultured C. ongoing monitoring of the patient's vitals for signs of septic shock D. taking a full set of vital signs and notifying the physician immediately E. administering infusions of WBCs treated to decrease immunogenicity

The diagnosis of dilated cardiomyopathy is based on A. chest radiography. B. contrast-enhanced computed tomography (CT). C. Doppler echocardiography. D. elevated levels of brain natriuretic peptide (BNP).

Rationale: C Doppler echocardiography provides the basis for the diagnosis of dilated cardiomyopathy in the majority of patients. Chest radiography can detect only cardiac enlargement and cannot differentiate possible causes. CT and BNP do not aid in the diagnosis of dilated cardiomyopathy Reference: 861

D

The nurse would anticipate that a patient with Von Willebrand disease undergoing surgery would be treated with administration of vWF and: A. thrombin B. factor VI C. factor VII D. factor VIII

What is the result of NTG interaction with Viagra?

They are both vasodilators can cause lethal hypotension. Ask, "when is the last time you took Viagra?" Viagra Cialis and Levitra (same class) Women can take it for pulmonary hypertension

Why did the physician order another 12-lead ECG?

To make a comparison after treatment

Which cardiac enzyme is the most sensitive and specific for myocardiac damage?

Troponin

What cardiac enzymes that routinley tested in ACS (Acute Coronary Syndrome)?

Troponin, CKMB, myoglobin

Substernal pain is highly associated with CV events. True or False.

True

The major pathology in atrial fibrillation (a. fib) is that the SA nodes fires at a very fast pace. True or False

True

When a patient presents with chest pain, you should assume it is of a cardiac origin until that can be ruled out. True or False

True

D

When reviewing the patient's hematologic laboratory values after a splenectomy, the nurse would expect to find: A. leukopenia B. RBC abnormalities C. decreased hemoglobin D. increased platelet count

Prevention of AKI is important because of the high mortality rate. Which patients are at increased risk for AKI (select all that apply)? a. An 86-year-old woman scheduled for a cardiac catheterization b. A 48-year-old man with multiple injuries from a motor vehicle accident c. A 32-year-old woman following a C-section delivery for abruptio placentae d. A 64-year-old woman with chronic heart failure admitted with bloody stools e. A 58-year-old man with prostate cancer undergoing preoperative workup for prostatectomy

a, b, c, d, e. High-risk patients include those exposed to nephrotoxic agents and advanced age (a), massive trauma (b), prolonged hypovolemia or hypotension (possibly b and c), obstetric complications (c), cardiac failure (d), preexisting chronic kidney disease, extensive burns, or sepsis. Patients with prostate cancer may have obstruction of the outflow tract, which increases risk of postrenal AKI (e).

Which classification of urinary tract infection (UTI) is described as infection of the renal parenchyma, renal pelvis, and ureters? a. Upper UTI c. Complicated UTI b. Lower UTI d. Uncomplicated UTI

a. An upper urinary tract infection (UTI) affects the renal parenchyma, renal pelvis, and ureters. A lower UTI is an infection of the bladder and/or urethra. A complicated UTI exists in the presence of obstruction, stones, or preexisting diseases. An uncomplicated UTI occurs in an otherwise normal urinary tract.

When obtaining a nursing history from a patient with cancer of the urinary system, what does the nurse recognize as a risk factor associated with both kidney cancer and bladder cancer? a. Smoking b. Family history of cancer c. Chronic use of phenacetin d. Chronic, recurrent nephrolithiasis

a. Both cancer of the kidney and cancer of the bladder are associated with smoking. A family history of renal cancer is a risk factor for kidney cancer and cancer of the bladder has been associated with the use of phenacetin-containing analgesics and recurrent upper UTIs.

Besides being mixed with struvite or oxalate stones, what characteristic is associated with calcium phosphate calculi? a. Associated with alkaline urine b. Genetic autosomal recessive defect c. Three times as common in women as in men d. Defective gastrointestinal (GI) and kidney absorption

a. Calcium phosphate calculi are typically mixed with struvite or oxalate stones and related to alkaline urine. Cystine calculi are associated with a genetic autosomal recessive defect and defective GI and kidney absorption of cystine. Struvite calculi are three to four times more common in women than in men.

Priority Decision: A patient on a medical unit has a potassium level of 6.8 mEq/L. What is the priority action that the nurse should take? a. Place the patient on a cardiac monitor. b. Check the patient's blood pressure (BP). c. Instruct the patient to avoid high-potassium foods. d. Call the lab and request a redraw of the lab to verify results.

a. Dysrhythmias may occur with an elevated potassium level and are potentially lethal. Monitor the rhythm while contacting the physician or calling the rapid response team. Vital signs should be checked. Depending on the patient's history and cause of increased potassium, instruct the patient about dietary sources of potassium; however, this would not help at this point. The nurse may want to recheck the value but until then the heart rhythm needs to be monitored.

Nursing implementation for Acute Pyelonephritis

a. Health promotion i. Early treatment for cystitis to prevent ascending infection 1. Patient with structural abnormalities is at high risk 2. Stress the need for regular medical care. b. Ambulatory and home care i. Need to continue drugs as prescribed ii. Need for follow-up urine culture iii. Identification of risk for recurrence or relapse iv. Encourage adequate fluids. v. Rest to increase comfort vi. Low-dose, long-term antibiotics to prevent relapse or reinfections vii. Explain rationale to enhance compliance.

A patient received a kidney transplant last month. Because of the effects of immunosuppressive drugs and CKD, what complication of transplantation should the nurse be assessing the patient for to decrease the risk of mortality? a. Infection b. Rejection c. Malignancy d. Cardiovascular disease

a. Infection is a significant cause of morbidity and mortality after transplantation because the surgery, the immunosuppressive drugs, and the effects of CKD all suppress the body's normal defense mechanisms, thus increasing the risk of infection. The nurse must assess the patient as well as use aseptic technique to prevent infections. Rejection may occur but for other reasons. Malignancy occurrence increases later due to immunosuppressive therapy. Cardiovascular disease is the leading cause of death after renal transplantation but this would not be expected to cause death within the first month after transplantation.

The nurse plans care for the patient with APSGN based on what knowledge? a. Most patients with APSGN recover completely or rapidly improve with conservative management. b. Chronic glomerulonephritis leading to renal failure is a common sequela to acute glomerulonephritis. c. Pulmonary hemorrhage may occur as a result of antibodies also attacking the alveolar basement membrane. d. A large percentage of patients with APSGN develop rapidly progressive glomerulonephritis, resulting in kidney failure.

a. Most patients recover completely from acute post streptococcal glomerulonephritis (APSGN) with supportive treatment. Chronic glomerulonephritis that progresses insidiously over years and rapidly progressive glomerulonephritis that results in renal failure within weeks or months occur in only a few patients with APSGN. In Goodpasture syndrome, antibodies are present against both the GBM and the alveolar basement membrane of the lungs and dysfunction of both renal and pulmonary are present.

A patient whose cardiac monitor shows sinus tachycardia, rate 102, is apneic and no pulses are palpable by the nurse. What is the first action that the nurse should take? a. Start CPR. b. Defibrillate. c. Administer atropine per hospital protocol. d. Give 100% oxygen per non-rebreather mask.

a. Start CPR. The patient's clinical manifestations indicate pulseless electrical activity and the nurseshould immediately start CPR. The other actions would not be of benefit to this patient.

Which urinary diversion is a continent diversion created by formation of an ileal pouch with a stoma for catheterization? a. Kock pouch b. Ileal conduit c. Orthotopic neobladder d. Cutaneous ureterostomy

a. The Kock pouch is a continent diversion created by formation of an ileal pouch with an external stoma requiring catheterization. Ileal conduit is the most common incontinent diversion using a stoma of resected ileum with implanted ureters. Orthotopic neobladder is a new bladder from a reshaped segment of intestine in the anatomic position of the bladder with urine discharged through the urethra. A cutaneous ureterostomy diverts the ureter from the bladder to the abdominal skin but there is frequent scarring and strictures of the ureters, so ileal conduits are used more often.

A man with end-stage kidney disease is scheduled for hemodialysis following healing of an arteriovenous fistula (AVF). What should the nurse explain to him that will occur during dialysis? a. He will be able to visit, read, sleep, or watch TV while reclining in a chair. b. He will be placed on a cardiac monitor to detect any adverse effects that might occur. c. The dialyzer will remove and hold part of his blood for 20 to 30 minutes to remove the waste products. d. A large catheter with two lumens will be inserted into the fistula to send blood to and return it from the dialyzer.

a. While patients are undergoing hemodialysis, they can perform quiet activities that do not require the limb that has the vascular access. Blood pressure is monitored frequently and the dialyzer monitors dialysis function but cardiac monitoring is not usually indicated. The hemodialysis machine continuously circulates both the blood and the dialysate past the semipermeable membrane in the machine. Graft and fistula access involve the insertion of two needles into the site: one to remove blood from and the other to return blood to the dialyzer.

*4.One of the nruse's most important roles in relation to acute poststreptococcal golmerulonephritis is to* a. promote early diagnosis and treatment of sore throats and skin lesions b.encourage patients to obtain antibiotic therapy for upper respiratory tract infections c.teach patients with APSGN that long term prophylactic antibiotic therapy is necessary to prevent recurrence d.monitor patients for respiratory symptoms that indicate the disease is affecting the alveolar basement membrane

a. promote early diagnosis and treatment of sore throats and skin lesions

Nurse is caring for a client on telemetry who has irregular radial pulse. The progress notes say that the client has had atrial flutter? Whick EKG pattern? p waves at 0.16 seconds before each QRS atrial rate of 300 with 80 QRS per min ventricular rate of 82 with atrial rate of 80 irregular ventricular rate of 125, with wide QRS

atrial rate of 300 with 80 QRS

A client's job involves working in a warm, dry room, frequently bending and crouching to check the underside of a high-speed press, and wearing eye guards. Given this information, the nurse should assess the client for which of the following? 1. Muscle aches. 2. Thirst. 3. Lethargy. 4. Orthostatic hypotension.

4. Possible dizziness from orthostatic hypotension when rising from a crouched or bent position increases the client's risk of being injured by the equipment. The nurse should assess the client's blood pressure in all three positions (lying, sitting, and standing) at all routine visits. The client may experience muscle aches, or thirst from working in a warm, dry room, but these are not as potentially dangerous as orthostatic hypotension. The client should not be experiencing lethargy.

The nurse notes that a patient's cardiac monitor shows that every other beat is earlier thanexpected, has no P wave, and has a QRS complex with a wide and bizarre shape. How will the nurse document the rhythm? A. Ventricular couplets b .Ventricular bigeminy c. Ventricular R-on-T phenomenon d. Ventricular multifocal contractions

b .Ventricular bigeminy Ventricular bigeminy describes a rhythm in which every other QRS complex is wide and bizarre looking. Pairs of wide QRS complexes are described as couplets. There is noindication that the premature ventricular contractions (PVCs) are multifocal or that the R-on-T phenomenon is occurring.

The male patient is Jewish, has a history of gout, and has been diagnosed with renal calculi. Which treatment will be used with this patient (select all that apply)? a. Reduce dietary oxalate b. Administer allopurinol c. Administer α-penicillamine d. Administer thiazide diuretics e. Reduce animal protein intake f. Reduce intake of milk products

b, e. This patient is most likely to have uric acid calculi,which have a high incidence in Jewish men, and gout is a predisposing factor. The treatment will include allopurinol and reducing animal protein intake to reduce purine, as uric acid is a waste product from purine metabolism. Reducing oxalate and using thiazide diuretics help to treat calcium oxalate calculi. Administration of α-penicillamine and tiopronin prevent cystine crystallization for cystine calculi. Reducing intake of milk products to reduce calcium intake may be used with calcium calculi.

Priority Decision: Following electrohydraulic lithotripsy for treatment of renal calculi, the patient has a nursing diagnosis of risk for infection related to the introduction of bacteria following manipulation of the urinary tract. What is the most appropriate nursing intervention for this patient? a. Monitor for hematuria. b. Encourage fluid intake of 3 L/day. c. Apply moist heat to the flank area. d. Strain all urine through gauze or a special strainer.

b. A high fluid intake maintains dilute urine, which decreases bacterial concentration in addition to washing stone fragments and expected blood through the urinary system following lithotripsy. High urine output also prevents supersaturation of minerals. Moist heat to the flank may be helpful to relieve muscle spasms during renal colic and all urine should be strained in patients with renal stones to collect and identify stone composition but these are not related to infection.

What is the primary way that a nurse will evaluate the patency of an AVF? a. Palpate for pulses distal to the graft site. b. Auscultate for the presence of a bruit at the site. c. Evaluate the color and temperature of the extremity. d. Assess for the presence of numbness and tingling distal to the site.

b. A patent arteriovenous fistula (AVF) creates turbulent blood flow that can be assessed by listening for a bruit or palpated for a thrill as the blood passes through the graft. Assessment of neurovascular status in the extremity distal to the graft site is important to determine that the graft does not impair circulation to the extremity but the neurovascular status does not indicate whether the graft is open.

In a patient with AKI, which laboratory urinalysis result indicates tubular damage? a. Hematuria b. Specific gravity fixed at 1.010 c. Urine sodium of 12 mEq/L (12 mmol/L) d. Osmolality of 1000 mOsm/kg (1000 mmol/kg)

b. A urine specific gravity that is consistently 1.010 and a urine osmolality of about 300 mOsm/kg is the same specific gravity and osmolality as plasma. This indicates that tubules are damaged and unable to concentrate urine. Hematuria is more common with postrenal damage. Tubular damage is associated with a high sodium concentration (greater than 40 mEq/L).

In providing care for the patient with adult-onset polycystic kidney disease, what should the nurse do? a. Help the patient to cope with the rapid progression of the disease b. Suggest genetic counseling resources for the children of the patient c. Expect the patient to have polyuria and poor concentration ability of the kidneys d. Implement measures for the patient's deafness and blindness in addition to the renal problems

b. Adult-onset polycystic kidney disease is an inherited autosomal dominant disorder that often manifests after the patient has children but the children should receive genetic counseling regarding their life choices. The disease progresses slowly, eventually causing progressive renal failure. Hereditary medullary cystic disease causes poor concentration ability of the kidneys and classic Alport syndrome is a hereditary nephritis that is associated with deafness and deformities of the optic lens.

Health Acquired UTI

i. Urinary tract infections account for approximately 40 percent of all hospital-acquired infections annually, with fully 80 percent of these hospital-acquired urinary tract infections attributable to indwelling urethral catheters. It is well established that the duration of catheterization is directly related to risk for developing a urinary tract infection (UTI). With a catheter in place, the daily risk of developing a UTI ranges from 3 percent to 7 percent.

ST elevation

injury

A nurse is caring for a client with CHF whose EKG shows flattening of T wave? Which lab result should nurse anticipate? potassium 2.8 digitalis 2 hgb of 9.8 calcium of 8.0

potassium 2.8 flat t or development of u waves is indicative of low potassium

A nurse is caring for a client who had an MI 24 hr ago. At this time, it is essential for the nurse to look for which of the following? sepsis pericarditis ventricular dysrhytmias pulmonary emboli

ventricular dysrhythmias

A nurse is watching the cardiac monitor and notices that the rhythm suddenly changes. There are no P waves, the QRS complexes are wide, and the ventricular rate is regular but more than 100 beats/min. The nurse determines that the client is experiencing which of the following dysrhythmias?

vtach

How much morphine should you give? What route?

2mg 4mg or 6mg by weight given by IV Morphine for pain and stress

A client with anemia asks, "Why am I feeling tired all the time?" How does the nurse respond? 1. "How many hours are you sleeping at night?" 2. "You are not getting enough iron." 3. "You need to rest more when you are sick." 4. "Your cells are delivering less oxygen than you need."

4. The single most common symptom of anemia is fatigue. This problem occurs because oxygen delivery to cells is less than is required to meet normal oxygen needs.

Which of the following nursing diagnoses is a priority in the care of a patient with renal calculi? A Acute pain B Deficient fluid volume C Risk for constipation D Risk for powerlessness

A Urinary stones are associated with severe abdominal or flank pain. Deficient fluid volume is unlikely to result from urinary stones, whereas constipation is more likely to be an indirect consequence rather than a primary clinical manifestation of the problem. The presence of pain supersedes powerlessness as an immediate focus of nursing care.

Which of the following cardiovascular effects of aging should the nurse anticipate when providing care for older adults (select all that apply)? A) a. Arterial stiffening B) b. Increased blood pressure C) c. Increased maximal heart rate D) d. Decreased maximal heart rate E) e. Increased recovery time from activity

A,B,D,E

Which statement regarding continuous ambulatory peritoneal dialysis (CAPD) is of highest priority when teaching a patient new to this procedure? A. "It is essential that you maintain aseptic technique to prevent peritonitis." B. "You will be allowed a more liberal protein diet after you complete CAPD." C. "It is important for you to maintain a daily written record of blood pressure and weight." D. "You must continue regular medical and nursing follow-up visits while performing CAPD."

A. "It is essential that you maintain aseptic technique to prevent peritonitis." Peritonitis is a potentially fatal complication of peritoneal dialysis, and it is imperative to teach the patient methods to prevent it from occurring. Although the other teaching statements are accurate, they do not address the potential for mortality by peritonitis, making that nursing action the highest priority.

9) How does a nurse assess for dysrhythmias? A. 12 lead EKG B. Listen to lung sounds C. blood test D. Urine sample

A. 12 lead EKG

Blood pressure is regulated by what body systems? (select all that apply) A. Autonomic nervous system B. Heart C. Lungs D. Kidneys E. Hormones

A. Autonomic nervous system B. Heart D. Kidneys E. Hormones

T wave inversion on ECG indicates: A. myocardial ischemia B. myocardial injury C. myocardial infarction D. myocardial necrosis

A. myocardial ischemia lack of blood oxygen know the image

To hear the mitral murmur, you should place the bell of the stethoscope is placed over A. the apex of the heart B. the base of the heart C. 2nd right ISC D. 2nd left ISC

A. the apex of the heart

What can best help to decrease the incidence of sudden cardiac death in the community? A. Availability of automatic external defibrillators B. Courses in cardiopulmonary resuscitation (CPR) techniques for the general public C. Public service announcements about myocardial infarction (MI) symptoms. D. Mandatory CPR certification for all school teachers

ANS: A The most common cause of sudden cardiac death is the lethal dysrhythmia ventricular fibrillation, which usually occurs within the first 4 hours after the onset of pain. This lethal ventricular dysrhythmia must be treated immediately with defibrillation. Most patients with sudden cardiac death have not had an MI and have no cardiac symptoms. Reference: 793

The patient has a pacemaker set for 70 beats/minute. When taking the patient's pulse, you obtain a heart rate of 60 beats/minute. What is the best interpretation of this finding? A. The patient's heart has become more effective. B. The pacemaker is not working properly. C. The patient is tolerating a lower heart rate now. D. The pacemaker is sensing a ventricular rhythm.

ANS: B If the pacemaker is set for 70 beats/minute, a slower rate means that the pacemaker is not working properly and the patient's spontaneous rate has taken over. This situation needs to be evaluated. Reference: 836-837

You expect the patient with a mechanical prosthetic valve to have life-long drug management with A. aspirin. B. anticoagulants. C. antibiotic. D. antihypertensive agents.

ANS: B Long-term anticoagulation is recommended for all patients with mechanical valves and for those with biologic valves who have atrial fibrillation. Reference: 857

The most common finding in individuals at risk for sudden cardiac death is A. aortic valve disease. B. mitral valve disease. C. left ventricular dysfunction. D. atherosclerotic heart disease.

ANS: C Left ventricular dysfunction (ejection fraction <30%) and ventricular dysrhythmias after an MI are the strongest predictors of SCD. Reference: 793

If a patient cannot do the physical stress test, what are his options?

Angiogram, cardiac cath, PCI

A patient with a diagnosis of heart failure has been started on a nitroglycerin patch by his primary care provider. What should this patient be taught to avoid? A High-potassium foods B Drugs to treat erectile dysfunction C Nonsteroidal antiinflammatory drugs D Over-the-counter H2 -receptor blockers

B Drugs to treat erectile dysfunction The use of erectile drugs concurrent with nitrates creates a risk of severe hypotension and possibly death. High-potassium foods, NSAIDs, and H2-receptor blockers do not pose a risk in combination with nitrates.

If the Purkinje system is damaged, conduction of the electrical impulse is impaired through the a. atria b. AV node c. ventricles d. bundle of HIS

C

Which of the following nursing interventions should the nurse prioritize in the care of a 30-year-old woman who has a diagnosis of immune thrombocytopenic purpura (ITP)? A. Administration of packed red blood cells B. Administration of clotting factors VIII and IX C. Administration of oral or intravenous corticosteroids D. Maintenance of reverse isolation and application of standard precautions

C. Common treatment modalities for ITP include corticosteroid therapy to suppress the phagocytic response of splenic macrophages. Blood transfusions, administration of clotting factors, and reverse isolation are not interventions that are indicated in the care of patients with ITP.

After having an MI, the nurse notes the patient has jugular venous distention, gained weight, developed peripheral edema, and has a heart rate of 108/minute. What should the nurse suspect is happening? A ADHF B Chronic HF C Left-sided HF D Right-sided HF

D Right-sided HF An MI is a primary cause of heart failure. The jugular venous distention, weight gain, peripheral edema, and increased heart rate are manifestations of right-sided heart failure.

After noting a pulse deficit when assessing a patient who has just arrived in the emergency department, the nurse will anticipate that the patient may require A) a 2-D echocardiogram. B) a cardiac catheterization. C) hourly blood pressure (BP) checks. D) electrocardiographic (ECG) monitoring.

D) electrocardiographic (ECG) monitoring.

Etiology and pathophysiology of UTIs

-Urinary tract (anything) above urethra normally sterile -Defense mechanisms exist to maintain sterility/prevent UTIs. -Alteration in defense mechanisms increases risk of contracting UTI.

In teaching the hypertensive client to avoid orthostatic hypotension, the nurse should emphasize which of the following instructions? Select all that apply. 1. Plan regular times for taking medications. 2. Arise slowly from bed. 3. Avoid standing still for long periods. 4. Avoid excessive alcohol intake. 5. Avoid hot baths.

2, 3. Changing positions slowly and avoiding long periods of standing may limit the occurrence of orthostatic hypotension. Scheduling regular medication times is important for blood pressure management but this aspect is not related to the development of orthostatic hypotension. Excessive alcohol intake and hot baths are associated with vasodilation.

A client with anemia asks the nurse, "Do most people have the same number of red blood cells?" How does the nurse respond? 1."No, they don't." 2. "The number varies with gender, age, and general health." 3. "Yes, they do." 4. "You have fewer red blood cells because you have anemia."'

2. This is the most educational and reasonable response to the client's question. the first option is true, but not informative.

The most important long-term goal for a client with hypertension would be to: 1. Learn how to avoid stress. 2. Explore a job change or early retirement. 3. Make a commitment to long-term therapy. 4. Lose weight.

3. Compliance is the most critical element of hypertension therapy. In most cases, hypertensive clients require lifelong treatment and their hypertension cannot be managed successfully without drug therapy. Stress management is an important component of hypertension therapy, but the priority goal is related to compliance. It is not necessary for the client to change jobs or retire, but rather to learn to manage stress if the job is stressful. Losing weight may be necessary and will contribute to lower blood pressure, but the client must first accept the need for a lifelong management plan to control the hypertension.

A nurse is assessing an adult client's endurance in performing ADLs. What question does the nurse ask the client? 1."Can you prepare your own meals?" 2."Has your weight changed by 5 pounds or more this year?" 3."How is your energy level-compared with last year?" 4."What medications do you take daily, weekly, monthly?"

3. This question from Gordon's Functional Health Pattern Assessment is an activity exercise question that correctly assesses endurance compared with self-assessment in the past. It is most likely to provide data about the client's ability and endurance with ADLs. The client may never have been able to prepare his or her own meals. This question does not really address endurance.

Patient at GREATEST risk for a UTI

72-year-old woman hospitalized with a stroke who has a urinary catheter because of urinary incontinence.

A patient has a severe blockage in his right coronary artery. Which cardiac structure is most likely to be affected by this? a. AV node b. Left ventricle c. Coronary Sinus d. Pulmonary valve

A

A patient in asystole is likely to receive which of the following drug treatments? A) a. Atropine and epinephrine B) b. Lidocaine and amiodarone C) c. Digoxin and procainamide D) d. β-Adrenergic blockers and dopamine

A

A patient with newly diagnosed hypertension has a BP of 158/98 mm Hg after 6 months of exercise and diet modifications. Which management strategy will be a priority for this patient? a. medication will be required because the BP is still not at goal b. continued BP monitoring for another 3 months is all that will be necessary at this time c. lifestyle modifications are no longer important since they were not effective and medications will be started d. more vigorous changes in the patient's lifestyle are needed for a longer period of time before starting medications

A

When assessing a patient, you note a pulse deficit of 23 beats. This finding may be caused by a. dysrhythmias b. heart murmurs c. gallop rhythms d. pericardial friction rubs

A

19. An outpatient who has developed heart failure after having an acute myocardial infarction has a new prescription for carvedilol (Coreg). After 2 weeks, the patient returns to the clinic. The assessment finding that will be of most concern to the nurse is that the patient a. has BP of 88/42. b. has an apical pulse rate of 56. c. complains of feeling tired. d. has 2+ pedal edema.

A Rationale: The patient's BP indicates that the dose of carvedilol may need to be decreased because the mean arterial pressure is only 57. Bradycardia is a frequent adverse effect of -Adrenergic blockade, but the rate of 56 is not as great a concern as the hypotension. -adrenergic blockade will initially worsen symptoms of heart failure in many patients, and patients should be taught that some increase in symptoms, such as fatigue and edema, is expected during the initiation of therapy with this class of drugs. Cognitive Level: Application Text Reference: p. 832 Nursing Process: Assessment NCLEX: Analysis

A patient with a recent diagnosis of heart failure has been prescribed furosemide (Lasix) in an effort to physiologically do what for the patient? A Reduce preload. B Decrease afterload. C Increase contractility. D Promote vasodilation.

A Reduce preload. Diuretics such as furosemide are used in the treatment of HF to mobilize edematous fluid, reduce pulmonary venous pressure, and reduce preload. They do not directly influence afterload, contractility, or vessel tone.

A 19-year-old has a mandatory electrocardiogram (ECG) before participating on a college swim team and is found to have sinus bradycardia, rate 52. BP is 114/54, and the student denies any health problems. What action by the nurse is appropriate? A) Allow the student to participate on the swim team. B) Refer the student to a cardiologist for further assessment. C) Obtain more detailed information about the student's health history. D) Tell the student to stop swimming immediately if any dyspnea occurs.

A) Allow the student to participate on the swim team. In an aerobically trained individual, sinus bradycardia is normal. The student's normal BPand negative health history indicate that there is no need for a cardiology referral or for more detailed information about the health history. Dyspnea during an aerobic activitysuch as swimming is normal.

Which action will the nurse include in the plan of care when caring for a patient admitted with acute decompensated heart failure (ADHF) who is receiving nesiritide (Natrecor)? A) a. Monitor blood pressure frequently. B) b. Encourage patient to ambulate in room. C) c. Titrate nesiritide rate slowly before discontinuing. D) d. Teach patient about safe home use of the medication.

A) Monitor blood pressure frequently.

To determine whether there is a delay in impulse conduction through the atria, the nurse will measure the length of the patient's A) P wave. B) PR interval. C) QT interval. D) QRS complex.

A) P wave. The P wave represents the depolarization of the atria. The P-R interval represents depolarization of the atria, atrioventricular (AV) node, bundle of His, bundle branches, and the Purkinje fibers. The QRS represents ventricular depolarization. The Q-T interval represents depolarization and repolarization of the entire conduction system.

How do you differentiate chest pain of cardiac origin from chest pain of non-cardiac origin based on clinical presentation? A. Cardiac chest pain is constant. B. Cardiac chest pain increases with inspiration.

A. Cardiac chest pain is constant. Pulmonary origin increases with inspiration

The presence of JVD indicates A. High venous pressure B. Low venous pressure C. High systemic resistance D. Low systemic resistance

A. High venous pressure

The chest pain in Mr. Cameron's case is caused by A. O2 demand > O2 supply B. O2 supply > O2 demand C. coronary spasm D. high serum CO2 level

A. O2 demand > O2 supply Produces lactic acid and causes chest pain

For the patient with AKI, which laboratory result would cause you the greatest concern? A. Potassium level of 5.9 mEq/L B. BUN level of 25 mg/dL C. Sodium level of 144 mEq/L D. pH of 7.5

A. Potassium level of 5.9 mEq/L Hyperkalemia is one of the most serious complications in AKI because it can cause life-threatening cardiac dysrhythmias.

What is the major pathology of AMI and what is the goal of treatment?

AMI - acute myocardial infraction (heart attack) The goal is to save the heart cells, minimize necrosis

The nurse is caring for a patient who had kidney transplantation several years ago. Which assessment finding may indicate that the patient is experiencing adverse effects to the prescribed corticosteroid? a. Joint pain b. Tachycardia c. Postural hypotension d. Increase in creatinine level

ANS: A Aseptic necrosis of the weight-bearing joints can occur when patients take corticosteroids over a prolonged period. Increased creatinine level, orthostatic dizziness, and tachycardia are not caused by corticosteroid use.

Hypoperfusion to the kidneys results in the release of A. catecholamines. B. renin. C. bradykinin. D. aldosterone.

ANS: B As the cardiac output falls, blood flow to the kidneys decreases. This is sensed by the juxtaglomerular apparatus in the kidney as decreased volume. In response, the kidneys release renin, which converts angiotensinogen to angiotensin I. Angiotensin I is subsequently converted to angiotensin II by a converting enzyme made in the lungs. Angiotensin II causes the adrenal cortex to release aldosterone, which results in sodium and water retention, and causes increased peripheral vasoconstriction, which increases blood pressure. Reference: 799

What should you teach patients with chronic stable angina? A. Do isometric arm exercises to build endurance. B. Wear a face mask when outdoors in cold weather. C. Drive to an emergency department if there is no relief after three nitroglycerin tablets. D. Notify a physician if you experience headache with nitroglycerin.

ANS: B Blood vessels constrict in response to cold and increase the workload of the heart. Isometric exercises can cause exertional anginal. Activity should focus on cardiac health, such as walking. Emergency medical services (EMS) should be called if there is no relief after three nitroglycerine tablets. Driving can be dangerous if a myocardial infarction occurs. Vasodilation occurs as a side effect of nitroglycerin. Reference: 773

After the insertion of an arteriovenous graft (AVG) in the right forearm, a patient complains of pain and coldness of the right fingers. Which action should the nurse take? a. Elevate the patient's arm above the level of the heart. b. Report the patient's symptoms to the health care provider. c. Remind the patient about the need to take a daily low-dose aspirin tablet. d. Educate the patient about the normal vascular response after AVG insertion.

ANS: B The patient's complaints suggest the development of distal ischemia (steal syndrome) and may require revision of the AVG. Elevation of the arm above the heart will decrease perfusion. Pain and coolness are not normal after AVG insertion. Aspirin therapy is not used to maintain grafts.

The RN observes an LPN/LVN carrying out all of the following actions while caring for a patient with stage 2 chronic kidney disease. Which action requires the RN to intervene? a. The LPN/LVN administers erythropoietin subcutaneously. b. The LPN/LVN assists the patient to ambulate in the hallway. c. The LPN/LVN gives the iron supplement and phosphate binder with lunch. d. The LPN/LVN carries a tray containing low-protein foods into the patient's room.

ANS: C Oral phosphate binders should not be given at the same time as iron because they prevent the iron from being absorbed. The phosphate binder should be given with a meal and the iron given at a different time. The other actions by the LPN/LVN are appropriate for a patient with renal insufficiency.

If the Purkinje system is damaged, conduction of the electrical impulse is impaired through the A. atria. B. AV node. C. ventricles. D. bundle of His.

ANS: C The action potential of the electrical impulse diffuses widely through the walls of both ventricles by means of Purkinje fibers. Reference: 716

Which information about a patient who was admitted 10 days previously with acute kidney injury (AKI) caused by dehydration will be most important for the nurse to report to the health care provider? a. The blood urea nitrogen (BUN) level is 67 mg/dL. b. The creatinine level is 3.0 mg/dL. c. Urine output over an 8-hour period is 2500 mL. d. The glomerular filtration rate is <30 mL/min/1.73m2.

ANS: C The high urine output indicates a need to increase fluid intake to prevent hypovolemia. The other information is typical of AKI and will not require a change in therapy

During hemodialysis, a patient complains of nausea and dizziness. Which action should the nurse take first? a. Slow down the rate of dialysis. b. Obtain blood to check the blood urea nitrogen (BUN) level. c. Check the patient's blood pressure. d. Give prescribed PRN antiemetic drugs.

ANS: C The patient's complaints of nausea and dizziness suggest hypotension, so the initial action should be to check the BP. The other actions also may be appropriate, based on the blood pressure obtained.

A patient in the oliguric phase of acute renal failure has a 24-hour fluid output of 150 mL emesis and 250 mL urine. The nurse plans a fluid replacement for the following day of ___ mL. a. 400 b. 800 c. 1000 d. 1400

ANS: C Usually fluid replacement should be based on the patient's measured output plus 600 mL/day for insensible losses.

A patient complains of leg cramps during hemodialysis. The nurse should first a. reposition the patient. b. massage the patient's legs. c. give acetaminophen (Tylenol). d. infuse a bolus of normal saline.

ANS: D Muscle cramps during dialysis are caused by rapid removal of sodium and water. Treatment includes infusion of normal saline. The other actions do not address the reason for the cramps.

The nurse would expect which of the following to be the first choice to treat a stable patient with ventricular tachycardia?

Amiodarone

A P wave on an ECG represents an impulse a. arising at the SA node and repolarizing the atria b. arising at the SA node and depolarizing the atria c. arising at the AV node and depolarizing the atria d. arising at the AV node and spreading to the bundle of HIS

B

Which of the following urinalysis results would the nurse recognize as an abnormal finding? A pH 6.0 B White blood cells (WBCs) 9/hpf C Amber yellow color D Specific gravity 1.025

B Normal WBC levels in urine are below 5/hpf, with levels exceeding this indicative of inflammation or urinary tract infection. Amber yellow is normal coloration, whereas a pH of 6.0 is average. Reference ranges for specific gravity are 1.003 to 1.030.

The nurse is preparing to administer digoxin to a patient with heart failure. In preparation, laboratory results are reviewed with the following findings: sodium 139 mEq/L, potassium 5.6 mEq/L, chloride 103 mEq/L, and glucose 106 mg/dL. What should the nurse do next? A Withhold the daily dose until the following day. B Withhold the dose and report the potassium level. C Give the digoxin with a salty snack, such as crackers. D Give the digoxin with extra fluids to dilute the sodium level.

B Withhold the dose and report the potassium level The normal potassium level is 3.5 to 5.0 mEq/L. The patient is hyperkalemic, which makes the patient more prone to digoxin toxicity. For this reason, the nurse should withhold the dose and report the potassium level. The physician may order the digoxin to be given once the potassium level has been treated and decreases to within normal range.

A patient with a history of end-stage renal disease (ESRD) resulting from diabetes mellitus has presented to the outpatient dialysis unit for his scheduled hemodialysis. Which assessment should you prioritize before, during, and after his treatment? A. Level of consciousness B. Blood pressure and fluid balance C. Temperature, heart rate, and blood pressure D. Assessment for signs and symptoms of infection

B. Blood pressure and fluid balance Although all of the assessments are relevant to the care of a patient receiving hemodialysis, the nature of the procedure indicates a particular need to monitor blood pressure and fluid balance.

Which assessment finding is commonly found in the oliguric phase of acute kidney injury (AKI)? A. Hypovolemia B. Hyperkalemia C. Hypernatremia D. Thrombocytopenia

B. Hyperkalemia In AKI, the serum potassium levels increase because the normal ability of the kidneys to excrete potassium is impaired. Sodium levels are typically normal or diminished, whereas fluid volume is normally increased due to decreased urine output. Thrombocytopenia is not a consequence of AKI, although altered platelet function may occur in AKI.

What classification of hypertension does J. G. have? His BP 172/94 A. Stage 1 hypertension B. Stage 2 hypertension C. Anginal hypertension D. Hypertensive Crisis

B. Stage 2 hypertension

D

Because myelodysplastic syndrome arises from pluripotent hematopoietic stem cells in the bone marrow, laboratory results the nurse would expect to find include: A. an excess of T cells B. an excess of platelets C. a deficiency of granulocytes D. a deficiency of all cellular blood components

The healthcare provider is performing an assessment on a patient who is taking propranolol (Inderal) for supraventricular tachycardia. Which assessment finding is an indication the patient is experiencing an adverse effect of this drug? Dry mouth Bradycardia Urinary retention Paresthesia

Bradycardia

The nurse is monitoring the ECG of a patient admitted with ACS. Which of the following ECG characteristics would be most suggestive of ischemia? a. sinus rhythm with a pathologic Q wave b. sinus rhythm with an elevated ST segment c. Sinus rhythm with a depressed ST segment d. sinus rhythm with premature atrial contractions

C

What is a trade name of Nitroglycerin? A.Nitrol B.Nirtrolingual C. NitroQuick D. All of the above

D. All of the above

10) A patient with potential heart failure enters the emergency room. What symptom should the nurse not consider for heart failure? A. cyanosis, cold and clammy skin B. lung sounds-- crackling and wheezing C. orthopnea, shortness of breath D. tightness & burning from the chest

D. tightness & burning from the chest

C

DIC is a disorder in which: A. the coagulation pathway is generally altered, leading to thrombus formation in all major blood vessels B. an underlying disease depletes hemolytic factors in the blood, leading to diffuse thrombotic episodes and infarcts C. a disease process stimulates coagulation process with resultant thrombosis, as well as depletion of clotting factors, leading to diffuse clotting and hemorrhage D. an inherited predisposition causes a deficiency of clotting factors that leads to overstimulation of coagulation processes in the vasculature

The nurse knows that which possible side effects or adverse reactions are important to consider when initiating treatment of hypertension using ACE-inhibitors or ARB's? A. ACE-inhibitors and ARB's can cause angioedema. B. Chronic cough may be related to hypertensive management with ACE-inhibitors. C. ACE is less effective in African Americans D. ACE and ARB can potentiate hyperkalemia especially in patients with renal impairment E. All of the above

E. All of the above

The client has a diagnosis of junctional rhythm. The nurse would expect to find which of the following during the assessment of vital signs? Heart rate <30 Heart rate 40-60 Heart rate 60-100 Heart rate 100-120

HR 40-60

A patient who has experienced atrial fibrillation for the past 3 days is admitted to the cardiac care unit. In addition to administering an antidysrhythmia medication, the healthcare provider should anticipate which of these orders? Initiate a heparin infusion Give atropine IV push Prepare for immediate cardioversion Prepare the patient for AV node ablation

Heparin Because blood tends to pool and clot in the fibrillating atria, patients with atrial fibrillation are at high risk for embolic stroke, so heparin will be given.

A

In severely anemic patients, the nurse would expect to find: A. dyspnea and tachycardia B. cyanosis and pulmonary edema C. cardiomegaly and pulmonary fibrosis D. ventricular dysrhythmia and wheezing

Classify the clinical manifestations of right-sided vs. left-sided HF.

Left: Pulmonary congestion Right: JVD, peripheral edema

D

Multiple drugs are often used in combination to treat leukemia and lymphoma because: A. there are fewer toxic side effects B. the chance that one drug will be effective is increased C. the drugs work more effectively without causing side effects D. the drugs work by different mechanisms to maximize killing of malignant cells

Before you give any nitroglycerin, what should you assess for?

Nitroglycerin is given sublingual or nasal spray Relieves pain in 3 mins with a duration of 60-90 mins One dose = 0.4mg/1 tablet One dose every 5 mins No more than 3 tablets It's a direct vasodilator Do not store in light, sensitive Expires in 6 months after opening Test for effectiveness by noticing tingling sensation Assess for viagra use (also BP and HR) More info on Nitroglycerin: https://sites.google.com/site/pharmacology2site/week-3-digoxin-nitroglycerin/digoxin-nitroglycerin-for-group-6/nitroglycerin

Is every chest pain is of a cardiac origin? Yes No

No Can be from pulmonary

A, B

Nursing interventions for a patient with severe anemia related to peptic ulcer disease would include (select all that apply): A. monitoring stools for guaiac B. instruction for a high iron diet C. taking vital signs every 8 hours D. teaching self injection of erythropoietin E. administration of cobalamin (vit B12) injections

When caring for a patient with a cardiac dysrhythmia, which laboratory value is a priority for the healthcare provider to monitor? BUN and creatinine Sodium, potassium, and calcium Hemoglobin and hematocrit PT and INR

Sodium, potassium, and chloride

Mr. Cameron indicated that his pain is in the middle of his chest and radiates to his left arm. It is about 6/10. He added that this was not the first time he had this pain. He said that the pain usually starts few minutes after gardening or climbing stairs and is relieved by rest. Angina is suspected. What type of angina did Mr. Cameron just describe?

Stable angina Stable can be predicted, like after exercise Unstable is unpredictable

What are the treatment options of AMI?

TPA - dissolve blood clots (initiate with 30 mins) Emergency PCI - to restart repurfusion (initiate with 90 mins) Contradictions for TPA - blood thinners, or bleeding disorders

Atrial fibrillation is a dysrhythmia with irregular rhythm. True or False

TRUE

D

The most common type of leukemia in older adults is: A. acute myelocytic leukemia B. acute lymphocytic leukemia C. chronic myelocytic leukemia D. chronic lymphocytic leukemia

C

The nurse is aware that a major difference between Hodgkin's lymphoma and non-Hodgkin's lymphoma is: A. Hodgkin's lymphoma only occurs in young adults B. Hodgkin's lymphoma is considered potentially curable C. non-Hodgkin's lymphoma can manifest in multiple organs D. non-Hodgkin's lymphoma is treated only with radiation therapy

B

When obtaining assessment data from a patient with a microcytic, hypochromic anemia, the nurse would question the patient about: A. folic acid intake B. dietary intake of iron C. a history of gastric surgery D. a history of sickle cell anemia

A

When providing care for a patient with thrombocyopenia, the nurse instructs the patient to: A. dab his or her nose instead of blowing B. be careful when shaving with a safety razor C. continue with physical activities to stimulate thrombopoiesis D. avoid t aspirin because it may mask the fever that occurs with thrombocytopenia

Which drugs will be used to treat the patient with CKD for mineral and bone disorder (select all that apply)? a. Cinacalcet (Sensipar) b. Sevelamer (Renagel) c. IV glucose and insulin d. Calcium acetate (PhosLo) e. IV 10% calcium gluconate

a, b, d. Cinacalcet (Sensipar), a calcimimetic agent to control secondary hyperparathyroidism; sevelamer (Renagel), a noncalcium phosphate binder; and calcium acetate (PhosLo), a calcium-based phosphate binder are used to treat mineral and bone disorder in CKD. IV glucose and insulin and IV 10% calcium gluconate along with sodium polystyrene sulfonate (Kayexalate) are used to treat the hyperkalemia of CKD.

Number in sequence the following ascending pathologic changes that occur in the urinary tract in the presence of a bladder outlet obstruction. a. Hydronephrosis b. Reflux of urine into ureter c. Bladder detrusor muscle hypertrophy d. Ureteral dilation e. Renal atrophy f. Vesicoureteral reflux g. Large residual urine in bladder h. Chronic pyelonephritis

a. 6; b. 3; c. 1; d. 4; e. 8; f. 5; g. 2; h. 7

Nursing Diagnosis of Acute Pyelonephritis

a. Acute pain b. Impaired urinary elimination c. Knowledge deficit

Which characteristic is more likely with acute pyelonephritis than with a lower UTI? a. Fever b. Dysuria c. Urgency d. Frequency

a. Systemic manifestations of fever and chills with leukocytosis and nausea and vomiting are more common in pyelonephritis than in a lower UTI. Dysuria, frequency, and urgency can be present with both.

*6. A patient is admitted to the hospital with severe renal colic. The nurse's first priority in management of the patient is to* a. administer opioids as prescribed. b. obtain supplies for straining all urine c. encourage fluid intake of 3-4L/day d. keep the patient NPO in preparation for surgery

a. administer opioids as prescribed.

Non-modifiable risks for CV

age gender family history

Risk factors for CAD

age, lifestyle, genetics

When a patient has long-term atrial fibrillation, the nurse would expect to include which drug in the plan of care to minimize the greatest risk that is commonly associated with atrial fibrillation?

anticoag greatest risk if thrombi or emboli

A patient who is on the telemetry unit develops atrial flutter, rate 150, with associateddyspnea and diaphoresis. Which of these actions that are included in the hospitaldysrhythmia protocol should the nurse take first? a. Obtain a 12-lead electrocardiogram (ECG). b. Give O2 at 3 to 4 L/min. c. Take the patient's blood pressure and respiratory rate. d. Notify the health care provider of the change in rhythm

b. Give O2 at 3 to 4 L/min. Since this patient has dyspnea in association with the new rhythm, the nurse's initialactions should be to address the patient's airway, breathing, and circulation (ABC) byoxygen administration. The other actions also are important and should be implementedrapidly.

The patient with CKD asks why she is receiving nifedipine (Procardia) and furosemide (Lasix). The nurse understands that these drugs are being used to treat the patient's a. anemia. b. hypertension. c. hyperkalemia. d. mineral and bone disorder.

b. Nifedipine (Procardia) is a calcium channel blocker and furosemide (Lasix) is a loop diuretic. Both are used to treat hypertension.

*10. A patient with ureterolithotomy returns from surgery with a nephrostomy tube in place. Postoperative nursing care of the patient includes:* a.encourage the patient to drink fruit juices and milk b.encouraging fluids of at least 2-3 L/day after nausea has subsided c. irrigating the nephrostomy tube with 10ml of NS solution as needed d. notifying the physician if nephrostomy tube drainage is more than 30ml/hr

b.encouraging fluids of at least 2-3 L/day after nausea has subsided

*8. The nurse identifies a risk factor for kidney and bladder cancer in a patient who relates a history of* a.aspirin use b.tobacco use c.chronic alcohol abuse d.use of artificial sweeteners

b.tobacco use

Which of the following classes of drugs is most widely used in the treatment of cardiomyopathy?

beta-adrenergic blockers

An 83-year-old female patient was found lying on the bathroom floor. She said she fell 2 days ago and has not been able to take her heart medicine or eat or drink anything since then. What conditions could be causing prerenal AKI in this patient (select all that apply)? a. Anaphylaxis b. Renal calculi c. Hypovolemia d. Nephrotoxic drugs e. Decreased cardiac output

c, e. Because the patient has had nothing to eat or drink for 2 days, she is probably dehydrated and hypovolemic. Decreased cardiac output (CO) is most likely because she is older and takes heart medicine, which is probably for heart failure or hypertension. Both hypovolemia and decreased CO cause prerenal AKI. Anaphylaxis is also a cause of prerenal AKI but is not likely in this situation. Nephrotoxic drugs would contribute to intrarenal causes of AKI and renal calculi would be a postrenal cause of AKI.

What indicates to the nurse that a patient with AKI is in the recovery phase? a. A return to normal weight b. A urine output of 3700 mL/day c. Decreasing sodium and potassium levels d. Decreasing blood urea nitrogen (BUN) and creatinine levels

d. The blood urea nitrogen (BUN) and creatinine levels remain high during the oliguric and diuretic phases of AKI. The recovery phase begins when the glomerular filtration returns to a rate at which BUN and creatinine stabilize and then decrease. Urinary output of 3 to 5 L/ day, decreasing sodium and potassium levels, and fluid weight loss are characteristic of the diuretic phase of AKI.

*11. A patient has has a cystectomy and ileal conduit diversion performed. Four days postoperatively, mucous shred are seen in the drainage bag. The nurse should.* a. notify the physician b.notify the charge nurse c. irrigate the drainage tube d. chart it as a normal observation

d. chart it as a normal observation

When do coronary arteries primarily receive blood flow?

during diastole

Which of the following arteries primarily feeds the anterior wall of the heart?

left anterior descending artery

Pathological Q wave

means necrosis

Modifiable risks for CV

smoking exercise hypertension

Defense mechanisms to maintain sterility

-Complete emptying of bladder -Ureterovesical junction competence -Peristaltic activity: patients with bowels could also have urinary problems -Acidic pH -High urea concentration -Abundant glycoproteins

Alteration in defense mechanisms increasing risk of contracting UTI

-Predisposing factors: Factors increasing urinary stasis, Examples: BPH (benign prostatic hypertrophy)—prostate enlarged are cuts off ureter, tumor, neurogenic bladder (bladder not emptying correctly); Foreign bodies, Examples: Catheters, calculi, instrumentation; Anatomic factors, Examples: Obesity, congenital defects, fistula; Compromising immune response factors (immune system), Examples: Age, HIV, diabetes; Functional disorders, Example: Constipation; Other factors (examples: Pregnancy, multiple sex partners (women); Menopause factor in incidence of UTI: Postmenopausal women have lower estrogen levels, ↓ in vaginal lactobacilli (which protects bladder), ↑ in vaginal pH, Overgrowth of other organisms results, Low-dose intravaginal estrogen replacement may be effective in treating recurrent UTIs.

Symptoms of a UTI

-Urinary urgency, frequency, suprapubic discomfort, -Urine appearance sediment, cloudy, bloody, -If flank, pain and fever, may have progressed to pyelonephritis -May have fatigue and/or anorexia -Elderly may NOT run fever with UTI and may not have typical or any symptoms!

A client is taking clonidine (Catapres) for treatment of hypertension. The nurse should teach the client about which of the following common adverse effects of this drug? Select all that apply. 1. Dry mouth. 2. Hyperkalemia. 3. Impotence. 4. Pancreatitis. 5. Sleep disturbance.

1, 3, 5. Clonidine (Catapres) is a central-acting adrenergic antagonist. It reduces sympathetic outflow from the central nervous system. Dry mouth, impotence, and sleep disturbances are possible adverse effects. Hyperkalemia and pancreatitis are not anticipated with use of this drug.

An industrial health nurse at a large printing plant finds a male employee's blood pressure to be elevated on two occasions 1 month apart and refers him to his private physician. The employee is about 25 lb overweight and has smoked a pack of cigarettes daily for more than 20 years. The client's physician prescribes atenolol (Tenormin) for the hypertension. The nurse should instruct the client to: 1. Avoid sudden discontinuation of the drug. 2. Monitor the blood pressure annually. 3. Follow a 2-g sodium diet. 4. Discontinue the medication if severe headaches develop.

1. Atenolol is a beta-adrenergic antagonist indicated for management of hypertension. Sudden discontinuation of this drug is dangerous because it may exacerbate symptoms. The medication should not be discontinued without a physician's order. Blood pressure needs to be monitored more frequently than annually in a client who is newly diagnosed and treated for hypertension. Clients are not usually placed on a 2-g sodium diet for hypertension.

Metoprolol (Toprol XL) is added to the pharmacologic therapy of a diabetic female diagnosed with stage 2 hypertension initially treated with Furosemide (Lasix) and Ramipril (Altace). An expected therapeutic effect is: 1. Decrease in heart rate. 2. Lessening of fatigue. 3. Improvement in blood sugar levels. 4. Increase in urine output.

1. The effect of a beta blocker is a decrease in heart rate, contractility, and afterload, which leads to a decrease in blood pressure. The client at first may have an increase in fatigue when starting the beta blocker. The mechanism of action does not improve blood sugar or urine output.

A patient with a tricuspid valve disorder will have impaired blood flow between the: a. vena cave and right atrium b. left atrium and left ventricle c. right atrium and right ventricle d. right ventricle and pulmonary artery

C

Which BP regulating mechanism(s) can result in the development of hypertension if defective (select all that apply)? a. release of norepinephrine b. secretion of prostaglandins c. stimulation of the sympathetic nervous system d. stimulation of the parasympathetic nervous system e. activation of the renin-angiotensin-aldosterone system

A,C,E

The optimal time from harvest of a heart until it is transplanted into the recipient should be A. less than 4 hours. B. less than 8 hours. C. less than 10 hours. D. less than 12 hours.

ANS: A Harvested organs are transported on ice until they can be implanted. For the heart, this period is optimally less than 4 hours. The donor heart often is flown to the recipient's hospital. Reference: 815

A client has a bone marrow biopsy done. Which nursing intervention is the priority postprocedure? 1.Applying pressure to the biopsy site 2. Inspecting the site for ecchymoses 3. Sending the biopsy specimens to the laboratory 4. Teaching the client about avoiding vigorous activity

1. The initial action should be to stop bleeding by applying pressure to the site. Inspecting for ecchymoses will be done after hemostasis has been achieved. Sending specimens to the laboratory will be done after hemostasis has been achieved. Teaching the client about activity levels will be done after hemostasis has been achieved.

Managing a UTI

1. Wipe from front to back, drink 6-8 glasses of water a day, high fluid intake (helps dilute urine and urinate more frequently), empty bladder soon after intercourse, avoid caffeine, alcohol, citrus juices, chocolate, spicy foods and products that irritate perineal areas 2. Not too much good solid evidence on preventing multiple UTIs 3. Probiotics—no evidence exists to show this really is effective 4. Cranberry juice may help women with UTIs but there is no evidence to show that it prevents UTIs 5. Antibiotics prophylaxis, self-starting antibiotics

Correct order of MONA...

1st Oxygen 2nd Aspin 3rd Nitroglycerin 4th Morphine

A nurse is assessing a client for hematologic function risks. The nurse seeks to determine whether there is a risk that cannot be reduced or eliminated. Which clinical health history question does the nurse ask to obtain this information? 1."Do you seem to have excessive bleeding or bruising?" 2. "Does anyone in your family bleed a lot?" 3. "Tell me what you eat in a day." 4. "Where do you work?"

2. An accurate family history is important because many disorders that affect blood and blood clotting are inherited. Genetics cannot be changed. Excessive bleeding or bruising is a symptom, not a risk.

A newly admitted client has an elevated reticulocyte count. Which disorder does the nurse suspect in this client? 1. Aplastic anemia 2. Hemolytic anemia 3. Infectious process 4. Leukemia

2. An elevated reticulocyte count in the anemic client indicates that the bone marrow is responding appropriately to a decrease in the total red blood cell (RBC) mass and is prematurely destroying red blood cells. Therefore more immature RBCs are in circulation. Aplastic anemia is associated with a low reticulocyte count. A high white blood cell count is expected in clients with infection, while a low WBC is expected in clients with leukemia

A nurse is reviewing complete blood count (CBC) data for a 76-year-old client. Which decreased value causes concern because it is not age related? 1.Hemoglobin (Hgb) level 2. Platelet (thrombocyte) count 3. Red blood cell (RBC) count 4. White blood cell (WBC) response

2. Platelet counts do not generally change with age. Hemoglobin levels in men and women fall after middle age. Iron-deficient diets may play a role in this reduction. Total red blood cell (RBC) and white blood cell (WBC) counts (especially lymphocyte counts) are lower in older adults. The WBC count does not rise as high in response to infection in older adults as it does in younger people.

The nurse teaches a client, who has recently been diagnosed with hypertension, about dietary restrictions: a low-calorie, low-fat, low-sodium diet. Which of the following menu selections would best meet the client's needs? 1. Mixed green salad with blue cheese dressing, crackers, and cold cuts. 2. Ham sandwich on rye bread and an orange. 3. Baked chicken, an apple, and a slice of white bread. 4. Hot dogs, baked beans, and celery and carrot sticks.

3. Processed and cured meat products, such as cold cuts, ham, and hot dogs, are all high in both fat and sodium and should be avoided on a low-calorie, low-fat, low-salt diet. Dietary restrictions of all types are complex and difficult to implement with clients who are basically asymptomatic.

A client on anticoagulant therapy is being discharged. Which statement indicates that the client has a correct understanding of this therapy's purpose or action? 1. "It is to dissolve blood clots." 2. "It might cause me to get injured more often." 3. "It should prevent my blood from clotting." 4. "It will thin my blood."

3. Anticoagulants work by interfering with one or more steps involved in the blood clotting cascade. Thus, these agents prevent new clots from forming and limit or prevent extension of formed clots. Anticoagulants do not cause any change in the thickness or viscosity of the blood. Anticoagulants do not cause more injuries but may cause more bleeding and bruising when someone is injured. Anticoagulants do not dissolve clots, rather fibrinolytics do.

Which nursing action does the RN delegate to unlicensed assistive personnel (UAP) who are assisting with the care of a female client with anemia? 1. Asking the client about the amount of blood loss with each menstrual period 2. Checking for sternal tenderness while applying fingertip pressure 3. Determining the respiratory rate before and after the client walks 20 feet 4. Monitoring her oral mucosa for pallor, bleeding, or ulceration

3. Assessment of respiratory rate before and after ambulation is within the scope of practice for UAP. UAP will report this information to the RN. Asking the client about the amount of blood loss with each menstrual period requires skilled assessment techniques and knowledge of normal parameters and should be done by the RN. Checking for sternal tenderness requires skilled assessment techniques and knowledge of normal parameters and should be done by the RN. Monitoring of oral mucosa requires skilled assessment techniques and knowledge of normal parameters and should be done by the RN.

A client with a low platelet count asks why platelets are important. How does the nurse answer? 1."Platelets make blood clots for you." 2. "Blood clotting is prevented by your platelets." 3. "The clotting process begins with your platelets." 4. "Your platelets finish the clotting process."

3. Platelets begin the blood clotting process by forming platelet plugs, but these platelet plugs are not clots and cannot provide complete hemostasis. Platelets do not clot blood. They are a part of the clotting process or cascade of coagulation.

A clinic nurse is discharging a 20-year-old client who had a bone marrow aspiration performed. What does the nurse advise the client to do? 1. "Avoid contact sports or activity that may traumatize the site for 24 hours." 2. "Inspect the site for bleeding every 4 to 6 hours." 3. "Place an ice pack over the site to reduce the bruising." 4. "Take a mild analgesic, such as 2 aspirin, for pain or discomfort at the site."

3. Ice to the site will help limit bruising and tissue damage during the first 24 hours postprocedure. Contact sports and traumatic activity needs to be excluded for 48 hours, or 2 days.The site should be carefully monitored by the client every 2 hours for the first 24 hours following the procedure.A mild analgesic is appropriate, but it should be aspirin free. Acetaminophen (Tylenol) would be a good choice.

An exercise program is prescribed for the client with hypertension. Which intervention would be most likely to assist the client in maintaining an exercise program? 1. Giving the client a written exercise program. 2. Explaining the exercise program to the client's spouse. 3. Reassuring the client that he or she can do the exercise program. 4. Tailoring a program to the client's needs and abilities.

4. Tailoring or individualizing a program to the client's lifestyle has been shown to be an effective strategy for changing health behaviors. Providing a written program, explaining the program to the client's spouse, and reassuring the client that he or she can do the program may be helpful but are not as likely to promote adherence as individualizing the program.

What diagnostic test should you do first (chest pain)? A. ECG B. Serum troponin C. Chest x-ray D. Echocardiogram

A. ECG ECG and EKG are the same thing Serum troponin is done second - elevated during myocardial injury. Troponin is specific and sensitive to diagnose MI. (could also be of renal origin). Normal range 0.0-0.4.

Which of the following types of cardiomyopathy can be associated with childbirth?

dilated

Which action does the RN delegate to unlicensed assistive personnel (UAP)? 1. Drawing a partial thromboplastin time (PTT) from a saline lock on a client with a pulmonary embolism 2. Performing a capillary fragility test to check vascular hemostatic function on a client with liver failure 3. Referring for counseling a client with a daily alcohol consumption of 12 beers a day 4. Reporting any bleeding noted when catheter care is given to a client with a history of hemophilia

4. Reporting findings during routine care is expected and required of unlicensed staff members. Drawing a partial thromboplastin time is more complex and should be done by licensed nursing staff. Performing a capillary fragility test is more complex and should be done by licensed nursing staff. Referring a client for alcohol counseling is more complex and should be done by licensed nursing staff.

A nurse is starting the shift by making rounds. Which client does the nurse decide to assess first? 1.42-year-old with anemia who is reporting shortness of breath when ambulating down the hallway 2. 47-year-old who recently had a Rumpel-Leede test and is requesting a nurse to "look at the bruises on my arm" 3. 52-year-old who has just had a bone marrow aspiration and is requesting pain medication 4. 59-year old who has a nosebleed and is receiving heparin to treat a pulmonary embolism

4. This client may be experiencing the bleeding as a result of excessive anticoagulation and should be assessed for the severity of the situation before the other clients, whose conditions are stable, are assessed. regarding option number 3, Making clients wait for pain medication is not desirable, but in this scenario, the client who is bleeding is the higher priority. This client should be next on the nurse's "to do" list.

After teaching about ways to decrease risk factors for CAD, the nurse recognizes that additional instruction is needed when the patient says, a. I would like to add weight lifting to my exercise program b. I can only keep my blood pressure with medication c. I can change my diet to decrease my intake of saturated fats d. I will change my lifestyle to reduce activities that increase my stress

A

The nurse teaches a patient with hypertension that uncontrolled hypertension may damage organs in the body primarily by which of the following mechanisms? A) a. Hypertension promotes atherosclerosis and damage to the walls of the arteries. B) b. Hypertension causes direct pressure on organs, resulting in necrosis and replacement of cells with scar tissue. C) c. Hypertension causes thickening of the capillary membranes, leading to hypoxia of organ systems. D) d. Hypertension increases blood viscosity, which contributes to intravascular coagulation and tissue necrosis distal to occlusions.

A

Eight months after the delivery of her first child, a 31-year-old woman has sought care because of occasional incontinence that she experiences when sneezing or laughing. Which of the following measures should the nurse first recommend in an attempt to resolve the woman's incontinence? A Kegel exercises B Use of adult incontinence pads C Intermittent self-catheterization D Dietary changes including fluid restriction

A Patients who experience stress incontinence frequently benefit from Kegel exercises (pelvic floor muscle exercises). The use of incontinence pads does not resolve the problem and intermittent self-catheterization would be a premature recommendation. Dietary changes are not likely to influence the patient's urinary continence.

A

A complication of the hyperviscosity of polycythemia is: A. thrombosis B. cardiomyopathy C. pulmonary edema D. disseminated intravascular coagulation (DIC)

D

A patient with multiple myeloma becomes confused and lethargic. The nurse would expect that these clinical manifestations may be explained by diagnostic results that indicate: A. hyperkalemia B. hyperuricemia C. hypercalcemia D. CNS myeloma

After the nurse teaches the patient about the use of atenolol (Tenormin) in preventing anginal episodes, which statement by a patient indicates that the teaching has been effective? A) "It is important not to suddenly stop taking the atenolol." B) "Atenolol will increase the strength of my heart muscle." C) "I can expect to feel short of breath when taking atenolol." D) "Atenolol will improve the blood flow to my coronary arteries."

A) "It is important not to suddenly stop taking the atenolol."

A patient's cardiac monitor has a pattern of undulations of varying contours and amplitude with no measurable ECG pattern. The patient is unconscious and pulseless. Which action should the nurse take first? A) Defibrillate at 360 joules. B) Give O2 per bag-valve-mask. C) Give epinephrine (Adrenalin) IV. D) Prepare for endotracheal intubation.

A) Defibrillate at 360 joules. The patient's rhythm and assessment indicate ventricular fibrillation and cardiac arrest;the initial action should be to defibrillate. If a defibrillator is not immediately available or is unsuccessful in converting the patient to a better rhythm, the other actions may beappropriate.

After the nurse administers IV atropine to a patient with symptomatic type 1, second-degree atrioventricular (AV) block, which finding indicates that the medication has been effective? A) Increase in the patient's heart rate B) Decrease in premature contractions C) Increase in peripheral pulse volume D) Decrease in ventricular ectopic beats

A) Increase in the patient's heart rate Atropine will increase the heart rate and conduction through the AV node. Because the medication increases electrical conduction, not cardiac contractility, the quality of the peripheral pulses is not used to evaluate the drug effectiveness. The patient does not have ventricular ectopy or premature contractions.

Four days after having a myocardial infarction (MI), a patient who is scheduled for discharge asks for assistance with all the daily activities, saying, "I am too nervous to take care of myself." Based on this information, which nursing diagnosis is appropriate? A) Ineffective coping related to anxiety B) Activity intolerance related to weakness C) Denial related to lack of acceptance of the MI D) Social isolation related to lack of support system

A) Ineffective coping related to anxiety

A patient is admitted to the hospital in hypertensive crisis (BP 243/142 mm Hg). Sodium nitroprusside is started to treat the elevated BP. Which management strategy(ies) would be appropriate for this patient? (select all that apply) a. measuring hourly urine output b. decreasing the MAP by 50% within the first hour c. Continuous BP monitoring with an intraarterial line d. maintaining bed rest and providing tranquilizers to lower the BP e. Assessing the patient for signs and symptoms of heart failure and changes in mental status

A, C, E

When reviewing the 12-lead electrocardiograph (ECG) for a healthy 86-year-old patient who is having an annual physical examination, which of the following will be of most concern to the nurse? A) The heart rate (HR) is 43 beats/minute. B) The PR interval is 0.21 seconds. C) There is a right bundle-branch block. D) The QRS duration is 0.13 seconds.

A) The heart rate (HR) is 43 beats/minute.

During a visit to a 72-year-old with chronic heart failure, the home care nurse finds that the patient has ankle edema, a 2-kg weight gain, and complains of "feeling too tired to do anything." Based on these data, the best nursing diagnosis for the patient is A) activity intolerance related to fatigue. B) disturbed body image related to leg swelling. C) impaired skin integrity related to peripheral edema. D) impaired gas exchange related to chronic heart failure.

A) activity intolerance related to fatigue.

Following an acute myocardial infarction, a previously healthy 67-year-old develops clinical manifestations of heart failure. The nurse anticipates discharge teaching will include information about A) a. angiotensin-converting enzyme (ACE) inhibitors. B) b. digitalis preparations. C) c. b-adrenergic agonists. D) d. calcium channel blockers.

A) angiotensin-converting enzyme (ACE) inhibitors.

A patient is diagnosed with hypertension and nadolol (Corgard) is prescribed. The nurse should consult with the health care provider before giving this medication upon finding a history of A) asthma. B) peptic ulcer disease. C) alcohol dependency. D) myocardial infarction (MI).

A) asthma.

To auscultate for S3 or S4 gallops in the mitral area, the nurse listens with the A) bell of the stethoscope with the patient in the left lateral position. B) bell of the stethoscope with the patient sitting and leaning forward. C) diaphragm of the stethoscope with the patient in a reclining position. D) diaphragm of the stethoscope with the patient lying flat on the left side.

A) bell of the stethoscope with the patient in the left lateral position.

In addition to urine function, the nurse recognizes that the kidneys perform numerous other functions important to the maintenance of homeostasis. Which of the following physiologic processes are performed by the kidneys (select all that apply)? A Production of renin B Hemolysis of old red blood cells (RBCs) C Activation of vitamin D D Carbohydrate metabolism E Erythropoietin production

A, C, E In addition to urine formation, the kidneys activate vitamin D to maintain calcium levels, produce erythropoietin to stimulate RBC production, and release renin to maintain blood pressure. Carbohydrate metabolism and hemolysis of old RBCs are not physiologic functions that are performed by the kidneys.

Which of the following nursing responsibilities are priorities when caring for a patient returning from a cardiac catherization (select all that apply)? a. monitoring vital signs and ECG b. checking the catheter insertion site and distal pulses c. assisting the patient to ambulate to the bathroom to void d. informing the patient that he will be sleeping from the general anesthesia e. instructing the patient about the risks of the radioactive isotope injection

A,B

When administering IV nitroglycerin (Tridil) to a patient with a myocardial infarction (MI), which action will the nurse take to evaluate the effectiveness of the medication? A) Check blood pressure. B) Monitor apical pulse rate. C) Monitor for dysrhythmias. D) Ask about chest discomfort.

D) Ask about chest discomfort.

When assessing a patient's nutritional-metabolic pattern related to hematologic health, the nurse would: A. Insepct the skin for petechiae B. ask about joint pain C. assess for Vit C deficiency D. Determine if the pt can perform ADLs

A. Any changes in the skin's texture or color should be explored when assessing the patient's nutritional-metabolic pattern related to hematologic health. The presence of petechiae or ecchymotic areas could be indicative of hematologic deficiencies related to poor nutritional intake or related causes.

A kidney transplant recipient complains of fever, chills, and dysuria over the past 2 days. What is the first action that you should take? A. Assess temperature and initiate a workup to rule out infection. B. Provide warm covers for the patient and give 1 gram of acetaminophen orally. C. Reassure the patient and let him know this is common after transplantation. D. Notify the nephrologist that the patient has developed symptoms of acute rejection.

A. Assess temperature and initiate a workup to rule out infection. You must be astute in the observation and assessment of kidney transplant recipients, because prompt diagnosis and treatment of infections can improve patient outcomes. Fever, chills, and dysuria indicate a possible infection. The temperature should be assessed, and the patient should have diagnostic tests to identify or rule out an infection.

When teaching a client about propranolol hydrochloride, the nurse should base the information on the knowledge that propranolol: A. Blocks beta-adrenergic stimulation and thus causes decreased heart rate, myocardial contractility, and conduction. B. Increases norepinephrine secretion and thus decreases blood pressure and heart rate. C. Is a potent arterial and venous vasodilator that reduces peripheral vascular resistance and lowers blood pressure. D. Is an angiotensin-converting enzyme inhibitor that reduces blood pressure by blocking the conversion of angiotensin I to angiotensin II.

A. Blocks beta-adrenergic stimulation and thus causes decreased heart rate, myocardial contractility, and conduction.

Nurses need to educate patients at risk for CKD. Which individuals are considered to be at increased risk (select all that apply)? A. Older African Americans B. Individuals older than 60 years C. Those with a history of pancreatitis D. Those with a history of hypertension E. Those with a history of type 2 diabetes

A. Older African Americans B. Individuals older than 60 years D. Those with a history of hypertension E. Those with a history of type 2 diabetes Risk factors for CKD include diabetes mellitus, hypertension, age older than 60 years, cardiovascular disease, family history of CKD, exposure to nephrotoxic drugs, and ethnic minorities (e.g., African American, Native American).

What characterizes AKI (select all that apply)? A. Primary cause of death is infection. B. It usually affects older people. C. The disease course is potentially reversible. D. The most common cause is diabetic nephropathy. E. Cardiovascular disease is the most common cause of death.

A. Primary cause of death is infection. C. The disease course is potentially reversible. AKI is potentially reversible. It has a high mortality rate, and the primary cause of death is infection; the primary cause of death for chronic kidney failure is cardiovascular disease. AKI commonly follows severe, prolonged hypotension or hypovolemia or exposure to a nephrotoxic agent. Although it can occur at any age, the older adult is more susceptible to AKI because the number of functioning nephrons decreases with age.

A patient is admitted to the hospital with CKD. You understand that this condition is characterized by A. Progressive irreversible destruction of the kidneys B. A rapid decrease in urinary output with an elevated BUN level C. Increasing creatinine clearance with a decrease in urinary output D. Prostration, somnolence, and confusion with coma and imminent death

A. Progressive irreversible destruction of the kidneys CKD involves progressive, irreversible loss of kidney function.

How do you determine that a patient's oliguria is associated with acute renal failure (ARF)? A. Specific gravity of urine at 3 different times is 1.010. B. The serum creatinine level is normal. C. The blood urea nitrogen (BUN) level is normal or below. D. Hypokalemia is identified.

A. Specific gravity of urine at 3 different times is 1.010. A urinalysis may show casts, red blood cells (RBCs), white blood cells (WBCs), a specific gravity fixed at about 1.010, and urine osmolality at about 300 mOsm/kg.

An ESRD patient receiving hemodialysis is considering asking a relative to donate a kidney for transplantation. In assisting the patient to make a decision about treatment, what do you tell the patient? A. Successful transplantation usually provides better quality of life than that offered by dialysis. B. If rejection of the transplanted kidney occurs, no further treatment for the renal failure is available. C. The immunosuppressive therapy that is required after transplantation causes fatal malignancies in many patients. D. Hemodialysis replaces the normal functions of the kidneys, and patients do not have to live with the continual fear of rejection.

A. Successful transplantation usually provides better quality of life than that offered by dialysis. Kidney transplantation is extremely successful, with 1-year graft survival rates of about 90% for deceased donor transplants and 95% for live donor transplants. An advantage of kidney transplantation compared with dialysis is that it reverses many of the pathophysiologic changes associated with renal failure when normal kidney function is restored. It also eliminates the dependence on dialysis and the accompanying dietary and lifestyle restrictions. Transplantation is also less expensive than dialysis after the first year.

Before beginning a transfusion of RBCs, which of the following actions by the nurse would be of highest priority to avoid an error during this procedure? A. Check the identifying information on the unit of blood against the patient's ID bracelet. B. Select new primary IV tubing primed with lactated Ringer's solution to use for the transfusion. C. Remain with the patient for 60 minutes after beginning the transfusion to watch for signs of a transfusion reaction. D. Add the blood transfusion as a secondary line to the existing IV and use the IV controller to maintain correct flow.

A. The patient's identifying information (name, date of birth, medical record number) on the ID bracelet should exactly match the information on the blood bank tag that has been placed on the unit of blood. If any information does not match, the transfusions should not be hung because of possible error and risk to the patient.

During the oliguric phase of AKI, you monitor the patient for (select all that apply) A. hypertension. B. electrocardiographic (ECG) changes. C. hypernatremia. D. pulmonary edema. E. urine with high specific gravity.

A. hypertension. B. electrocardiographic (ECG) changes. D. pulmonary edema. You monitor the patient in the oliguric phase of AKI for hypertension and pulmonary edema. When urinary output decreases, fluid retention occurs. The severity of the symptoms depends on the extent of the fluid overload. In the case of reduced urine output (anuria and oliguria), the neck veins may become distended and have a bounding pulse. Edema and hypertension may develop. Fluid overload can eventually lead to heart failure, pulmonary edema, and pericardial and pleural effusions. The patient is monitored for hyponatremia. Damaged tubules cannot conserve sodium, and the urinary excretion of sodium may increase, resulting in normal or below-normal levels of serum sodium. Monitoring may reveal ECG changes and hyperkalemia. Initially, clinical signs of hyperkalemia are apparent on electrocardiogram, which demonstrate peaked T waves, widening of the QRS complex, and ST-segment depression. Urinary specific gravity is fixed at about 1.010.

You teach a patient with hypertension that uncontrolled hypertension may damage organs in the body primarily by which mechanism? A. Hypertension promotes atherosclerosis and damage to the walls of the arteries. B. Hypertension causes direct pressure on organs, resulting in necrosis and replacement of cells with scar tissue. C. Hypertension causes thickening of the capillary membranes, leading to hypoxia of organs. D. Hypertension increases blood viscosity, which contributes to intravascular coagulation and tissue necrosis distal to occlusions.

ANS A Hypertension is a major risk factor for the development of atherosclerosis by mechanisms not fully understood. However, after atherosclerosis develops, it damages the walls of the arteries and decreases circulation to target organs and tissues. Reference: 743

You are caring for a patient with hypertension who is scheduled to receive a dose of atenolol (Tenormin). You should withhold the dose and consult the prescribing physician for which vital sign taken just before administration? A. Pulse of 48 beats/minute B. Respiration rate of 24 breaths/minute C. Blood pressure 118/74 mm Hg D. Oxygen saturation level of 93%

ANS: A Because atenolol is a β1-adrenergic blocking agent, it can cause hypotension and bradycardia as adverse effects. You should withhold the dose and consult with the prescriber for parameters regarding pulse rate limits. Reference: 749

The patient with Prinzmetal's angina is being managed with a calcium channel blocker. The patient is also on Lanoxin (digoxin). Which statement is most important for you to follow-up? A. "Everything now looks fuzzy to me." B. "I cannot perform like I use to." C. "I gained weight this week." D. "I have no interest in things."

ANS: A Calcium channel blockers potentiate the action of digoxin by increasing serum digoxin levels during the first week of therapy. Halo vision is a sign of digoxin toxicity. The others are side effects of antihypertensives but are not as serious. Reference: 776, 807

When assessing a patient, you notice a pulse deficit of 23 beats. This finding may be caused by A. dysrhythmias. B. heart murmurs. C. gallop rhythms. D. pericardial friction rubs.

ANS: A A pulse deficit occurs if there is a difference between the apical and radial beats per minute. A pulse deficit indicates cardiac dysrhythmias. Reference: 726

An elderly patient presents to the emergency department after a fall. She states she does not remember the incident. What is most important to assess first? A. Heart rate and rhythm B. Hemoglobin C. Home environment D. Alcohol consumption

ANS: A Although all options will be assessed eventually, determining a cardiac cause for this brief lapse of consciousness is most important. Reference: 839

You are administering a dose of digoxin (Lanoxin) to a patient with heart failure. You become concerned about the possibility of digitalis toxicity when the patient reports which symptoms? A. Anorexia and nausea B. Muscle aches C. Pounding headache D. Constipation

ANS: A Anorexia, nausea, vomiting, blurred or yellow vision, and cardiac dysrhythmias are all signs of digitalis toxicity. You notify the prescriber if the patient exhibits any of these symptoms. Reference: 807

A patient in asystole is likely to receive which drug treatments? A. Atropine and epinephrine B. Lidocaine and amiodarone C. Digoxin and procainamide D. β-Adrenergic blockers and dopamine

ANS: A Normally, the patient in asystole cannot be successfully resuscitated. However, administration of atropine and epinephrine may prompt the return of depolarization and ventricular contraction. Reference: 832

Which diagnostic study best differentiates the various types of cardiomyopathy? A. Echocardiography B. Arterial blood gases C. Cardiac catheterization D. Endomyocardial biopsy

ANS: A The echocardiogram is the primary diagnostic tool used to differentiate the different types of cardiomyopathies and other structural cardiac abnormalities. Reference: 862

The community health nurse is planning health promotion teaching targeted at preventing coronary artery disease. Which ethnic group would you select as the highest priority for this intervention? A. White male B. Hispanic male C. African American male D. Native American female

ANS: A The incidence of CAD and myocardial infarction is highest among white, middle-aged men. Reference: 763

A patient has a severe blockage in his right coronary artery. Which cardiac structure is mostly likely to be affected by this? A. Atrioventricular (AV) node B. Left ventricle C. Coronary sinus D. Pulmonary valve

ANS: A The right coronary artery (RCA) supplies blood to the right atrium, the right ventricle, and a portion of the posterior wall of the left ventricle. In 90% of people, the RCA supplies blood to the AV node, the bundle of His, and part of the cardiac conduction system. Reference: 716

A patient needing vascular access for hemodialysis asks the nurse what the differences are between an arteriovenous (AV) fistula and a graft. The nurse explains that one advantage of the fistula is that it a. is much less likely to clot. b. increases patient mobility. c. can accommodate larger needles. d. can be used sooner after surgery. .

ANS: A AV fistulas are much less likely to clot than grafts, although it takes longer for them to mature to the point where they can be used for dialysis. The choice of an AV fistula or a graft does not have an impact on needle size or patient mobility

A patient needing vascular access for hemodialysis asks the nurse what the differences are between an arteriovenous (AV) fistula and a graft. The nurse explains that one advantage of the fistula is that it a. is much less likely to clot. b. increases patient mobility. c. can accommodate larger needles. d. can be used sooner after surgery.

ANS: A AV fistulas are much less likely to clot than grafts, although it takes longer for them to mature to the point where they can be used for dialysis. The choice of an AV fistula or a graft does not have an impact on needle size or patient mobility.

When assessing a newly admitted patient, the nurse notes a thrill along the left sternal border. To obtain more information about the cause of the thrill, which action will the nurse take next? a. Auscultate for any cardiac murmurs. b. Find the point of maximal impulse. c. Compare the apical and radial pulse rates. d. Palpate the quality of the peripheral pulses.

ANS: A Both thrills and murmurs are caused by turbulent blood flow, such as occurs when blood flows through a damaged valve. Relevant information includes the quality of the murmur, where in the cardiac cycle the murmur is heard, and where on the thorax the murmur is heard best. The other information also is important in the cardiac assessment but will not provide information that is relevant to the thrill. DIF: Cognitive Level: Application REF: 726

A patient who has recently started taking rosuvastatin (Crestor) and niacin (Nicobid) reports all the following symptoms to the nurse. Which is most important to communicate to the health care provider? a. Generalized muscle aches and pains b. Skin flushing after taking the medications c. Dizziness when changing positions quickly d. Nausea when taking the drugs before eating

ANS: A Muscle aches and pains may indicate myopathy and rhabdomyolysis, which have caused acute renal failure and death in some patients who have taken the statin medications. These symptoms indicate that the rosuvastatin may need to be discontinued. The other symptoms are common side effects when taking niacin, and although the nurse should follow up with the patient, they do not indicate that a change in medication is needed. DIF: Cognitive Level: Application REF: 768-770

After the nurse teaches the patient about the use of atenolol (Tenormin) in preventing anginal episodes, which statement by a patient indicates that the teaching has been effective? a. "It is important not to suddenly stop taking the atenolol." b. "Atenolol will increase the strength of my heart muscle." c. "I can expect to feel short of breath when taking atenolol." d. "Atenolol will improve the blood flow to my coronary arteries."

ANS: A Patients who have been taking -blockers can develop intense and frequent angina if the medication is suddenly discontinued. Atenolol (Tenormin) decreases myocardial contractility. Shortness of breath that occurs when taking -blockers for angina may be due to bronchospasm and should be reported to the health care provider. Atenolol works by decreasing myocardial oxygen demand, not by increasing blood flow to the coronary arteries. DIF: Cognitive Level: Application REF: 775 | 776

The nurse has instructed a patient who is receiving hemodialysis about appropriate dietary choices. Which menu choice by the patient indicates that the teaching has been successful? a. Scrambled eggs, English muffin, and apple juice b. Oatmeal with cream, half a banana, and herbal tea c. Split-pea soup, whole-wheat toast, and nonfat milk d. Cheese sandwich, tomato soup, and cranberry juice

ANS: A Scrambled eggs would provide high-quality protein, and apple juice is low in potassium. Cheese is high in salt and phosphate, and tomato soup would be high in potassium. Split-pea soup is high in potassium, and dairy products are high in phosphate. Bananas are high in potassium, and the cream would be high in phosphate.

In a patient with acute kidney injury (AKI) who requires hemodialysis, a temporary vascular access is obtained by placing a catheter in the left femoral vein. Which intervention will be included in the plan of care? a. Place the patient on bed rest. b. Start continuous pulse oximetry. c. Discontinue the retention catheter. d. Restrict the patient's oral protein intake.

ANS: A The patient with a femoral vein catheter must be on bed rest to prevent trauma to the vein. Protein intake is likely to be increased when the patient is receiving dialysis. The retention catheter is likely to remain in place because accurate measurement of output will be needed. There is no indication that the patient needs continuous pulse oximetry

Which nursing responsibilities are priorities when caring for a patient returning from cardiac catheterization (select all that apply)? A. Monitoring vital signs and the electrocardiogram (ECG) B. Checking the catheter insertion site and distal pulses C. Assisting the patient to ambulate to the bathroom to void D. Informing the patient that he will be sleeping because of general anesthesia E. Instructing the patient about the risks of the radioactive isotope injection

ANS: A, B The nursing responsibilities after cardiac catheterization include assessing the puncture site for hematoma and bleeding; assessing circulation to the extremity used for catheter insertion and peripheral pulses, color, and sensation of the extremity; and monitoring vital signs and ECG rhythm. Other nursing responsibilities are described in Table 32-6. Reference: 732, 736

When evaluating the outcomes of preoperative teaching with a patient scheduled for a coronary artery bypass graft (CABG) using the internal mammary artery, the nurse determines that additional teaching is needed when the patient says, a. "I will have incisions in my leg where they will remove the vein." b. "They will circulate my blood with a machine during the surgery." c. "I will need to take an aspirin a day after the surgery to keep the graft open." d. "They will use an artery near my heart to bypass the area that is obstructed."

ANS: A When the internal mammary artery is used there is no need to have a saphenous vein removed from the leg. The other statements by the patient are accurate and indicate that the teaching has been effective. DIF: Cognitive Level: Application REF: 783-784

You are caring for a patient immediately after a transesophageal echocardiogram (TEE). Which assessments are appropriate for this patient (select all that apply)? A. Assess for return of the gag reflex. B. Assess the groin for hematoma or bleeding. C. Monitor vital signs and oxygen saturation. D. Position the patient supine with the head of the bed flat. E. Assess lower extremities for circulatory compromise.

ANS: A, C The patient undergoing TEE has been given conscious sedation and has had the throat numbed with a local anesthetic spray, eliminating the gag reflex until the effects wear off. You must therefore assess for return of the gag reflex before allowing the patient to eat or drink. Vital signs and oxygen saturation are also important assessment parameters resulting from the use of sedation. TEE does not involve invasion of the circulatory blood vessels, and it is not necessary to monitor the patient's groin or lower extremities in relation to this procedure. Reference: 730, 734

When caring for a patient with infective endocarditis, you assess the patient for which vascular manifestations (select all that apply)? A. Osler's nodes B. Janeway's lesions C. Splinter hemorrhages D. Subcutaneous nodules E. Erythema marginatum lesions

ANS: A,B,C Osler's nodes, Janeway's lesions, and splinter hemorrhages are vascular manifestations of infective endocarditis. Subcutaneous nodules and erythema marginatum lesions occur with rheumatic fever. Reference: 843

Which cardiovascular effects of aging should you anticipate when providing care for older adults (select all that apply)? A. Arterial stiffening B. Increased blood pressure C. Increased maximal heart rate D. Decreased maximal heart rate E. Increased recovery time from activity

ANS: A,B,D,E Well-documented cardiovascular effects of the aging process include arterial stiffening, possible increased blood pressure, and an increased amount of time that is required for recovery from activity. Maximal heart rate tends to decrease rather than increase with age. Reference: 720

You are caring for a patient 2 days after an MI. She reports that she is experiencing chest pain. She states, "It hurts when I take a deep breath." Which action is a priority? A. Notify the physician STAT, and obtain a 12-lead ECG. B. Obtain vital signs, and auscultate for a pericardial friction rub. C. Apply high-flow oxygen by face mask, and auscultate breath sounds. D. Medicate the patient with an analgesic as needed, and reevaluate in 30 minutes.

ANS: B Acute pericarditis is an inflammation of the visceral or parietal pericardium; it often occurs 2 to 3 days after an acute MI. Chest pain may vary from mild to severe and is aggravated by inspiration, coughing, and movement of the upper body. Sitting in a forward position often relieves the pain. The pain is usually different from pain associated with an MI. Assessment of the patient with pericarditis may reveal a friction rub over the pericardium. Reference: 780

While obtaining subjective assessment data from a patient with hypertension, you recognize that a modifiable risk factor for hypertension is A. a low-calcium diet. B. excessive alcohol consumption. C. a family history of hypertension. D. consumption of a high-protein diet.

ANS: B Alcohol intake is a modifiable risk factor for hypertension. Excessive alcohol intake is strongly associated with hypertension. Patients with hypertension should limit their daily intake to 1 oz of alcohol. Reference: 742

You are providing teaching to a patient recovering from an MI. Discussion regarding resumption of sexual activity should be A. delegated to the primary care provider. B. discussed along with other physical activities. C. avoided because it is embarrassing to the patient. D. accomplished by providing the patient with written material.

ANS: B Although some nurses may not feel comfortable discussing sexual activity with patients, it is a necessary component of patient teaching. It is helpful to consider sex as a physical activity and to discuss or explore feelings in this area when other physical activities are discussed. Although providing the patient with written material is appropriate, it should not replace a verbal dialogue that can address the individual patient's questions and concerns. Reference: 792

The home care nurse visits a 73-year-old Hispanic woman with chronic heart failure. Which clinical manifestations, if assessed by the nurse, would indicate acute decompensated heart failure (pulmonary edema)? A. Fatigue, orthopnea, and dependent edema B. Severe dyspnea and blood-streaked, frothy sputum C. Temperature is 100.4o F and pulse is 102 beats/minute D. Respirations 26 breaths/minute despite oxygen by nasal cannula

ANS: B Clinical manifestations of pulmonary edema include anxiety, pallor, cyanosis, clammy and cold skin, severe dyspnea, use of accessory muscles of respiration, a respiratory rate > 30 breaths per minute, orthopnea, wheezing, and coughing with the production of frothy, blood-tinged sputum. Auscultation of the lungs may reveal crackles, wheezes, and rhonchi throughout the lungs. The heart rate is rapid, and blood pressure may be elevated or decreased.

You recognize that indications for the use of dopamine (Intropin) in the care of a patient with heart failure include A. acute anxiety. B. hypotension and tachycardia. C. peripheral edema and weight gain. D. paroxysmal nocturnal dyspnea (PND).

ANS: B Dopamine is a β-adrenergic agonist whose inotropic action is reserved for treatment of severe heart failure accompanied by hemodynamic instability. Such a state may be indicated by tachycardia accompanied by hypotension. PND, anxiety, edema, and weight gain are common signs and symptoms of heart failure, but they do not warrant the use of dopamine. Reference: 805

When providing dietary instruction to a patient with hypertension, you advise the patient to restrict intake of which meat? A. Broiled fish B. Roasted duck C. Roasted turkey D. Baked chicken breast

ANS: B Roasted duck is high in fat, which should be avoided by the patient with hypertension. The other meats are lower in fat and are therefore acceptable in the diet. Reference: 745

The portion of the vascular system responsible for hemostasis is the A. thin capillary vessels. B. endothelial layer of the arteries. C. elastic middle layer of the veins. D. smooth muscle of the arterial wall.

ANS: B The innermost lining of the arteries is the endothelium. The endothelium maintains hemostasis, promotes blood flow, and under normal conditions, inhibits blood coagulation. Reference: 718

The auscultatory area in the left midclavicular line at the level of the fifth intercostal space (ICS) is the A. aortic valve area. B. mitral valve area. C. tricuspid valve area. D. pulmonic valve area.

ANS: B The mitral valve can be assessed by auscultation at the left midclavicular line at the fifth ICS. Reference: 724

When evaluating laboratory results of the patient with heart failure you note a significant increase in the B-type natriuretic peptide (BNP). You interpret this finding to mean the patient's A. blood pressure is dropping and pulse rate is rising. B. heart failure is worsening. C. aldosterone levels are dropping. D. ventricular hypertrophy is resolving.

ANS: B The release of BNP is triggered by increased pressure, especially in the left ventricle. BNP levels are significantly increased in patients with heart failure. Reference: 800

A patient with a diagnosis of heart failure has been started on a nitroglycerin patch by his primary care provider. This patient should be advised to avoid A. high-potassium foods. B. drugs to treat erectile dysfunction. C. over-the-counter H2-receptor blockers. D. nonsteroidal antiinflammatory drugs (NSAIDs).

ANS: B The use of erectile drugs concurrent with nitrates creates a risk of severe hypotension. NSAIDs, H2-receptor blockers, and high-potassium foods do not pose a risk in combination with nitrates. Reference: 807

The patient has a permanent cardiac pacemaker. On the electrocardiographic tracing, you notice a spike before each P wave. What action should you take? A. Assess the patient for syncope. B. Document the findings. C. Notify the physician. D. Take blood pressure in both arms.

ANS: B These pacer spikes show that the pacemaker is firing and the atrium is responding to the impulse. It is a normal, expected finding in this situation. Reference: 834-835

A patient with hypertension and stage 2 chronic kidney disease (CKD) is receiving captopril (Capoten). Before administration of the medication, the nurse will check the patient's a. glucose. b. potassium. c. creatinine. d. phosphate.

ANS: B Angiotensin-converting enzyme (ACE) inhibitors are frequently used in patients with CKD because they delay the progression of the CKD, but they cause potassium retention. Therefore, careful monitoring of potassium levels is needed in patients who are at risk for hyperkalemia. The other laboratory values also would be monitored in patients with CKD but would not affect whether the captopril was given or not.

When admitting a patient with a myocardial infarction (MI) to the intensive care unit, which action should the nurse carry out first? a. Obtain the blood pressure. b. Attach the cardiac monitor. c. Assess the peripheral pulses. d. Auscultate the breath sounds.

ANS: B Because dysrhythmias are the most common complication of MI, the first action should be to place the patient on a cardiac monitor. The other actions also are important and should be accomplished as quickly as possible. DIF: Cognitive Level: Application REF: 779-780 | 787-788

Which patient information will the nurse plan to obtain in order to determine the effectiveness of the prescribed calcium carbonate (Caltrate) for a patient with chronic kidney disease (CKD)? a. Blood pressure b. Phosphate level c. Neurologic status d. Creatinine clearance

ANS: B Calcium carbonate is prescribed to bind phosphorus and prevent mineral and bone disease in patients with CKD. The other data will not be helpful in evaluating the effectiveness of calcium carbonate

Which patient information will the nurse plan to obtain in order to determine the effectiveness of the prescribed calcium carbonate (Caltrate) for a patient with chronic kidney disease (CKD)? a. Blood pressure b. Phosphate level c. Neurologic status d. Creatinine clearance .

ANS: B Calcium carbonate is prescribed to bind phosphorus and prevent mineral and bone disease in patients with CKD. The other data will not be helpful in evaluating the effectiveness of calcium carbonate

The nurse has received the laboratory results for a patient who developed chest pain 4 hours ago and may be having a myocardial infarction. The most important laboratory result to review will be a. LDL cholesterol. b. troponins T and I. c. C-reactive protein. d. creatine kinase-MB (CK-MB).

ANS: B Cardiac troponins start to elevate hours (average 4 to 6 hours) after myocardial injury and are specific to myocardium. Creatine kinase (CK-MB) is specific to myocardial injury and infarction, but it does not increase until 6 hours after the infarction occurs. LDL cholesterol and C-reactive protein are useful in assessing cardiovascular risk but are not helpful in determining whether a patient is having an acute myocardial infarction. DIF: Cognitive Level: Application REF: 726-727

Which nursing action for a patient who has arrived for a scheduled hemodialysis session is most appropriate for the RN to delegate to a dialysis technician? a. Educate patient about fluid restrictions. b. Check blood pressure before starting dialysis. c. Assess for reasons for increase in predialysis weight. d. Determine the ultrafiltration rate for the hemodialysis.

ANS: B Dialysis technicians are educated in monitoring for blood pressure. Assessment, adjustment of the appropriate ultrafiltration rate, and patient teaching require the education and scope of practice of an RN.

The priority nursing goal for the patient with pericarditis is A. treatment of infection. B. pain relief. C. management of dysrhythmias. D. maintenance of renal blood flow.

ANS: B Management of the patient's pain and anxiety during acute pericarditis is your primary nursing consideration. You must assess the amount, quality, and location of the pain to differentiate the pain of myocardial ischemia (angina) from the pain of pericarditis. Pericarditic pain is usually located in the precordium or left trapezius region and has a sharp, pleuritic quality that increases with inspiration. Pain is often relieved when the patient sits up or leans forward, and it is worse when lying supine. Reference: 848

Amlodipine (Norvasc) is ordered for a patient with newly diagnosed Prinzmetal's (variant) angina. When teaching the patient, the nurse will include the information that amlodipine will a. reduce the "fight or flight" response. b. decrease spasm of the coronary arteries. c. increase the force of myocardial contraction. d. help prevent clotting in the coronary arteries.

ANS: B Prinzmetal's angina is caused by coronary artery spasm. Calcium channel blockers (e.g., amlodipine, nifedipine [Procardia]) are a first-line therapy for this type of angina. Platelet inhibitors, such as aspirin, help prevent coronary artery thrombosis, and -blockers decrease sympathetic stimulation of the heart. Medications or activities that increase myocardial contractility will increase the incidence of angina by increasing oxygen demand. DIF: Cognitive Level: Application REF: 776

When admitting a patient for a coronary arteriogram and angiogram, which information about the patient is most important for the nurse to communicate to the health care provider? a. The patient's pedal pulses are +1. b. The patient is allergic to shellfish. c. The patient has not eaten anything today. d. The patient had an arteriogram a year ago.

ANS: B The contrast dye used for the procedure is iodine based, so patients who have shellfish allergies will require treatment with medications such as corticosteroids and antihistamines before the arteriogram. The other information also is communicated to the health care provider but will not require a change in the usual prearteriogram orders or medications. DIF: Cognitive Level: Application REF: 732

When the nurse is caring for a patient who has been admitted with a severe crushing injury after an industrial accident, which laboratory result will be most important to report to the health care provider? a. Serum creatinine level 2.1 mg/dL b. Serum potassium level 6.5 mEq/L c. White blood cell count 11,500/µL d. Blood urea nitrogen (BUN) 56 mg/dL

ANS: B The hyperkalemia associated with crushing injuries may cause cardiac arrest and should be treated immediately. The nurse also will report the other laboratory values, but abnormalities in these are not immediately life threatening. DIF: Cognitive Level: Application REF: 1167

Which action will the nurse implement for a patient who arrives for a calcium-scoring CT scan? a. Administer oral sedative medications. b. Teach the patient about the procedure. c. Ask whether the patient has eaten today. d. Insert a large gauge intravenous catheter.

ANS: B The nurse will need to teach the patient that the procedure is rapid and involves little risk. The other actions are not necessary. DIF: Cognitive Level: Application REF: 731-732

A patient with a history of benign prostatic hyperplasia (BPH) is admitted with acute urinary retention and an elevated blood urea nitrogen (BUN) and creatinine. Which of these prescribed therapies should the nurse implement first? a. Obtain renal ultrasound. b. Insert retention catheter. c. Infuse normal saline at 50 mL/hour. d. Draw blood for complete blood count.

ANS: B The patient's elevation in BUN is most likely associated with hydronephrosis caused by the acute urinary retention, so the insertion of a retention catheter is the first action to prevent ongoing postrenal failure for this patient. The other actions also are appropriate, but should be implemented after the retention catheter.

When assessing the cardiovascular system of a 79-year-old patient, you expect to find A. a narrowed pulse pressure. B. diminished carotid artery pulses. C. difficulty in isolating the apical pulse. D. an increased heart rate in response to stress.

ANS: C Myocardial hypertrophy and the downward displacement of the heart in an older adult may result in difficulty isolating the apical pulse. Reference: 720

The nurse provides discharge instructions for a 40-year-old woman who is newly diagnosed with cardiomyopathy. Which statement, if made by the patient, indicates that further teaching is necessary? A. "I will avoid lifting heavy objects." B. "I can drink alcohol in moderation." C. "My family will need to take a CPR course." D. "I will reduce stress by learning guided imagery."

ANS: B Patients with cardiomyopathy should avoid alcohol consumption, especially in patients with alcohol-related dilated cardiomyopathy. Avoiding heavy lifting and stress, as well as family members learning CPR, are recommended teaching points.

In teaching a patient about coronary artery disease (CAD), you explain which changes that occur in this disorder (select all that apply)? A. Diffuse involvement of plaque formation in coronary veins B. Abnormal levels of cholesterol, particularly low-density lipoproteins C. Accumulation of lipid and fibrous tissue within the coronary arteries D. Development of angina due to decreased blood supply to the heart muscle E. Chronic vasoconstriction of coronary arteries leading to permanent vasospasm

ANS: B,C,D Atherosclerosis is the major cause of coronary artery disease (CAD) and is characterized by a focal deposit of cholesterol and lipids, primarily within the intimal wall of the artery. The endothelial lining of the coronary arteries becomes inflamed by unstable plaques and the oxidation of low-density lipoprotein (LDL) cholesterol. Fibrous plaque causes progressive changes in the endothelium of the arterial wall. The result is a narrowed vessel lumen and a reduction in blood flow to the myocardial tissue. Reference: 761

You are caring for a patient 24 hours after pacemaker insertion. Which nursing intervention is most appropriate at this time? A. Reinforcing the pressure dressing as needed B. Encouraging range-of-motion exercises of the involved arm C. Assessing the incision for any redness, swelling, or discharge D. Applying wet-to-dry dressings every 4 hours to the insertion site

ANS: C After pacemaker insertion, it is important for the patient to limit activity of the involved arm to minimize pacemaker lead displacement. The nonpressure dressing is kept dry until removed, usually 24 hours postoperatively. It is important for you to observe signs of infection by assessing for redness, swelling, or discharge from the incision site. Reference: 836-837

The patient hospitalized with unstable angina exhibits signs of anxiety. What should you ask or tell the patient? A.Why are you feeling anxious? B. Modern medicine can help heart conditions so effectively these days. C. Fear of dying is a common concern for patients with your condition. D. What information do you need to know about your disease?

ANS: C All patients with acute coronary syndrome (ACS) have anxiety. Frequently, the patient cannot verbalize the most pervasive concern: "Am I going to die?" It is helpful for you to initiate conversation by remarking that fear of dying is a common concern among most patients who have ACS. Reference: 788

The patient with Prinzmetal's angina is prescribed nitroglycerin tablets. What should you teach the patient about this drug? A. Do not use the tablets if they taste tingling when used. B. Store the medication bottle in the bathroom so it is easily located. C. Do not take any drug for erectile dysfunction, such as Viagra. D. Call 911 if you experience flushing when taking the tablet.

ANS: C Nitroglycerin cannot be combined with erectile dysfunction medications because it potentiates vasodilation. Nitroglycerin tablets should taste tingling if still potent (the drug is easily degraded). The medicine should always be easily accessible, but it is not recommended to store it near light or heat. It can degrade and lose potency, and the patient should purchase a new supply every 6 months. Common side effects include increased heart rate, headache, dizziness, and flushing. Reference: 775

What should you closely monitor in the patient with aortic stenosis who is also receiving nitroglycerin? A. Heart rate B. Heart rhythm C. Blood pressure D. Peripheral pulses

ANS: C Nitroglycerin is used cautiously to treat angina in the patient with aortic stenosis, but it can significantly reduce blood pressure and worsen chest pain. The blood pressure must be closely monitored. Reference: 855

Postoperative care of a patient undergoing coronary artery bypass graft (CABG) surgery includes monitoring for which common complication? A. Dehydration B. Paralytic ileus C. Atrial dysrhythmias D. Acute respiratory distress syndrome

ANS: C Postoperative dysrhythmias, specifically atrial dysrhythmias, are common in the first 3 days after CABG surgery. Although the other complications can occur, they are not common. Reference: 789

A compensatory mechanism involved in heart failure that leads to inappropriate fluid retention and additional workload of the heart is A. ventricular dilation. B. ventricular hypertrophy. C. neurohormonal response. D. sympathetic nervous system activation

ANS: C Several mechanisms in heart failure lead to inappropriate fluid retention and additional workload of the heart, including activation of the renin-angiotensin-aldosterone system (RAAS) cascade and release of antidiuretic hormone from the posterior pituitary gland in response to low cerebral perfusion pressure from low cardiac output.

The patient with acute coronary syndrome (ACS) undergoes coronary revascularization with balloon angioplasty with placement of a drug-eluting stent. The physician prescribes a glycoprotein IIb/IIIa inhibitor tirofiban (Aggrastat). What is the main rationale for administering this drug? A. To promote vasodilation B. To inhibit ST elevation C. To prevent closure of the stent D. To inhibit hemorrhage

ANS: C The IIb/IIIa inhibitor prevents the abrupt closure of the stent and is initiated during the percutaneous coronary intervention and maintained for 12 hours after the procedure. Reference: 777

Which is a correct understanding of an electrocardiogram (ECG)? A. The U wave represents ventricular repolarization. B. The impulse to trigger ventricular contraction should take 1.2 seconds. C. The P wave indicates the impulse that is coming from the atrium. D. There is an absolute refractory period during the T wave.

ANS: C The P wave indicates the beginning of the firing of the sinoatrial (SA) node and represents depolarization of the fibers of the atria. Normally, there should be a P wave before every QRS. The U wave, if seen, may represent repolarization of the Purkinje fibers, or it may be associated with hypokalemia. The efficient ventricular conduction system delivers the impulse within 0.12 seconds to trigger a synchronized ventricular contraction. The T wave represents repolarization of the ventricles. Reference: 717

A 54-year-old male patient who had bladder surgery 2 days ago develops acute decompensated heart failure (ADHF) with severe dyspnea. Which action by the nurse would be indicated first? A. Perform a bladder scan to assess for urinary retention. B. Restrict the patient's oral fluid intake to 500 mL per day. C. Assist the patient to a sitting position with arms on the overbed table. D. Instruct the patient to use pursed-lip breathing until the dyspnea subsides.

ANS: C The nurse should place the patient with ADHF in a high Fowler's position with the feet horizontal in the bed or dangling at the bedside. This position helps decrease venous return because of the pooling of blood in the extremities. This position also increases the thoracic capacity, allowing for improved ventilation. Pursed-lip breathing helps with obstructive air trapping but not with acute pulmonary edema. Restricting fluids takes considerable time to have an effect.

While assessing the cardiovascular status of a patient, you perform auscultation. Which practice should you implement into the assessment during auscultation? A. Position the patient supine. B. Ask the patient to hold his or her breath. C. Palpate the radial pulse while auscultating the apical pulse. D. Use the bell of the stethoscope when auscultating S1 and S2.

ANS: C To detect a pulse deficit, simultaneously palpate the radial pulse when auscultating the apical area. The diaphragm of the stethoscope is more appropriate than the bell when auscultating S1 and S2. A sitting or side-lying position is most appropriate for cardiac auscultation. It is not necessary to ask the patient to hold his or her breath during cardiac auscultation. Reference: 726

To improve cardiac contractility in the patient with myocarditis, you anticipate the health care provider will prescribe A. diuretics. B. nitroprusside. C. digoxin. D. β-blockers.

ANS: C Treatment for myocarditis consists of managing associated cardiac symptoms. Digoxin (Lanoxin) improves myocardial contractility and reduces ventricular rate but is used cautiously in patients with myocarditis because of the increased sensitivity of the heart to the adverse effects of this drug. Diuretics may be used, but they do not increase contractility. Nitroprusside lowers blood pressure, but it does not improve contractility. The β-blockers may help heart enlargement. Reference: 849

A patient is scheduled for a cardiac catheterization with coronary angiography. Before the test, the nurse informs the patient that a. electrocardiographic (ECG) monitoring will be required for 24 hours after the test. b. it will be important to lie completely still during the procedure. c. a warm feeling may be noted when the contrast dye is injected. d. monitored anesthesia care will be provided during the procedure.

ANS: C A sensation of warmth or flushing is common when the iodine-based contrast material is injected, which can be anxiety-producing unless it has been discussed with the patient. The patient may receive a sedative drug before the procedure, but monitored anesthesia care is not used. ECG monitoring is used during the procedure to detect dysrhythmias, but there is not a risk for dysrhythmias after the procedure. The patient is not immobile during cardiac catheterization and may be asked to cough or take deep breaths. DIF: Cognitive Level: Application REF: 732 | 735-736

Which of the following information obtained by the nurse who is caring for a patient with end-stage renal disease (ESRD) indicates the nurse should consult with the health care provider before giving the prescribed epoetin alfa (Procrit)? a. Creatinine 1.2 mg/dL b. Oxygen saturation 89% c. Hemoglobin level 13 g/dL d. Blood pressure 98/56 mm Hg

ANS: C High hemoglobin levels are associated with a higher rate of thromboembolic events and increased risk of death from serious cardiovascular events (heart attack, heart failure, stroke) when EPO is administered to a target hemoglobin of >12 g/dL. Hemoglobin levels higher than 12 g/dL indicate a need for a decrease in epoetin alfa dose. The other information also will be reported to the health care provider, but will not affect whether the medication is administered

Which information obtained by the nurse who is admitting the patient for magnetic resonance imaging (MRI) will be most important to report to the health care provider before the MRI? a. The patient has an allergy to shellfish and iodine. b. The patient has a history of coronary artery disease. c. The patient has a permanent ventricular pacemaker in place. d. The patient took all the prescribed cardiac medications today.

ANS: C MRI is contraindicated for patients with implanted metallic devices such as pacemakers. The other information also will be reported to the health care provider but does not impact on whether or not the patient can have an MRI. DIF: Cognitive Level: Application REF: 731-732 | 734-735

When a patient who has had progressive chronic kidney disease (CKD) for several years is started on hemodialysis, which information about diet will the nurse include in patient teaching? a. Increased calories are needed because glucose is lost during hemodialysis. b. Unlimited fluids are allowed since retained fluid is removed during dialysis. c. More protein will be allowed because of the removal of urea and creatinine by dialysis. d. Dietary sodium and potassium are unrestricted because these levels are normalized by dialysis.

ANS: C Once the patient is started on dialysis and nitrogenous wastes are removed, more protein in the diet is encouraged. Fluids are still restricted to avoid excessive weight gain and complications such as shortness of breath. Glucose is not lost during hemodialysis. Sodium and potassium intake continues to be restricted to avoid the complications associated with high levels of these electrolytes

When a patient who has had progressive chronic kidney disease (CKD) for several years is started on hemodialysis, which information about diet will the nurse include in patient teaching? a. Increased calories are needed because glucose is lost during hemodialysis. b. Unlimited fluids are allowed since retained fluid is removed during dialysis. c. More protein will be allowed because of the removal of urea and creatinine by dialysis. d. Dietary sodium and potassium are unrestricted because these levels are normalized by dialysis.

ANS: C Once the patient is started on dialysis and nitrogenous wastes are removed, more protein in the diet is encouraged. Fluids are still restricted to avoid excessive weight gain and complications such as shortness of breath. Glucose is not lost during hemodialysis. Sodium and potassium intake continues to be restricted to avoid the complications associated with high levels of these electrolytes.

Before administering sodium polystyrene sulfonate (Kayexalate) to a patient with hyperkalemia, the nurse should assess the a. blood urea nitrogen (BUN) and creatinine. b. blood glucose level. c. patient's bowel sounds. d. level of consciousness (LOC).

ANS: C Sodium polystyrene sulfonate (Kayexalate) should not be given to a patient with a paralytic ileus (as indicated by absent bowel sounds) because bowel necrosis can occur. The BUN and creatinine, blood glucose, and LOC would not affect the nurse's decision to give the medication.

Three days after a myocardial infarction (MI), the patient develops chest pain that increases when taking a deep breath and is relieved by leaning forward. Which action should the nurse take next? a. Palpate the radial pulses bilaterally. b. Assess the feet for peripheral edema. c. Auscultate for a pericardial friction rub. d. Check the cardiac monitor for dysrhythmias.

ANS: C The patient's symptoms are consistent with the development of pericarditis, a possible complication of MI. The other assessments listed are not consistent with the description of the patient's symptoms. DIF: Cognitive Level: Application REF: 780

A patient in the oliguric phase of acute renal failure has a 24-hour fluid output of 150 mL emesis and 250 mL urine. The nurse plans a fluid replacement for the following day of ___ mL. a. 400 b. 800 c. 1000 d. 1400

ANS: C Usually fluid replacement should be based on the patient's measured output plus 600 mL/day for insensible losses. DIF: Cognitive Level: Application REF: 1168

The ECG monitor of a patient in the cardiac care unit after a myocardial infarction (MI) indicates ventricular bigeminy with a rate of 50 beats/minute. You anticipate A. performing defibrillation. B. treatment with IV lidocaine. C. insertion of a temporary, transvenous pacemaker. D. assessing the patient's response to the dysrhythmia.

ANS: D A premature ventricular contraction (PVC) is a contraction originating in an ectopic focus in the ventricles. When every other beat is a PVC, the rhythm is called ventricular bigeminy. PVCs are usually a benign finding in the patient with a normal heart. In heart disease, PVCs may reduce the cardiac output and precipitate angina and heart failure, depending on the frequency. Because PVCs in coronary artery disease or acute MI indicate ventricular irritability, the patient's physiologic response to PVCs must be monitored. Assessment of the patient's hemodynamic status is important to determine whether treatment with drug therapy is needed. Reference: 830

For which antilipemic medication would you question an order for a patient with cirrhosis of the liver? A. Niacin (Nicobid) B. Ezetimibe (Zetia) C. Gemfibrozil (Lopid) D. Atorvastatin (Lipitor)

ANS: D Adverse effects of atorvastatin, a statin drug, include liver damage and myopathy. Liver enzymes must be monitored frequently, and the medication should be stopped if these enzyme levels increase. Liver disease is a contraindication for using atorvastatin. Reference: 770

You are caring for a patient admitted with pulmonary edema due to acute decompensated heart failure. During the morning assessment, the patient complains of yellow vision. Which medication is most likely causing this clinical manifestation related to toxicity? A. Meperidine (Demerol) B. Atenolol (Tenormin) C. Cefotetan (Cefotan) D. Digoxin (Lanoxin)

ANS: D Blurred or yellow vision is a sign of digitalis toxicity. You then assess the patient for other manifestations, such as anorexia, nausea, vomiting, and cardiac dysrhythmias. Reference: 807

When evaluating a patient's knowledge regarding a low-sodium, low-fat cardiac diet, you recognize additional teaching is needed when the patient selects which food choice? A. Baked flounder B. Angel food cake C. Baked potato with margarine D. Canned chicken noodle soup

ANS: D Canned soups contain very high sodium levels. Patients need to be taught to read food labels for sodium and fat content. Reference: 768

A 55-year-old female patient develops acute pericarditis after a myocardial infarction. It is most important for the nurse to assess for which clinical manifestation of a possible complication? A. Presence of a pericardial friction rub B. Distant and muffled apical heart sounds C. Increased chest pain with deep breathing D. Decreased blood pressure with tachycardia

ANS: D Cardiac tamponade is a serious complication of acute pericarditis. Signs and symptoms indicating cardiac tamponade include narrowed pulse pressure, tachypnea, tachycardia, a decreased cardiac output, and decreased blood pressure. The other symptoms are consistent with acute pericarditis.

For what is per-cutaneous coronary intervention (PCI) most clearly indicated? A. Chronic stable angina B. Left-sided heart failure C. Coronary artery disease (CAD) D. Acute MI

ANS: D PCI is used to restore coronary perfusion in cases of MI. Chronic stable angina and CAD are normally treated with more conservative measures initially. PCI is not relevant to the pathophysiology of heart failure. Reference: 780-781

The patient with primary hypertension is going to take the initial dose of doxazosin (Cardura) at home. What is most important for you to teach the patient? A. How to take a radial pulse B. To chew sugarless gum C. To take diuretic in the morning D. To take this drug at bedtime

ANS: D The initial dose is most likely to produce a side effect. Syncope (sudden loss of consciousness) occasionally occurs 30 to 90 minutes after the initial dose. The side effects of hypotension and syncope are less likely if the drug is taken in the evening. The pulse is not as important as blood pressure monitoring of patients taking α-adrenergic blockers. Dry mouth is a common side effect that can be helped with sugarless gum, but hypotension is more important. Frequent voiding is a common side effect, and nocturia can be helped by taking a diuretic in the morning, but the hypotension is most important. Reference: 752

Nadolol (Corgard) is prescribed for a patient with angina. To determine whether the drug is effective, the nurse will monitor for a. decreased blood pressure and apical pulse rate. b. fewer complaints of having cold hands and feet. c. improvement in the quality of the peripheral pulses. d. the ability to do daily activities without chest discomfort.

ANS: D Because the medication is ordered to improve the patient's angina, effectiveness is indicated if the patient is able to accomplish daily activities without chest pain. Blood pressure (BP) and apical pulse rate may decrease, but these data do not indicate that the goal of decreased angina has been met. The noncardioselective -blockers can cause peripheral vasoconstriction, so the nurse would not expect an improvement in peripheral pulse quality or skin temperature. DIF: Cognitive Level: Application REF: 776

Which information given by a patient admitted with chronic stable angina will help the nurse confirm this diagnosis? a. The patient rates the pain at a level 3 to 5 (0 to 10 scale). b. The patient states that the pain "wakes me up at night." c. The patient says that the frequency of the pain has increased over the last few weeks. d. The patient states that the pain is resolved after taking one sublingual nitroglycerin tablet.

ANS: D Chronic stable angina is typically relieved by rest or nitroglycerin administration. The level of pain is not a consistent indicator of the type of angina. Pain occurring at rest or with increased frequency is typical of unstable angina. DIF: Cognitive Level: Comprehension REF: 771-776

A patient with a non-ST-segment-elevation myocardial infarction (NSTEMI) is receiving heparin. What is the purpose of the heparin? a. Platelet aggregation is enhanced by IV heparin infusion. b. Heparin will dissolve the clot that is blocking blood flow to the heart. c. Coronary artery plaque size and adherence are decreased with heparin. d. Heparin will prevent the development of new clots in the coronary arteries.

ANS: D Heparin helps prevent the conversion of fibrinogen to fibrin and decreases coronary artery thrombosis. It does not change coronary artery plaque, dissolve already formed clots, or enhance platelet aggregation. DIF: Cognitive Level: Comprehension REF: 775

Which of these nursing interventions included in the plan of care for a patient who had an acute myocardial infarction (AMI) 3 days ago is most appropriate for the RN to delegate to an experienced LPN/LVN? a. Evaluating the patient's response to ambulation in the hallway b. Completing the documentation for a home health nurse referral c. Educating the patient about the pathophysiology of heart disease d. Reinforcing teaching about the purpose of prescribed medications

ANS: D LPN/LVN education and scope of practice include reinforcing education that has previously been done by the RN. Evaluating the patient response to exercise after an AMI requires more education and should be done by the RN. Teaching and discharge planning/documentation are higher level skills that require RN education and scope of practice. DIF: Cognitive Level: Application REF: 789-793

A patient with acute kidney injury (AKI) has an arterial blood pH of 7.30. The nurse will assess the patient for a. vasodilation. b. poor skin turgor. c. bounding pulses. d. rapid respirations.

ANS: D Patients with metabolic acidosis caused by AKI may have Kussmaul respirations as the lungs try to regulate carbon dioxide. Bounding pulses and vasodilation are not associated with metabolic acidosis. Because the patient is likely to have fluid retention, poor skin turgor would not be a finding in AKI.

A patient with hyperlipidemia has a new order for colesevelam (Welchol). Which nursing action is appropriate when giving the medication? a. Administer the medication at the patient's bedtime. b. Have the patient take this medication with an aspirin. c. Encourage the patient to take the colesevelam with a sip of water. d. Give the patient's other medications 2 hours after the colesevelam.

ANS: D The bile acid sequestrants interfere with the absorption of other drugs, and giving other medications at the same time should be avoided. Taking an aspirin concurrently with the colesevelam may increase the incidence of gastrointestinal side effects such as heartburn. An increased fluid intake is encouraged for patients taking the bile acid sequestrants to reduce the risk for constipation. For maximum effect, colesevelam should be administered with meals. DIF: Cognitive Level: Application REF: 770-771

A male patient with long-standing heart failure has qualified for hospice care. Which measure should you prioritize when providing care for this patient? A. Tapering the patient off his current medications B. Continuing education for the patient and his family C. Pursuing experimental therapies or surgical options D. Choosing interventions to promote comfort and prevent suffering

ANS: D The central focus of hospice care is the promotion of comfort and the prevention of suffering. Patient education should continue, but providing comfort is paramount. Medications should be continued unless they are not tolerated, and experimental therapies and surgeries are not commonly used in the care of hospice patients. Reference: 814

When caring for a patient with acute coronary syndrome who has returned to the coronary care unit after having balloon angioplasty, the nurse obtains the following assessment data. Which data indicate the need for immediate intervention by the nurse? a. Pedal pulses 1+ b. Heart rate 100 beats/min c. Blood pressure 104/56 mm Hg d. Chest pain level 8 on a 10-point scale

ANS: D The patient's chest pain indicates that restenosis of the coronary artery may be occurring and requires immediate actions, such as administration of oxygen and nitroglycerin, by the nurse. The other information indicates a need for ongoing assessments by the nurse. DIF: Cognitive Level: Application REF: 781-782

A new order for IV gentamicin (Garamycin) 60 mg BID is received for a patient with diabetes who has pneumonia. When evaluating for adverse effects of the medication, the nurse will plan to monitor the patient's a. urine osmolality. b. serum potassium. c. blood glucose level. d. blood urea nitrogen (BUN) and creatinine.

ANS: D When a patient at risk for chronic kidney disease (CKD) receives a nephrotoxic medication, it is important to monitor renal function with BUN and creatinine levels. The other laboratory values would not be useful in determining the effect of the gentamicin

You prepare a patient for synchronized cardioversion knowing that cardioversion differs from defibrillation in that A. defibrillation requires lower dose of electrical energy. B. cardioversion is indicated for treatment of atrial bradydysrhythmias. C. defibrillation is synchronized to deliver a shock during the QRS complex. D. patients should be sedated if cardioversion is done on a nonemergent basis.

ANS:D Synchronized cardioversion is the therapy of choice for the patient with hemodynamically unstable ventricular or supraventricular tachydysrhythmias. A synchronized circuit in the defibrillator delivers a countershock that is programmed to occur on the R wave of the QRS complex seen on the ECG tracing. The synchronizer switch must be turned on when cardioversion is planned. The procedure for synchronized cardioversion is the same as for defibrillation, with the following exceptions. If synchronized cardioversion is done on a nonemergent basis, the patient is sedated before the procedure. The initial energy needed for synchronized cardioversion is less than the energy needed for defibrillation. Reference: 833

Albuterol is contraindicated for ... Beta blockers are contraindicated for ...

Albuterol is contraindicated for ... cardiovascular disease Beta blockers are contraindicated for ... asthma

When teaching a patient about dietary management of stage 1 hypertension, which of the following instructions is most appropriate? A) a. Restrict all caffeine. B) b. Restrict sodium intake. C) c. Increase protein intake. D) d. Use calcium supplements.

B

The most significant factor in long-term survival of a patient with sudden cardiac death is a. absence of underlying heart disease. b. rapid institution of emergency services and procedures. c. performance of perfect technique in resuscitation procedures. d. maintenance of 50% of normal cardiac output during resuscitation efforts.

B

The nurse is admitting a patient who is scheduled to undergo a cardiac catheterization. Which of the following allergies is most important for the nurse to assess before this procedure? A) a. Iron B) b. Iodine C) c. Aspirin D) d. Penicillin

B

Which of the following assessment findings is a consequence of the oliguric phase of acute kidney injury (AKI)? A Hypovolemia B Hyperkalemia C Hypernatremia D Thrombocytopenia

B In AKI the serum potassium levels increase because the normal ability of the kidneys to excrete potassium is impaired. Sodium levels are typically normal or diminished, whereas fluid volume is normally increased because of decreased urine output. Thrombocytopenia is not a consequence of AKI, although altered platelet function may occur in AKI.

15. While admitting an 80-year-old patient with heart failure to the medical unit, the nurse obtains the information that the patient lives alone and sometimes confuses the "water pill" with the "heart pill." The nurse makes a note that discharge planning for the patient will need to include a. transfer to a dementia care service. b. referral to a home health care agency. c. placement in a long-term-care facility. d. arrangements for around-the-clock care.

B Rationale: The data about the patient suggest that assistance in developing a system for taking medications correctly at home is needed. A home health nurse will assess the patient's home situation and help the patient to develop a method for taking the two medications as directed. There is no evidence that the patient requires services such as dementia care, long-term-care, or around-the-clock home care. Cognitive Level: Application Text Reference: pp. 836-837 Nursing Process: Assessment NCLEX: Health Promotion and Maintenance

The nurse is admitting a patient with the diagnosis of advanced renal carcinoma. Based upon this diagnosis, the nurse will expect to find which of the following as the "classic triad" of presenting symptoms occurring in patients with renal cancer? A Fever, chills, flank pain B Hematuria, flank pain, palpable mass C Hematuria, proteinuria, palpable mass D Flank pain, palpable abdominal mass, and proteinuria

B There are no characteristic early symptoms of renal carcinoma. The classic manifestations of gross hematuria, flank pain, and a palpable mass are those of advanced disease.

A patient has ST segment changes that indicate an acute inferior wall myocardial infarction. Which lead will be best for monitoring the patient? A) I B) II C) V6 D) MCL1

B) II Lead II reflects the inferior area of the heart that is experiencing the ST segment changesand will best reflect any electrocardiographic (ECG) changes that indicate further damageto the myocardium. The other leads do not reflect the inferior part of the myocardial walland will not provide data about further ischemic changes.

The nurse obtains this information from a patient with prehypertension. Which finding is most important to address with the patient? A) Low dietary fiber intake B) No regular aerobic exercise C) Weight 5 pounds above ideal weight D) Drinks wine with dinner once a week

B) No regular aerobic exercise

Which information collected by the nurse who is admitting a patient with chest pain suggests that the pain is caused by an acute myocardial infarction (AMI)? A) The pain increases with deep breathing. B) The pain has persisted longer than 30 minutes. C) The pain worsens when the patient raises the arms. D) The pain is relieved after the patient takes nitroglycerin.

B) The pain has persisted longer than 30 minutes.

When caring for a patient who has survived a sudden cardiac death (SCD) event and has no evidence of an acute myocardial infarction (AMI), the nurse will anticipate teaching the patient A) that sudden cardiac death events rarely reoccur. B) about the purpose of outpatient Holter monitoring. C) how to self-administer low-molecular-weight heparin. D) to limit activities after discharge to prevent future events.

B) about the purpose of outpatient Holter monitoring.

When auscultating over the patient's abdominal aorta, the nurse hears a humming sound. The nurse documents this finding as a A) thrill. B) bruit. C) heave. D) murmur.

B) bruit.

Amlodipine (Norvasc) is ordered for a patient with newly diagnosed Prinzmetal's (variant) angina. When teaching the patient, the nurse will include the information that amlodipine will A) reduce the "fight or flight" response. B) decrease spasm of the coronary arteries. C) increase the force of myocardial contraction. D) help prevent clotting in the coronary arteries.

B) decrease spasm of the coronary arteries.

In teaching a patient about coronary artery disease, the nurse explains that the changes that occur in this disorder include which of the following (select all that apply)? a. diffuse involvement of plaque formation in coronary veins b. abnormal levels of cholesterol, particularly low-density lipoproteins c. accumulation of lipid and fibrous tissue within the coronary arteries d. development of angina due to decreased blood supply to the heart muscle e. chronic vasoconstriction of coronary arteries leading to permanent vasospasm

B,C,D

When collecting subjective data related to the cardiovascular system, which of the following should be obtained from the patient (select all that apply)? a. annual income b. smoking history c. religious preference d. number of pillows used to sleep e. blood for basic laboratory data

B,C,D

The nurse will suspect that the patient with stable angina is experiencing a side effect of the prescribed metoprolol (Lopressor) if A) the patient is restless and agitated. B) the blood pressure is 190/110 mm Hg. C) the patient complains about feeling anxious. D) the cardiac monitor shows a heart rate of 45.

D) the cardiac monitor shows a heart rate of 45.

Results of a patient's most recent blood work indicate an elevated neutrophil level. You recognize that this diagnostic finding most likely suggests: A. Hypoxemia B. an infection C. risk of hypocoagulation D. an acute thrombotic event

B. An increase in neutrophil count most commonly occurs in response to infection or inflammation. Hypoxemia and coagulation do not directly affect neutrophil production.

You are providing care for older adults on a subacute, geriatric medicine unit. Which of the following effects is aging likely to have on hematologic function of older adults? A. Hypercoagulability B. decreased hemoglobin C. decreased blood volume D. decreased WBC count

B. An individual with type A blood has A antigens, not A antibodies, on his RBCs. An AB transfusion would result in agglutination, but he may be transfused with either type A or type O.

Before starting a transfusion of packed red blood cells for an anemic patient, the nurse would arrange for a peer to monitor his or her other assigned patients for how many minutes when the nurse begins the transfusion? A. 5 B. 15 C. 30 D. 60

B. As part of standard procedure, the nurse remains with the patient for the first 15 minutes after starting a blood transfusion. Patients who are likely to have a transfusion reaction will more often exhibit signs within the first 15 minutes that the blood is infusing.

5) Which of the following is not effective nursing management of heart failure? A. High Fowlers position B. Assisting with rigorous exercise 2x a day C. Daily weights, intake & output monitoring D. Continuous EKG monitoring

B. Assisting with rigorous exercise 2x a day

The correct formula for blood pressure is? A. BP= HR x SVR B. BP=CO x SVR C. BP= SV x CO D. BP= CO x HR

B. BP=CO x SVR CO = Cardiac output (HR X SV) amount of blood pumped in one minute SV = amount of blood pumped in one beat SVR = Systemic vascular resistance

4) To improve gas exchange and oxygenation for a patient with heart failure, what nursing management should be implemented? A. Check vital signs B. Place patient in high Fowlers position. C. Place patient in semi-Fowlers position. D. Administer diuretic.

B. Place patient in high Fowlers position.

Caring for a patient with a diagnosis of polycythemia vera will likely require the nurse to A. Encourage deep-breathing and coughing. B. Assist with or perform phlebotomy at the bedside. C. Teach the patient how to maintain a low-activity lifestyle. D. Perform thorough and regularly scheduled neurologic assessments.

B. Primary polycythemia may often require phlebotomy in order to reduce blood volume. The increased risk of thrombus formation that accompanies the disease requires regular exercises and ambulation. Deep-breathing and coughing exercises do not directly address the etiology or common sequelae of polycythemia, and neurologic manifestations are not typical.

When caring for a patient with metastatic cancer, the nurse notes a hemoglobin level of 8.7 g/dl and hematocrit of 26%. The nurse would place highest priority on initiating interventions that will reduce which of the following? A. thirst B. fatigue C. headache D. abdominal pain

B. The patient with a low hemoglobin and hematocrit is anemic and would be most likely to experience fatigue. Fatigue develops because of the lowered oxygen-carrying capacity that leads to reduced tissue oxygenation to carry out cellular functions.

ST elevation indicates A. myocardial ischemia B. myocardial injury C. myocardial infarction D. myocardial necrosis

B. myocardial injury Know the image

BNP is a major marker of congestive HF and it is released in response to: A. decreased renal perfusion B. ventricular stretch C. development of pulmonary edema D. low preload

B. ventricular stretch

A 30-year-old patient has undergone a splenectomy as a result of injuries suffered in a motor vehicle accident. Which of the following phenomena is likely to result from the absence of the patient's spleen (select all that apply)? Impaired fibrinolysis Increased platelet levels Increased eosinophil levels Fatigue and cold intolerance Impaired immunologic function

B., E.: Splenectomy can result in increased platelet levels and impaired immunologic function as a consequence of the loss of storage and immunologic functions of the spleen. Fibrinolysis, fatigue, and cold intolerance are less likely to result from the loss of the spleen since coagulation and oxygenation are not primary responsibilities of the spleen

16. A home health care patient has recently started taking oral digoxin (Lanoxin) and furosemide (Lasix) for control of heart failure. The patient data that will require the most immediate action by the nurse is if the patient's a. weight increases from 120 pounds to 122 pounds over 3 days. b. liver is palpable 2 cm below the ribs on the right side. c. serum potassium level is 3.0 mEq/L after 1 week of therapy. d. has 1 to 2+ edema in the feet and ankles in the morning.

C Rationale: Hypokalemia potentiates the actions of digoxin and increases the risk for digoxin toxicity, which can cause life-threatening dysrhythmias. The other data indicate that the patient's heart failure requires more effective therapies, but they do not require nursing action as rapidly as the low serum potassium level. Cognitive Level: Application Text Reference: pp. 832-833 Nursing Process: Assessment NCLEX: Physiological Integrity

A 54-year-old male patient who had bladder surgery 2 days ago develops acute decompensated heart failure (ADHF) with severe dyspnea. Which action by the nurse would be indicated first? A Perform a bladder scan to assess for urinary retention. B Restrict the patient's oral fluid intake to 500 mL per day. C Assist the patient to a sitting position with arms on the overbed table. D Instruct the patient to use pursed-lip breathing until the dyspnea subsides.

C Assist the patient to a sitting position with arms on the overbed table. The nurse should place the patient with ADHF in a high Fowler's position with the feet horizontal in the bed or dangling at the bedside. This position helps decrease venous return because of the pooling of blood in the extremities. This position also increases the thoracic capacity, allowing for improved ventilation. Pursed-lip breathing helps with obstructive air trapping but not with acute pulmonary edema. Restricting fluids takes considerable time to have an effect.

What is the priority assessment by the nurse caring for a patient receiving IV nesiritide (Natrecor) to treat heart failure? A Urine output B Lung sounds C Blood pressure D Respiratory rate

C Blood pressure Although all identified assessments are appropriate for a patient receiving IV nesiritide, the priority assessment would be monitoring for hypotension, the main adverse effect of nesiritide.

The nurse prepares to administer digoxin (Lanoxin) 0.125 mg to an 82-year-old man admitted with influenza and a history of chronic heart failure. What should the nurse assess before giving the medication? A Prothrombin time B Urine specific gravity C Serum potassium level D Hemoglobin and hematocrit

C Serum potassium level Serum potassium should be monitored because hypokalemia increases the risk for digoxin toxicity. Changes in prothrombin time, urine specific gravity, and hemoglobin or hematocrit would not require holding the digoxin dose.

After giving a patient the initial dose of oral labetalol (Normodyne) for treatment of hypertension, which action should the nurse take? A) Encourage oral fluids to prevent dry mouth or dehydration. B) Instruct the patient to ask for help if heart palpitations occur. C) Ask the patient to request assistance when getting out of bed. D) Teach the patient that headaches may occur with this medication.

C) Ask the patient to request assistance when getting out of bed.

To assist the patient with coronary artery disease (CAD) in making appropriate dietary changes, which of these nursing interventions will be most effective? A) Instruct the patient that a diet containing no saturated fat and minimal sodium will be necessary. B) Emphasize the increased risk for cardiac problems unless the patient makes the dietary changes. C) Assist the patient to modify favorite high-fat recipes by using monosaturated oils when possible. D) Provide the patient with a list of low-sodium, low-cholesterol foods that should be included in the diet.

C) Assist the patient to modify favorite high-fat recipes by using monosaturated oils when possible.

Three days after a myocardial infarction (MI), the patient develops chest pain that increases when taking a deep breath and is relieved by leaning forward. Which action should the nurse take next? A) Palpate the radial pulses bilaterally. B) Assess the feet for peripheral edema. C) Auscultate for a pericardial friction rub. D) Check the cardiac monitor for dysrhythmias.

C) Auscultate for a pericardial friction rub.

A patient who has had severe chest pain for several hours is admitted with a diagnosis of possible acute myocardial infarction (AMI). Which of these ordered laboratory tests should the nurse monitor to help determine whether the patient has had an AMI? A) Homocysteine B) C-reactive protein C) Cardiac-specific troponin I and troponin T D) High-density lipoprotein (HDL) cholesterol

C) Cardiac-specific troponin I and troponin T

Which action will the nurse in the hypertension clinic take in order to obtain an accurate baseline blood pressure (BP) for a new patient? A) Obtain a BP reading in each arm and average the results. B) Deflate the BP cuff at a rate of 5 to 10 mm Hg per second. C) Have the patient sit in a chair with the feet flat on the floor. D) Assist the patient to the supine position for BP measurements.

C) Have the patient sit in a chair with the feet flat on the floor.

Following an acute myocardial infarction (AMI), a patient ambulates in the hospital hallway. When the nurse is evaluating the patient's response, which of these assessment data would indicate that the exercise level should be decreased? A) BP changes from 118/60 to 126/68 mm Hg. B) Oxygen saturation drops from 100% to 98%. C) Heart rate increases from 66 to 90 beats/minute. D) Respiratory rate goes from 14 to 22 breaths/minute.

C) Heart rate increases from 66 to 90 beats/minute.

During change-of-shift report, the nurse obtains this information about a hypertensive patient who received the first dose of propranolol (Inderal) during the previous shift. Which information indicates that the patient needs immediate intervention? A) The patient's most recent BP reading is 156/94 mm Hg. B) The patient's pulse has dropped from 64 to 58 beats/minute. C) The patient has developed wheezes throughout the lung fields. D) The patient complains that the fingers and toes feel quite cold

C) The patient has developed wheezes throughout the lung fields.

A patient is scheduled for a cardiac catheterization with coronary angiography. Before the test, the nurse informs the patient that A) electrocardiographic (ECG) monitoring will be required for 24 hours after the test. B) it will be important to lie completely still during the procedure. C) a warm feeling may be noted when the contrast dye is injected. D) monitored anesthesia care will be provided during the procedure.

C) a warm feeling may be noted when the contrast dye is injected

Which action will be included in the plan of care when the nurse is caring for a patient who is receiving sodium nitroprusside (Nipride) to treat a hypertensive emergency? A) Organize nursing activities so that the patient has undisturbed sleep for 6 to 8 hours at night. B) Assist the patient up in the chair for meals to avoid complications associated with immobility. C) Use an automated noninvasive blood pressure machine to obtain frequent BP measurements. D) Place the patient on NPO status to prevent aspiration caused by nausea and the associated vomiting.

C) Use an automated noninvasive blood pressure machine to obtain frequent BP measurements.

The nurse working in the heart failure clinic will know that teaching for a 74-year-old patient with newly diagnosed heart failure has been effective when the patient A) uses an additional pillow to sleep when feeling short of breath at night. B) tells the home care nurse that furosemide (Lasix) is taken daily at bedtime. C) calls the clinic when the weight increases from 124 to 130 pounds in a week. D) says that the nitroglycerin patch will be used for any chest pain that develops.

C) calls the clinic when the weight increases from 124 to 130 pounds in a week.

The nurse teaches the patient being evaluated for rhythm disturbances with a Holter monitor to A) exercise more than usual while the monitor is in place. B) remove the electrodes when taking a shower or tub bath. C) keep a diary of daily activities while the monitor is worn. D) connect the recorder to a telephone transmitter once daily.

C) keep a diary of daily activities while the monitor is worn.

Which of these statements made by the patient about the therapeutic lifestyle changes (TLC) diet indicates that further teaching is needed? A. "I will switch from whole milk to 1% or nonfat milk." B. "I like fresh salmon and I will plan to eat it more often." C. "I will miss being able to eat peanut butter sandwiches." D. "I can have a cup of coffee with breakfast if I want one."

C. "I will miss being able to eat peanut butter sandwiches." Patients can still have peanut butter

Based on the pathophysiology of Mr. Cameron's condition, his coronary artery is blocked by A. 30% B. 50% C. 70%

C. 70% Must be blocked at least 70% to have angina

Longstanding hypertension can reset what receptor A. Chemoreceptors B. Osmoreceptors C. Baroreceptors D. Hemoreceptors

C. Baroreceptors refers to pressure

What misconceptions about hypertension should be corrected? A. "Hyper" and hypertension are the same diagnosis. B. Hypertension is called the "silent" killer because it does not always cause visible symptoms. C. Erectile dysfunction is something all men have to deal with when taking antihypertensive medications. D. Hypertension can not be treated without medication

C. Erectile dysfunction is something all men have to deal with when taking antihypertensive medications. - False D. Hypertension can not be treated without medication. - False DASH diet

To prepare a patient for the stress test, you should: A. Have him NPO after midnight B. Allow him to have a light breakfast with a cup of coffee C. Hold his cardizem 24 hours before the test. D. shave Mr. Cameron's groin area.

C. Hold his cardizem 24 hours before the test. Stress test is done on a treadmill No NPO No coffee Hold meds for 24 hrs before No need to shave groin area

If a patient is in the diuretic phase of AKI, you must monitor for which serum electrolyte imbalances? A. Hyperkalemia and hyponatremia B. Hyperkalemia and hypernatremia C. Hypokalemia and hyponatremia D. Hypokalemia and hypernatremia

C. Hypokalemia and hyponatremia In the diuretic phase of AKI, the kidneys have recovered their ability to excrete wastes but not to concentrate the urine. Hypovolemia and hypotension can result from massive fluid losses. Because of the large losses of fluid and electrolytes, the patient must be monitored for hyponatremia, hypokalemia, and dehydration.

What is a common side effect of Nitroglycerin? A. Sleepyness B. Itchyness C. Hypotension D. Brachacardia

C. Hypotension

A patient is recovering in the intensive care unit (ICU) after receiving a kidney transplant approximately 24 hours earlier. What is an expected assessment finding for this patient during the early stage of recovery? A. Hypokalemia B. Hyponatremia C. Large urine output D. Leukocytosis with cloudy urine output

C. Large urine output Patients frequently experience diuresis in the hours and days immediately after kidney transplantation. Electrolyte imbalances and signs of infection are unexpected findings that warrant prompt intervention.

How should you assess the patency of a newly placed arteriovenous graft for dialysis? A. Irrigate the graft daily with low-dose heparin. B. Monitor for any increase in blood pressure in the affected arm. C. Listen with a stethoscope over the graft for presence of a bruit. D. Frequently monitor the pulses and neurovascular status distal to the graft.

C. Listen with a stethoscope over the graft for presence of a bruit. A thrill can be felt by palpating the area of anastomosis of the arteriovenous graft, and a bruit can be heard with a stethoscope. The bruit and thrill are created by arterial blood rushing into the vein.

The most important long-term goal for a client with hypertension would be to: A. Learn how to avoid stress B. Explore a job change or early retirement. C. Make a commitment to long-term therapy. D. Lose weight.

C. Make a commitment to long-term therapy. Diet, exercise, avoid stress, lose weight for the long-term High in saturated fat: meat, dairy, butter, eggs LDL is the bad lipid HDL is the good lipid

Diagnosis of hypertension is obtained by A. One high blood pressure reading at any clinic visit B. Average of one blood pressure on any clinic visit. C. Two or more blood pressure readings at two or more office visits. D. Two high blood pressure readings taken less than one minute apart on the same visit

C. Two or more blood pressure readings at two or more office visits.

You are caring for a patient receiving continuous replacement therapy and notice that the filtrate is blood tinged. What is your priority action? A. Place the patient in Trendelenburg position. B. Initiate a peripheral intravenous line. C. Suspend treatment immediately. D. Administer vitamin K (Aquamephyton) per order.

C. Suspend treatment immediately. The ultrafiltrate should be clear yellow, and specimens may be obtained for evaluation of serum chemistries. If the ultrafiltrate becomes bloody or blood tinged, a possible rupture in the filter membrane should be suspected, and treatment is suspended immediately to prevent blood loss and infection.

You should terminate stress test if Mr. Cameron develops A. HR of 120 B. sweating C. chest pain D. RR of 25

C. chest pain

The occlusion/obstruction of coronary arteries are determined using: A. echocardiogram B. ECG C. coronary angiogram D. chest x-ray

C. coronary angiogram Echocardiogram is noninvasive, with ultrasound. Prepare patient laying to the right. ECG - electrolyte conductivity of the heart Coronary angiogram - uses dye and contrast CXR - see structure of heart, for heart enlargement

Measures indicated in the conservative therapy of CKD include A. decreased fluid intake, carbohydrate intake, and protein intake. B. increased fluid intake; decreased carbohydrate intake and protein intake. C. decreased fluid intake and protein intake; increased carbohydrate intake. D. decreased fluid intake and carbohydrate intake; increased protein intake.

C. decreased fluid intake and protein intake; increased carbohydrate intake. Water and any other fluids are not routinely restricted in the pre-end-stage renal disease (ESRD) stages. Patients on hemodialysis have a more restricted diet than patients receiving peritoneal dialysis. For those receiving hemodialysis, as their urinary output diminishes, fluid restrictions are enhanced. Intake depends on the daily urine output. Generally, 600 mL (from insensible loss) plus an amount equal to the previous day's urine output is allowed for a patient receiving hemodialysis. Patients are advised to limit fluid intake so that weight gains are no more than 1 to 3 kg between dialyses (interdialytic weight gain). For the patient who is undergoing dialysis, protein is not routinely restricted. The beneficial role of protein restriction in CKD stages 1 through 4 as a means to reduce the decline in kidney function is being studied. Historically, dietary counseling often encouraged restriction of protein for CKD patients. Although there is some evidence that protein restriction has benefits, many patients find these diets difficult to adhere to. For CKD stages 1 through 4, many clinicians encourage a diet with normal protein intake. However, you should teach patients to avoid high-protein diets and supplements because they may overstress the diseased kidneys.

D

Complications of transfusions that can be decreased by the use of leukocyte depletion or reduction of RBC transfusion are A. chills and hemolysis B. leukostasis and neutrophilia C. fluid overload and pulmonary edema D. transmission of cytomegalovirus and fever

The nurse prepares a patient for synchronized cardioversion knowing that cardioversion differs from defibrillation in that a. defibrillation requires a lower dose of electrical charge b. cardioversion is indicated to treat atrial bradydysrhythmias c. defibrillation is synchronized to deliver a shock during the QRS complex d. patients should be sedates if cardioversion is done on a non-emergency basis

D

When evaluating a patient's knowledge regarding a low-sodium, low-fat cardiac diet, the nurse recognizes additional teaching is needed when the patient selects which of the following food choices? A) a. Baked flounder B) b. Angel food cake C) c. Baked potato with margarine D) d. Canned chicken noodle soup

D

In reviewing medication instructions with a patient being discharged on antihypertensive medications, which of the following statements would be most appropriate for the nurse to make when discussing atenolol (Tenormin)? A) a. "A fast heart rate is a side effect to watch for while taking atenolol." B) b. "Stop the drug and notify your doctor if you experience any nausea or vomiting." C) c. "Because this drug may affect the lungs in large doses, it may also help your breathing." D) d. "Make position changes slowly, especially when going from a lying down to a standing position."

D

13. The nurse plans discharge teaching for a patient with chronic heart failure who has prescriptions for digoxin (Lanoxin), hydrochlorothiazide (HydroDIURIL), and a potassium supplement. Appropriate instructions for the patient include a. avoid dietary sources of potassium because too much can cause digitalis toxicity. b. take the pulse rate daily and never take digoxin if the pulse is below 60 beats/min. c. take the hydrochlorothiazide before bedtime to maximize activity level during the day. d. notify the health care provider immediately if nausea or difficulty breathing occurs.

D Rationale: Difficulty breathing is an indication of acute decompensated heart failure and suggests that the medications are not achieving the desired effect. Nausea is an indication of digoxin toxicity and should be reported so that the provider can assess the patient for toxicity and adjust the digoxin dose, if necessary. Digoxin toxicity is potentiated by hypokalemia, rather than hyperkalemia. Patients should be taught to check their pulse daily before taking the digoxin and, if the pulse is less than 60, to call their provider before taking the digoxin. Diuretics should be taken early in the day to avoid sleep disruption. Cognitive Level: Application Text Reference: p. 835 Nursing Process: Implementation NCLEX: Health Promotion and Maintenance

The nurse has just finished teaching a hypertensive patient about the newly prescribed quinapril (Accupril). Which patient statement indicates that more teaching is needed? A) "The medication may not work as well if I take any aspirin." B) "The doctor may order a blood potassium level occasionally." C) "I will call the doctor if I notice that I have a frequent cough." D) "I won't worry if I have a little swelling around my lips and face."

D) "I won't worry if I have a little swelling around my lips and face."

Which action should the nurse take when preparing for cardioversion of a patient with supraventricular tachycardia who is alert and has a blood pressure of 110/66 mm Hg? A) Turn the synchronizer switch to the "off" position. B) Perform cardiopulmonary resuscitation (CPR) until the paddles are in correct position. C) Set the defibrillator/cardioverter energy to 300 joules. D) Administer a sedative before cardioversion is implemented.

D) Administer a sedative before cardioversion is implemented. When a patient has a nonemergency cardioversion, sedation is used just before the procedure. The synchronizer switch is turned on for cardioversion. The initial level of joules for cardioversion is low (e.g., 50). CPR is not indicated for this patient

To determine the effects of therapy for a patient who is being treated for heart failure, which laboratory result will the nurse plan to review? A) Myoglobin B) Homocysteine (Hcy) C) Low-density lipoprotein (LDL) D) B-type natriuretic peptide (BNP)

D) B-type natriuretic peptide (BNP)

A patient's cardiac monitor shows sinus rhythm, rate 60 to 70. The P-R interval is 0.18 seconds at 1:00 AM, 0.20 seconds at 2:30 PM, and 0.23 seconds at 4:00 PM. Which action should the nurse take at this time? A) Prepare for possible temporary pacemaker insertion. B) Administer atropine sulfate 1 mg IV per agency protocol. C) Document the patient's rhythm and assess the patient's response to the rhythm. D) Call the health care provider before giving the prescribed metoprolol (Lopressor).

D) Call the health care provider before giving the prescribed metoprolol (Lopressor). The patient has progressive first-degree atrioventricular (AV) block,and the β- blocker should be held until discussing the medication with the health care provider. Documentation and assessment are appropriate but not fully adequateresponses. The patient with first-degree AV block usually is asymptomatic, and a pacemaker is not indicated. Atropine is sometimes used for symptomatic bradycardia, butthere is no indication that this patient is symptomatic.

A patient has just been diagnosed with hypertension and has a new prescription for captopril (Capoten). Which information is important to include when teaching the patient? A) Check BP daily before taking the medication. B) Increase fluid intake if dryness of the mouth is a problem. C) Include high-potassium foods such as bananas in the diet. D) Change position slowly to help prevent dizziness and falls.

D) Change position slowly to help prevent dizziness and falls.

While doing the admission assessment for a thin 72-year-old patient, the nurse observes pulsation of the abdominal aorta in the epigastric area. Which action should the nurse take? A) Notify the hospital rapid response team. B) Instruct the patient to remain on bed rest. C) Teach the patient about aortic aneurysms. D) Document the finding in the patient chart.

D) Document the finding in the patient chart.

A patient with a non-ST-segment-elevation myocardial infarction (NSTEMI) is receiving heparin. What is the purpose of the heparin? A) Platelet aggregation is enhanced by IV heparin infusion. B) Heparin will dissolve the clot that is blocking blood flow to the heart. C) Coronary artery plaque size and adherence are decreased with heparin. D) Heparin will prevent the development of new clots in the coronary arteries.

D) Heparin will prevent the development of new clots in the coronary arteries.

When analyzing the waveforms of a patient's electrocardiogram (ECG), the nurse will need to investigate further upon finding a A) T wave of 0.16 second. B) P-R interval of 0.18 second. C) Q-T interval of 0.34 second. D) QRS interval of 0.14 second.

D) QRS interval of 0.14 second. Because the normal QRS interval is 0.04 to 0.10 seconds, the patient's QRS interval of 0.14 seconds indicates that the conduction through the ventricular conduction system is prolonged. The P-R interval, Q-T interval, and T wave interval are within the normalrange.

During a physical examination of a patient, the nurse palpates the point of maximal impulse (PMI) in the sixth intercostal space lateral to the left midclavicular line. The most appropriate action for the nurse to take next will be to A) document that the PMI is in the normal anatomic location. B) ask the patient about risk factors for coronary artery disease. C) auscultate both the carotid arteries for the presence of a bruit. D) assess the patient for symptoms of left ventricular hypertrophy.

D) assess the patient for symptoms of left ventricular hypertrophy.

When developing a health teaching plan for a 60-year-old man with the following risk factors for coronary artery disease (CAD), the nurse should focus on the A) family history of coronary artery disease. B) increased risk associated with the patient's gender. C) high incidence of cardiovascular disease in older people. D) elevation of the patient's serum low density lipoprotein (LDL) level.

D) elevation of the patient's serum low density lipoprotein (LDL) level.

A 52-year-old patient who has no previous history of hypertension or other health problems suddenly develops a BP of 188/106 mm Hg. After reconfirming the BP, it is appropriate for the nurse to tell the patient that A) a BP recheck should be scheduled in a few weeks. B) the dietary sodium and fat content should be decreased. C) there is an immediate danger of a stroke and hospitalization will be required. D) more diagnostic testing may be needed to determine the cause of the hypertension.

D) more diagnostic testing may be needed to determine the cause of the hypertension.

The nurse obtains a monitor strip on a patient who has had a myocardial infarction and makes the following analysis: P wave not apparent, ventricular rate 162, R-R interval regular, P-R interval not measurable, and QRS complex wide and distorted, QRS duration 0.18 second. The nurse interprets the patient's cardiac rhythm as A) atrial fibrillation. B) sinus tachycardia. C) ventricular fibrillation. D) ventricular tachycardia.

D) ventricular tachycardia. The absence of P waves, wide QRS, rate >150, and the regularity of the rhythm indicate ventricular tachycardia. Atrial fibrillation is grossly irregular, has a narrow QRS configuration, and has fibrillatory atrial activity. Sinus tachycardia has P waves.Ventricular fibrillation is irregular and does not have a consistent QRS duration

You are preparing to care for a patient following the cardiac catheterization performed through the femoral artery. After the catheterization you should place the patient on: A. Bedrest in high Fowler's position B. Bedrest with bathroom privileges only C. Bedrest with head elevation at 60 degrees D. Bedrest with head elevation no greater than 30 degrees

D. Bedrest with head elevation no greater than 30 degrees

The nurse is caring for a patient who is to receive a transfusion of two units of packed red blood cells. After obtaining the first unit from the blood bank, the nurse would ask which of the following health team members in the nurses' station to assist in checking the unit before administration? A. unit secretary B. a phlebotomist C. a physician's assistant D. another RN

D. Before hanging a transfusion, the registered nurse must check the unit with another RN or with a licensed practical (vocational) nurse, depending on agency policy.

2) Which patient should the nurse attend to first? A. Diabetic patient experiencing an increased blood sugar at 8am in the morning. B. Influenza patient experiencing a fever spike of 100.0 F C. Pneumonia patient experiencing productive cough with green sputum. D. Heart failure patient experiencing abnormal (symptomatic) Bradycardia at rest.

D. Heart failure patient experiencing abnormal (symptomatic) Bradycardia at rest.

Since she is allergic to Lisinopril, an alternative would be: A. Atenolol (Tenormin) B. Simvastatin (Zocor) C. Amlodipine (Norvasc) D. Losartan (Cozaar)

D. Losartan (Cozaar) If patient is allergic to ACE use ARB - Losartan. A. Atenolol (Tenormin) is a Beta Blocker check BP and HR B. Simvastatin (Zocor) is statin, for hyperlipidemia, monitor for liver function C. Amlodipine (Norvasc) - Calcium Channel Blocker, will dilate blood vessels D. Losartan (Cozaar) - an ARB

Mr. Cameron is diagnosed with type 2 diabetes mellitus. Which of the following medications would need to be withheld after the procedure? A. Regular insulin B. Glipizide (Glucotrol) C. Repaglinide (Prandin) D. Metformin (Glucophage)

D. Metformin (Glucophage) For two days after procedure check liver function

8) A nurse is assessing the client with left sided heart failure. The client states that he needs to use 3 pillows under the head and chest at night to be able to breathe comfortably while sleeping. The documents that the client is experience: A. ORTHOPNEA B. DYSPNEA at rest C. DYSPNEA on exertion D. Paroxysmal nocturnal dyspnea

D. Paroxysmal nocturnal dyspnea

You are preparing to administer a dose of PhosLo to a patient with chronic kidney disease (CKD). This medication should have a beneficial effect on which laboratory value? A. Sodium B. Potassium C. Magnesium D. Phosphorus

D. Phosphorus Phosphorus and calcium have inverse or reciprocal relationships, meaning that when phosphorus levels are high, calcium levels tend to be low. Administration of calcium should help to reduce a patient's abnormally high phosphorus level, as seen in CKD.

When caring for a patient during the oliguric phase of acute kidney injury, what would be an appropriate nursing intervention? A. Weigh patient three times weekly B. Increase dietary sodium and potassium C. Provide a low-protein, high-carbohydrate diet D. Restrict fluids according to the previous day's fluid loss

D. Restrict fluids according to the previous day's fluid loss Patients in the oliguric phase of acute kidney injury have fluid volume excess with potassium and sodium retention. They will need to have dietary sodium, potassium, and fluids restricted. Daily fluid intake is based on the previous 24-hour fluid loss (measured output plus 600 mL for insensible loss). The diet also needs to provide adequate, not low, protein intake to prevent catabolism. The patient should also be weighed daily, not just three times per week.

Which of the following is not a classification of hypertension? A. Primary Hypertension B. Pseudohypertension C. Isolated systolic hypertension D. Stage 3 hypertension

D. Stage 3 hypertension Pseudohypertension is a thing, i.e. white coat hypertension

1) Which of the following is not found in acute decompensated heart failure? A. ORTHOPNEA B. TACHYCARDIA C. DYSPNEA D. UNPRODUCTIVE COUGH

D. UNPRODUCTIVE COUGH

The patient admitted to the intensive care unit after a motor vehicle accident has been diagnosed with AKI. Which finding indicates the onset of oliguria resulting from AKI? A. Urine output less than 1000 mL for the past 24 hours B. Urine output less than 800 mL for the past 24 hours C. Urine output less than 600 mL for the past 24 hours D. Urine output less than 400 mL for the past 24 hours

D. Urine output less than 400 mL for the past 24 hours The most common initial manifestation of AKI is oliguria, a reduction to urine output to less than 400 mL/day.

When assessing lab values on a patient admitted with septicemia, the nurse would expect to find A. Increased platelets. B. Decreased red blood cells. C. Decreased erythrocyte sedimentation rate (ESR). D. Increased bands in the white blood cell (WBC) differential (shift to the left).

D. When infections are severe, such as in septicemia, more granulocytes are released from the bone marrow as a compensatory mechanism. To meet the increased demand, many young, immature polymorphonuclear neutrophils (bands) are released into circulation. WBCs are usually reported in order of maturity, with the less mature forms on the left side of a written report. Hence, the term "shift to the left" is used to denote an increase in the number of bands.

What are signs and symptoms of hypertension?

It's asymptomatic

Based on the ECG findings the diagnosis is angina. The physician prescribes "MONA". What does MONA stand for?

Morphine Oxygen Nitroglycerin Aspirin

Serum troponin is ordered. The result is 0.4 ng/dL. What is the interpretation of this value?

Normal troponin level 0.0-0.4 Has to be taken 3 times to be conclusive

Stages of hypertension

Normal: 120/80 Prehypertension: 120-129/80-89 Stage 1: 140-159/90-99 Stage 2: 160+/100+

The patient with CKD is receiving dialysis, and the nurse observes excoriations on the patient's skin. What pathophysiologic changes in CKD can contribute to this finding (select all that apply)? a. Dry skin b. Sensory neuropathy c. Vascular calcifications d. Calcium-phosphate skin deposits e. Uremic crystallization from high BUN

a, b, d. Pruritus is common in patients receiving dialysis. It causes scratching from dry skin, sensory neuropathy, and calcium-phosphate deposition in the skin. Vascular calcifications contribute to cardiovascular disease, not to itching skin. Uremic frost rarely occurs without BUN levels greater than 200 mg/dL, which should not occur in a patient on dialysis; urea crystallizes on the skin and also causes pruritis.

Number the following in the order of the phases of exchange in PD. Begin with 1 and end with 3. a. Drain b. Dwell c. Inflow

a. 3; b. 2; c. 1

Priority Decision: During the immediate postoperative care of a recipient of a kidney transplant, what should the nurse expect to do? a. Regulate fluid intake hourly based on urine output. b. Monitor urine-tinged drainage on abdominal dressing. c. Medicate the patient frequently for incisional flank pain. d. Remove the urinary catheter to evaluate the ureteral implant

a. Fluid and electrolyte balance is critical in the transplant recipient patient, especially because diuresis often begins soon after surgery. Fluid replacement is adjusted hourly based on kidney function and urine output. Urine-tinged drainage on the abdominal dressing may indicate leakage from the ureter implanted into the bladder and the health care provider should be notified. The donor patient may have a flank or laparoscopic incision(s) where the kidney was removed. The recipient has an abdominal incision where the kidney was placed in the iliac fossa. The urinary catheter is usually used for 2 to 3 days to monitor urine output and kidney function.

The patient has a thoracic spinal cord lesion and incontinence that occurs equally during the day and night. What type of incontinence is this patient experiencing? a. Reflex incontinence b. Overflow incontinence c. Functional incontinence d. Incontinence after trauma

a. Reflex incontinence occurs with no warning, equally during the day and night, and with spinal cord lesions above S2. Overflow incontinence is when the pressure of urine in the overfull bladder overcomes sphincter control and is caused by bladder or urethral outlet obstruction. Functional incontinence is loss of urine resulting from cognitive, functional, or environmental factors. Incontinence after trauma or surgery occurs when fistulas have occurred or after a prostatectomy.

Nursing assessment of Acute Pyelonephritis

a. Subjective Data-Health History i. Previous UTIs, calculi, stasis, retention, pregnancy, STDs, bladder cancer ii. Antibiotics, anticholinergics, antispasmodics iii. Urologic instrumentation iv. Urinary hygiene v. Nausea, vomiting, anorexia, chills, nocturia, frequency, urgency vi. Suprapubic or lower back pain, bladder spasms, dysuria, burning on urination b. Objective data i. Fever ii. Hematuria, foul-smelling urine, tender, enlarged kidney iii. Leukocytosis, positive findings for bacteria, WBCs, RBCs, pyuria, ultrasound, CT scan, IVP

Evaluation of Acute Pyelonephritis

a. Use of nonanalgesic relief measures b. Appropriate use of analgesics c. Passage of urine without urgency d. Urine free of blood e. Adequate intake of fluids

*9. In planning nursing interventions to increase bladder control in the patient with urinary incontinence, the nurse includes:* a. teaching the patient to use Kegel exercises b.clamping and releasing a catheter to increase bladder tone c.teaching the patient biofeedback mechanisms to suppress the urge to void d.counseling the patient concerning choices of incontinence containment devices

a. teaching the patient to use Kegel exercises

Which infection is asymptomatic in the male patient at first and then progresses to cystitis, frequent urination, burning on voiding, and epididymitis? a. Urosepsis b. Renal tuberculosis c. Urethral diverticula d. Goodpasture syndrome

b. The manifestations of renal tuberculosis are described. Urosepsis is when the UTI has spread systemically. Urethral diverticula are localized out pouching of the urethra and occur more often in women. Goodpasture syndrome manifests with flu-like symptoms with pulmonary symptoms that include cough, shortness of breath, and pulmonary insufficiency and renal manifestations that include hematuria, weakness, pallor, anemia, and renal failure.

During assessment of the patient who has a nephrectomy, what should the nurse expect to find? a. Shallow, slow respirations b. Clear breath sounds in all lung fields c. Decreased breath sounds in the lower left lobe d. Decreased breath sounds in the right and left lower lobes

b. A nephrectomy incision is usually in the flank, just below the diaphragm or in the abdominal area. Although the patient is reluctant to breathe deeply because of incisional pain, the lungs should be clear. Decreased sounds and shallow respirations are abnormal and would require intervention.

A patient has a right ureteral catheter placed following a lithotripsy for a stone in the ureter. In caring for the patient after the procedure, what is an appropriate nursing action? a. Milk or strip the catheter every 2 hours. b. Measure ureteral urinary drainage every 1 to 2 hours. c. Irrigate the catheter with 30-mL sterile saline every 4 hours. d. Encourage ambulation to promote urinary peristaltic action

b. Output from ureteral catheters must be monitored every 1 to 2 hours because an obstruction will cause over distention of the renal pelvis and renal damage. The renal pelvis has a capacity of only 3 to 5 mL and if irrigation is ordered, no more than 5 mL of sterile saline is used. The patient with a ureteral catheter is usually kept on bed rest until specific orders for ambulation are given. Suprapubic tubes may be milked to prevent obstruction of the catheter by sediment and clots.

In replying to a patient's questions about the seriousness of her chronic kidney disease (CKD), the nurse knows that the stage of CKD is based on what? a. Total daily urine output b. Glomerular filtration rate c. Degree of altered mental status d. Serum creatinine and urea levels

b. Stages of chronic kidney disease are based on the GFR. No specific markers of urinary output, mental status, or azotemia classify the degree of chronic kidney disease (CKD).

With which diagnosis will the patient benefit from being taught to do self-catheterization? a. Renal trauma b. Urethral stricture c. Renal artery stenosis d. Accelerated nephrosclerosis

b. The patient with urethral stricture will benefit from being taught to dilate the urethra by self-catheterization every few days. Renal trauma is treated related to the severity of the injury with bed rest, fluids, and analgesia. Renal artery stenosis includes control of hypertension with possible surgical revascularization. Accele aged nephrosclerosis is associated with malignant hypertension that must be aggressively treated as well as monitoring kidney function.

A patient on hemodialysis develops a thrombus of a subcutaneous arteriovenous (AV) graft, requiring its removal. While waiting for a replacement graft or fistula, the patient is most likely to have what done for treatment? a. Peritoneal dialysis b. Peripheral vascular access using radial artery c. Silastic catheter tunneled subcutaneously to the jugular vein d. Peripherally inserted central catheter (PICC) line inserted into subclavian vein

c. A more permanent, soft, flexible Silastic double-lumen catheter is used for long-term access when other forms of vascular access have failed. These catheters are tunneled subcutaneously and have Dacron cuffs that prevent infection from tracking along the catheter.

A patient who is complaining of a "racing" heart and nervousness comes to theemergency department. The nurse places the patient on a cardiac monitor and obtains thefollowing electrocardiographic (ECG) tracing. <<see image>> Which action should the nurse take next? a. Get ready to perform electrical cardioversion. b. Have the patient perform the Valsalva maneuver. c. Obtain the patient's blood pressure and oxygen saturation. d. Prepare to give β-blocker medication to slow the heart rate.

c. Obtain the patient's blood pressure and oxygen saturation. The patient has sinus tachycardia, which may have multiple etiologies such as pain, dehydration, anxiety, and myocardial ischemia; further assessment is needed before determining the treatment. Vagal stimulation or β-blockade may be used after further assessment of the patient. Electrical cardioversion is used for some tachydysrhythmias, but would not be used for sinus tachycardia.

For a patient with CKD the nurse identifies a nursing diagnosis of risk for injury: fracture related to alterations in calcium and phosphorus metabolism. What is the pathologic process directly related to the increased risk for fractures? a. Loss of aluminum through the impaired kidneys b. Deposition of calcium phosphate in soft tissues of the body c. Impaired vitamin D activation resulting in decreased GI absorption of calcium d. Increased release of parathyroid hormone in response to decreased calcium levels

c. The calcium-phosphorus imbalances that occur in CKD result in hypocalcemia, from a deficiency of active vitamin D and increased phosphorus levels. This leads to an increased rate of bone remodeling with a weakened bone matrix. Aluminum accumulation is also believed to contribute to the osteomalacia. Osteitis fibrosa involves replacement of calcium in the bone with fibrous tissue and is primarily a result of elevated levels of parathyroid hormone resulting from hypocalcemia.

Which characteristics are associated with urge incontinence (select all that apply)? a. Treated with Kegel exercises b. Found following prostatectomy c. Common in postmenopausal women f. Bladder contracts by reflex, overriding central inhibition d. Involuntary urination preceded by urgency e. Caused by overactivity of the detrusor muscle

d, e, f. Urge incontinence is involuntary urination preceded by urgency caused by overactivity of the detrusor muscle when the bladder contracts by reflex, which overrides central inhibition. Treatment includes treating the underlying cause and retraining the bladder with urge suppression, anticholinergic drugs, or containment devices. The other options are characteristic of stress incontinence. Patients may have a combination of urge and stress incontinence.

Priority Decision: What is the most appropriate snack for the nurse to offer a patient with stage 4 CKD? a. Raisins b. Ice cream c. Dill pickles d. Hard candy

d. A patient with CKD may have unlimited intake of sugars and starches (unless the patient is diabetic) and hard candy is an appropriate snack and may help to relieve the metallic and urine taste that is common in the mouth. Raisins are a high-potassium food. Ice cream contains protein and phosphate and counts as fluid. Pickled foods have high sodium content.

Acute tubular necrosis (ATN) is the most common cause of intrarenal AKI. Which patient is most likely to develop ATN? a. Patient with diabetes mellitus b. Patient with hypertensive crisis c. Patient who tried to overdose on acetaminophen d. Patient with major surgery who required a blood transfusion

d. Acute tubular necrosis (ATN) is primarily the result of ischemia, nephrotoxins, or sepsis. Major surgery is most likely to cause severe kidney ischemia in the patient requiring a blood transfusion. A blood transfusion hemolytic reaction produces nephrotoxic injury if it occurs. Diabetes mellitus, hypertension, and acetaminophen overdose will not contribute to ATN

Which disease causes connective tissue changes that cause glomerulonephritis? a. Gout b. Amyloidosis c. Diabetes mellitus d. Systemic lupus erythematosus

d. Systemic lupus erythematosus causes connective tissue damage that affects the glomerulus. Gout deposits uric acid crystals in the kidney. Amyloidosis deposits hyaline bodies in the kidney. Diabetes mellitus causes microvascular damage affecting the kidney.

A patient with suprapubic pain and symptoms of urinary frequency and urgency has two negative urine cultures. What is one assessment finding that would indicate interstitial cystitis? a. Residual urine greater than 200 mL b. A large, atonic bladder on urodynamic testing c. A voiding pattern that indicates psychogenic urinary retention d. Pain with bladder filling that is transiently relieved by urination

d. The symptoms of interstitial cystitis (IC) imitate those of an infection of the bladder but the urine is free of infectious agents. Unlike a bladder infection, the pain with IC increases as urine collects in the bladder and is temporarily relieved by urination. Acidic urine is very irritating to the bladder in IC and the bladder is small but urinary retention is not common.

Which of the following recurring conditions most commonly occurs in clients with cardiomyopathy?

heart failure

Describe how does each risk factor can lead to HF?

hypertension will increase after load

Septal involvement occurs in which type of cardiomyopathy?

hypertrophic

What are the two most common side effects of NTG?

hypotension and headache Dizziness, headaches, blurred vision, hypotension and tachycardia.

Nursing Care of a UTI

i. Formulate nursing diagnoses based on collected subjective and objective data 1. impaired urinary elimination 2. knowledge deficit 3. pain 4. To prevent future infections... a. readiness for enhanced self-care management (not a problem yet but you are trying to prevent one from occurring) ii. Interventions 1. Ensure adequate fluid intake 2. For discomfort-application of heat to suprapubic area or to back, warm shower 3. Importance of taking antibiotic exactly as prescribed and finishing prescription 4. Report extended symptoms if not resolved

T wave inversion

ischemia

A nurse is caring for a client dx with complete heart block who has a demand pacemaker inserted? The pace is set for 72 bpm. Which EKG pattern indicates to the nurse a failure to capture? QRS complexes occuring at 73bpm and no sharp spikes sharp spikes at 72; QRS at 50 P waves at 78: QRS at 50 QRS at 100 bpm

sharp spikes at 72 bpm; QRS at 50

The healthcare provider is examining the electrocardiogram (EKG) of a patient and notes the PR interval is 6 small boxes in length. What is the significance of this finding? Stress is causing increased sympathetic stimulation. This should be documented as an expected finding. There may be some scar tissue in one of the ventricles. There may be a delay in the conduction through the AV node.

there may be a delay


Set pelajaran terkait

Study Guide- Introduction and Part 1

View Set

EAQ N4510 Respiratory Summer '20

View Set

Quiz 9 Consumer's and Producer's Surplus

View Set

Kantor Pos - Leçon 5 (Ibu Chandra)

View Set

Washington Life and Health (Disability)

View Set

Spanish 1 Final oped ended questions

View Set

Europe During WWI Map (Allies, Central Powers, and Neutral Countries)

View Set